Download Module 3 Metabolism of carbohydrates, lipids

Document related concepts

Point mutation wikipedia , lookup

Lactate dehydrogenase wikipedia , lookup

Nucleic acid analogue wikipedia , lookup

Digestion wikipedia , lookup

Basal metabolic rate wikipedia , lookup

Adenosine triphosphate wikipedia , lookup

Enzyme wikipedia , lookup

Oxidative phosphorylation wikipedia , lookup

Peptide synthesis wikipedia , lookup

Evolution of metal ions in biological systems wikipedia , lookup

Metalloprotein wikipedia , lookup

Proteolysis wikipedia , lookup

Genetic code wikipedia , lookup

Glucose wikipedia , lookup

Butyric acid wikipedia , lookup

Phosphorylation wikipedia , lookup

Hepoxilin wikipedia , lookup

Ketosis wikipedia , lookup

Amino acid synthesis wikipedia , lookup

Biosynthesis wikipedia , lookup

Glyceroneogenesis wikipedia , lookup

Citric acid cycle wikipedia , lookup

Fatty acid synthesis wikipedia , lookup

Metabolism wikipedia , lookup

Glycolysis wikipedia , lookup

Fatty acid metabolism wikipedia , lookup

Biochemistry wikipedia , lookup

Transcript
Module 3. Metabolism of carbohydrates, lipids, amino acids and its regulation.
Test questions in text form
1. In which of the below-mentioned glycolysis reactions substrate level phosphorylation
takes place?
A. * pyruvate kinase
B. aldolase
C. phosphofructokinase
D. lactate dehydrogenase
E. hexokinase
2. Name the glycolysis reactions, flowing with the formation of ATP:
A. hexokinase, enolase
B. enolase, aldolase
C. * phosphoglycerate kinase, pyruvate kinase
D. pyruvate kinase, lactate dehydrogenase
E. hexokinase, phosphofructokinase
3. The concentration of blood lactate is normal:
A. 4,4-6,6 mmol/l
B. 3,3-5,5 mmol/l
C. 4,4-5,5 mmol/l
D. * 1-2 mmol/l
E. 5,5-6,6 mmol/l
4. The preparatory stage of glycolysis ends with the formation of:
A. fructose-1,6-diphosphate
B. glucose-6-phosphate
C. * two trioses (dihydroxyacetone phosphate and glyceraldehydetryphosphate)
D. 2-phosphoglycerate
E. phosphoenolpyruvate
5. Which glycolysis enzyme requires insulin as activator?
A. * hexokinase
B. glucose-6-phosphatase, phosphoglucomutase
C. glucose-6-phosphate-isomerase, aldolase
D. aldolase, lactate dehydrogenase
E. phosphoglucomutase, aldolase
6. A basic role in digestion of carbohydrates in a digestive tract belongs following enzymes:
A. Alfa-amylase, hexokinase, lactase
B. Maltase, sucrase, enterokinase
C. * Lactase, alfa-amylase, sucrase
D. Sucrase, maltase, glycogen phosphorylase
E. Aldolase, glucokinase, beta-amylase
7. A patient is found to be deficient in the enzyme galactose 1-phosphate
uridylyltransferase. Specifically due to this deficiency what might a doctor recommend?
A. avoid all strenuous exercise
B. eat a fat-free diet
C. increase intake of vitamin C
D. * avoid ingestion of milk and milk products
E. all of the above.
8. A UDP-galactose is used as a donor of galactose for the reaction of synthesis of:
A. Lactose (in a mammary gland)
B. Glycoproteins
C. Glycolipids
D. Proteoglycans
E. * All answers are correct
9. Aerobic glycolysis is a multi-stage process of transformation of molecule of glucose to
pyruvate. Choose this process:
A. С6Н12О6 С3Н4О3
B. С6Н12О6 С5Н10О4
C. С6Н12О6 6 С2Н5ОН + 2 СО2
D. * С6Н12О6 2С3Н4О3
E. С6Н12О6 2С3Н6О3
10. Aerobic oxidation of pyruvate to CO2 and H2O results in formation of molecules ATP:
A. 38 ATP
B. 12 ATP
C. 3 ATP
D. * 15 ATP
E. 2 ATP
11. An enzyme that catalyzes conversions of L-sugars to D-sugars is called an
A. Lyase
B. Hydrolase
C. Synthetase
D. Synthase
E. * Isomerase
12. An intramolecular phosphoryl-group transfer occurs when:
A. 2-Phosphoglycerate is converted to phosphoenolpyruvate
B. 1,3 bisphosphoglycerate is converted to 3-phosphoglycerate
C. Both a and b
D. * All of the above
E. None of the above
13. Anabolic and catabolic reactions in eukaryotes can occur simultaneously in cells. This is
possible because _______.
A. they all occur in the cytosol
B. the anabolic and catabolic pathways do not share any intermediate metabolites
C. of the compartmentation of metabolites for the reactions of the opposing
pathways
D. * all catabolic reactions are exergonic and all anabolic reactions are endergonic
E. None of the above
14. Anaerobic glycolysis is a complex of reactions as a result of which the molecule of
glucose converts to the lactate or pyruvate. Choose this process:
A. С6Н12О6 С2Н5ОН + 2 СО2
B. С6Н12О6 С5Н10О4
C. С6Н12О6 С7Н14О7
D. * С6Н12О6 2С3Н6О3
E. С6Н12О6 2С3Н4О3
15. Arsenate
A. Competes with NAD for the binding site in 1,3 bisphosphoglycerate
B. Competes with phosphate for its binding site in glyceraldehyde 3-phosphate
dehydrogenase
C. Produces a stable analog of 1,3 bisphosphoglycerate
D. All of the above
E. None of the above
16. Arsenite
A. Poisons by the same mechanism as arsenate
B. Is less toxic than arsenate
C. * Binds tightly to lipoamide sulfur atoms
D. All of the above
E. None of the above
17. ATP is a cosubstrate of the enzyme PFK-1. In most species ATP is also an inhibitor of
PFK-1 at higher concentrations. This seems to violate Le Chatelier's Principle. Which
statement below would provide a suitable explanation?
A. PFK-1 must be phosphorylated by ATP in the active site and the phosphorylated
PFK-1 must be the less active form.
B. There must be another cofactor interacting with ATP at high concentrations to
achieve inhibition of PFK-1.
C. ATP actually activates the reverse of the reaction preceding the PFK-1 step in the
pathway. It likely has no direct effect on PFK-1.
D. * There are two sites on PFK-1 that bind ATP. One is the active site; the other is
the regulatory site where inhibition occurs.
E. None of the above
18. Choose from the below mentioned carbohydrates those, which contains a galactose:
A. Hepatin
B. Maltose
C. Sucrose
D. Starch
E. * Lactose
19. Choose the reaction of glycolysis, in which an inorganic phosphate takes part:
A. Glucose G glucose -6-phosphate
B. Fructose -6-phosphate - fructose -1, 6-biphosphate
C. * Glyceraldehyde 3-Phosphate 1,3- biphosphoglycerate
D. Glucose G glucose -1-phosphate
E. Pyruvate P lactate
20. Choose the reaction of glycolysis, which catalyzed with phosphoglucoisomerase:
A. Glucose -6-phosphate - glucose -1-phosphate
B. Glucose -6-phosphate - glucose
C. Glucose G glucose -6-phosphate
D. * Glucose -6-phosphate - fructose -6-phosphate
E. Glucose -1-phosphate - fructose -6-phosphate
21. Cofactors of dehydrogenases are all, except one:
A. NAD+
B. * c-AMP
C. NADP+
D. FMN
E. FAD
22. Compared to pyruvate, the carbon atoms in lactate ______________.
A. * are more reduced
B. are more oxidized
C. are equally as oxidized
D. carry more charge
E. All of the above
23. Compared to pyruvate, the carbon atoms in lactate ______________.
A. * are more reduced
B. are more oxidized
C. are equally as oxidized
D. carry more charge
E. none of above
24. Converting of glucose into lactic acid consists of eleven reactions. Almost of all reactions
as activators need ions:
A. Zn2+
B. Ca2+
C. ClD. Na+
E. * Mg2+
25. During glycolysis, isomerization occurs during which of the following reactions:
A. Fructose 1,6 bisphosphate -> dihydroxyacetone phosphate and glyceraldehyde 3phosphate
B. Fructose 6 phosphate -> fructose 1,6 bisphosphate
C. Glucose 6- phosphate -> fructose 6- phosphate
D. Glucose -> glucose 6- phosphate
E. All of these
26. During glycolysis, isomerization occurs during which of the following reactions:
A. Fructose 1,6 bisphosphate -> dihydroxyacetone phosphate and glyceraldehyde 3phosphate
B. Fructose 6 phosphate -> fructose 1,6 bisphosphate
C. * Glucose 6- phosphate -> fructose 6- phosphate
D. Glucose -> glucose 6- phosphate
E. None of the above
27. Energetic value of anaerobic glycolysis:
A. * 2 ATP molecules
B. 10 ATP molecules
C. ATP molecules
D. 40 ATP molecules
E. 32 ATP molecules
28. Enzyme that catalyzes the reaction of fructose-1,6-diphosphate disintegration into two
trioses in the process of glycolysis is named:
A. glucokinase
B. glucose-6-phosphatase
C. phosphofructikinase
D. * aldolase
E. enolase
29. Enzyme that catalyzes the reaction of transformation acetaldehyde to ethanol is:
A. pyruvate decarboxylase
B. malate dehydrogenase
C. lactate dehydrogenase
D. succinate dehydrogenase
E. * alcohol dehydrogenase
30. Enzyme which catalyzes the first glycolysis reaction:
A. * hexokinase
B. glucose-6-phosphatase
C. phosphorylase
D. glucose-6-phosphate dehydrogenase
E. amylase
31. Enzymes which catalyze the same reaction are called ___________.
A. * isozymes
B. complementary enzymes
C. cofactors
D. catalytes
E. All of the above
32. For absorption of galactose and fructose into the enterocytes of intestine the presence of
such ions is needed:
A. Mg2+
B. Zn2+
C. * Na+
D. Ca2+
E. H+
33. For transformation of fructose in fructose -1, 6-byphosphate, except for the proper
enzyme, needed:
A. ADP
B. NADP
C. Co A
D. * ATP
E. Fructose-1-phosphate
34. Fragments containing three carbons can be transferred from a ketose phosphate to an
aldose phosphate by the enzyme ________________.
A. pyruvate carboxylase
B. * transaldolase
C. debranching enzyme
D. ribose-5-phosphate isomerase
E. none of the above.
35. Fragments containing three carbons can be transferred from a ketose phosphate to an
aldose phosphate by the enzyme ________________.
A. pyruvate carboxylase
B. * transaldolase
C. debranching enzyme
D. ribose-5-phosphate isomerase
E. aconitase
36. From the diet of a boy with such symptoms of galactosemia as vomiting, diarrhea,
underdevelopment, galactosuria, it is necessary to eliminate:
A. Fats
B. Meat, fish
C. * Milk and milk products
D. Tea, coffees
E. Fruits
37. Fructose -6-phosphate in muscles, kidneys, adipocytes joins glycolysis by its
transformation under the action of phosphofructo kinase into fructose -1, 6-byphosphate.
Which components are needed for this reaction?
A. * ATP and Mg2+
B. AMP and Zn2+
C. GTP and Ca2+
D. UTP and Na+
E. GDP and Mg2+
38. Galactose which an organism gets with food stuffs joins metabolism by transformation of
it into:
A. Ribulose-5- phosphate
B. Fruktozo-1,6-biphosphate
C. Glycerophosphate
D. Fructose-1- phosphate
E. * Glucose-1- phosphate
39. Glucose 6-phosphate allosterically inhibits
A. Hexokinase I
B. Glucokinase
C. Hexokinase II
D. All of the above
E. * a and c only
40. Glucose not part of a diet may be provided by degradation of
A. stored glycogen
B. starch
C. dextrin
D. * a and b
E. a, b and c
41. Glucose-6-phosphate dehydrogenase deficiency is the ______________ which is
characterized by __________________________________________.
A. genetically determined disease; the absence of receptors to LDL on cells and
prominent atherosclerosis;
B. diet-induced disease; the deficit of thiamin in the organism and neurological and
cardiac symptoms;
C. * genetically determined disease; the low level of reduced glutathione in
erythrocytes and increased susceptibility of erythrocytes to hemolysis;
D. disease induced by environmental factors; the high level of oxidized glutathione
in the red blood cell and increased resistance of erythrocytes to hemolysis.
E. none of the above.
42. Glucose, that arrive with blood from an intestine to the liver, is utillized for all functions,
except:
A. Formation of glycogen
B. Oxidation to CO2 and H2O with selection of energy
C. Convert into fats for stocking of energy
D. Carried by blood to the brain as source of energy
E. * Use for detoxification of ammonia
43. Glyceraldehyde 3-phosphate dehydrogenase causes
A. The reduction and phosphorylation of glyceraldehyde 3-phosphate to produce 1,3
bisphosphoglycerate
B. The oxidation of a molecule of NAD+ to NADH
C. Neither a nor b
D. * Both a and b
E. The oxidation of a molecule of FAD
44. Glycogen synthesis takes place in the following oragans:
A. brain and muscles;
B. * liver and muscles;
C. liver and kidney;
D. liver and pancreas.
E. all of the above.
45. Glycolysis has 2 stages. The peculiarities of preparatory stage are:
A. phosphorylation and energy secretion
B. glycolysis metabolites oxidation
C. * energy usage
D. are performed the reactions of substrate phosphorylation
E. formation of pyruvate
46. Glycolysis is defined as:
A. aerobic process in which glucose is transformed to CO2 and H2O;
B. anaerobic process in which glucose is transformed to pyruvate;
C. anaerobic process in which glucose is converted to pentoses and NADPH is
produced;
D. aerobic process in which glucose is converted to glycogen.
E. * anaerobic process in which glucose is transformed to lactate;
47. Glycolysis takes place in:
A. mitochondria;
B. nucleus;
C. lysosomes;
D. * cytoplasm.
E. none of the above.
48. Hamsters love to run on exercise wheels. Prolonged running at a high rate of speed
requires ATP. Could a hamster with a defective gene for the enzyme lactate
dehydrogenase meet the extra ATP demand for prolonged, fast wheel-running by
maintaining a high rate of glycolysis? Why or why not?
A. * No, not enough NAD+ can be regenerated for glycolysis to continue at a high
rate.
B. No, the defective gene will cause a rapid decline in pH in the muscles used for
running.
C. Yes, the defective enzyme has no effect on the glycolytic pathway.
D. Yes, the enzyme alcohol dehydrogenase will supply the needed NAD+ if the
lactose dehydrogenase cannot.
E. None of the above
49. How does the number of molecules of ATP produced compare for conversion of one
molecule of either glucose or fructose to pyruvate?
A. fructose produces one less ATP than glucose
B. * fructose produces the same number of ATP's
C. fructose produces one more ATP than glucose
D. fructose produces twice the number of ATP compared to glucose
E. fructose produces two more ATP than glucose
50. How many ATP molecules are formed with complete oxidation of glucose in aerobic
condition?
A. 2
B. 8
C. * 38
D. 42
E. 30
51. How many ATP molucules are formed as a result of anaerobic oxidation of a single
glucose molecule (net output)?
A. * 2
B. 6
C. 8
D. 4
E. 1
52. How many molecules of ATP have to be spent during glycolysis?
A. 1;
B. * 2;
C. 3;
D. 4.
E. 0
53. How many substrate level phosphorylation reactions are in glycolysis?
A. 1;
B. * 2;
C. 3;
D. 4.
E. 0
54. In anaerobic glycolysis which does not need mitochondrial respiratory chain, ATP forms
with:
A. * Two reactions of the substrate level phosphorylation
B. One reaction of the substrate level phosphorylation and oxidative phosphorylation
C. Three reactions of the substrate level phosphorylation
D. One reaction of the substrate level phosphorylation
E. Two reactions of the oxidative phosphorylation
55. In cells which don’t have mitochondria (mature red cells), cells with low oxidize ability
(retina, malignant cells), the glycolic breaking up of glucose takes place to product:
A. Glycogen
B. Pyruvate
C. CO2 and N2O
D. Acetyl-CoA
E. * Lactate
56. In skeletal muscles and cells of brain the transfer of hydrogen from cytoplasmatic NADH
is carries out:
A. * Glycerophosphate shuttle system
B. Ionophores
C. Permeases
D. Carnytin
E. Malate-aspartate shuttle system
57. In the cells of liver, myocardium and kidneys a malate-aspartate shuttle system on one
molecule of NADH provides formation:
A. 1 ATP
B. * 3 ATP
C. 8 ATP
D. 12 ATP
E. 15 ATP
58. In the muscles of person, which executes a physical work, accumulates:
A. Glycogen
B. Pyruvate
C. * Lactate
D. Glucose
E. CO2 and N2O
59. In the oxygen insufficiency glycolisis is used as the energy source. Select glycolisis
among the below mentioned processes.
A. * glucose to lactate oxidation
B. glucose to CO2 and H2O oxidation
C. pyruvate to acetyl CoA oxidation
D. glucose to glucuronic acid oxidation
E. glycogen to glucose oxidation
60. In the process of alcohol fermentation, which in the human organism takes place in the
cavity of intestine, acetaldehyde is reduces to the ethanol with the:
A. NADPH•H+
B. FAD•H2
C. FMN•H2
D. * NADH•H+
E. CoQ•H2
61. Lactate is formed from pyruvate in an ____________ organism in ____________
conditions.
A. yeast; aerobic;
B. animal; aerobic;
C. fungi; aerobic;
D. * animal; anaerobic.
E. yeast; aerobic.
62. Last reaction of anaerobic glycolysis: Pyruvate P lactate is catalyzed with NADdependent enzyme:
A. Pyruvate kinase
B. Pyruvate dehydrogenase
C. * Lactate dehydrogenase
D. Lactase
E. Enolase
63. Mature red cells, retina, medulla of kidneys, utilize energy mainly glycolytic
decomposition of glucose because in the cells of these organs and tissues:
A. The TCA does not function
B. The respiratory chain is absent
C. There are no any mitochondria
D. High activity of glycolytic enzymes
E. * All answers are correct
64. Metabolite, formed in the process of glucose phosphorylation under influence of
hexokinase is:
A. * glucose-6-phosphate
B. fructose-1-phosphate
C. fructose-6-phosphate
D. glucose-1,6-diphosphate
E. fructose-1,6-diphosphate
65. More than one step in the glycolytic pathway is subject to regulation. It might seem most
efficient to regulate only the first step of a pathway to avoid buildup of intermediates and
to conserve materials and energy. Why is the first step of glycolysis not the only
regulated step?
A. * Some sugars can enter the glycolytic pathway beyond the first step. If steps
other than step one were not regulated, the breakdown of these sugars would be
essentially uncontrolled.
B. Having more than one regulated step in the pathway allows for feedback
inhibition.
C. Control of a single step in a reaction pathway is difficult because the
concentrations of enzymes in cells are very low. It's easier to control more than
one enzyme than just one.
D. All the ATP in a cell would be depleted very quickly if only the first step of
glycolysis were regulated.
E. Both A and B
66. Mutases are described as
A. Polymerases which catalyze phosphoryl group transfers
B. * Isomerases which catalyze the transfer of phosphoryl groups from one part of a
substrate molecule to another
C. Forming intermediate free phosphate (Pi)
D. All of the above
E. None of the above
67. Name 2 products of the first reaction of penthosophosphate cycle which flows in a liver:
A. Glucose and phosphatidic acid
B. NADH and pentose
C. Gluconic acid and ribulose
D. Glucose-1,6-diphosphate and NADPH
E. * 6-phosphogluconolactone and NADPH
68. Name irreversible reactions of glycolysis :
A. * hexokinase, phosphofructikinase, pyruvatekinase
B. aldolase, hexokinase, lactate dehydrogenase
C. enolase, aldolase, pyruvatekinase
D. phosphoglyceratekinase, enolase, lactate dehydrogenase
E. pyruvatekinase, phosphoglyceratekinase, aldolase
69. Name the glycolysis reaction, which requires ATP energy:
A. * hexokinase, phosphofructokinase
B. hexokinase, enolase
C. enolase, aldolase
D. pyruvatekinase
E. phosphoglucomutase, enolase
70. On activating of molecule of glucose and preparation of it to the splitting on two trioses
in the glycolysis used:
A. 1 ATP
B. 5 ATP
C. * 2 ATP
D. 12 ATP
E. 8 ATP
71. Once inside a cell, glucose is rapidly phosphorylated to glucose-6-phosphate. What is the
main purpose of this phosphorylation?
A. to keep glucose inside the cell
B. to form a high-energy compound
C. * to activate PFK-1
D. to prevent mutarotation
E. none of the above.
72. Once inside a cell, glucose is rapidly phosphorylated to glucose-6-phosphate. What is the
main purpose of this phosphorylation?
A. * to keep glucose inside the cell
B. to form a high-energy compound
C. to activate PFK-1
D. to prevent mutarotation
E. All of the above
73. Oxidation of carbohydrates in an organism is carried out by indirect and direct ways. The
followings processes belong to indirect, except:
A. Glycolysis
B. Oxidative decarboxilation of pyruvate
C. Cycle of Krebs
D. * Pentose-phosphate pathway
E. C and A only
74. Pentoses which appear from glucose-6-phosphate in the pentosophosphate cycle can be
utillized in such aims:
A. On the biosynthesis of nucleotides
B. On the construction of nucleic acids
C. On the construction of coenzymes
D. Transformation in to fructose-6-phosphate
E. * All adopted processes are correct
75. Pentosophosphate cycle oxidation of glucose provides reconstraction of NADPH such
processes:
A. Synthesis of fatty acids
B. Synthesis of cholesterol
C. Synthesis of bile acids from cholesterol
D. Inactivation of steroid hormones
E. All adopted processes are correct
76. PFK-2 and fructose 2,6-bisphosphatase are two names for the same enzyme. The name
PFK-2 is used for the enzyme's catalysis of the phosphorylation of fructose 6-phosphate
to fructose 2,6-bisphosphate. The name fructose 2,6-biphosphatase is used for its
catalysis of the reverse reaction. What is unique about this enzyme that makes it logical
to use both names?
A. It is one of very few enzymes that can catalyze both the forward and reverse
reactions.
B. The enzyme is a monomer when catalyzing the phosphorylation reaction and a
dimer when catalyzing the reverse reaction.
C. The forward and reverse reactions occur in different compartments within the
cell, so a different name is used for each activity.
D. * The enzyme is bifunctional. The forward and reverse reactions are catalyzed by
different sites on the same enzyme.
E. all of the above.
77. PFK-2 and fructose 2,6-bisphosphatase are two names for the same enzyme. The name
PFK-2 is used for the enzyme's catalysis of the phosphorylation of fructose 6-phosphate
to fructose 2,6-bisphosphate. The name fructose 2,6-bisphosphatase is used for its
catalysis of the reverse reaction. What is unique about this enzyme that makes it logical
to use both names?
A. It is one of very few enzymes that can catalyze both the forward and reverse
reactions.
B. The enzyme is a monomer when catalyzing the phosphorylation reaction and a
dimer when catalyzing the reverse reaction.
C. The forward and reverse reactions occur in different compartments within the
cell, so a different name is used for each activity.
D. * The enzyme is bifunctional. The forward and reverse reactions are catalyzed by
different sites on the same enzyme.
E. Both A and D
78. Phosphofructokinase I deficiency results in:
A. * An overproduction of Fructose 6-phosphate
B. An overproduction of Fructose 1,6 bisphosphate
C. Fructose 1,6 bisphosphate deficiency
D. a and c only
E. none of the above.
79. Phospholructokinase I deficiency results in:
A. An overproduction of Fructose 6-phosphate
B. An overproduction of Fructose 1,6 bisphosphate
C. Fructose 1,6 bisphosphate deficiency
D. * a and c only
E. All of the above
80. Phosphorylation at the expense of ATP is catalyzed by __________.
A. * protein kinases
B. phosphoryl isomerases
C. phosphatases
D. all of the above
E. None of the above
81. Phosphorylation of fructose catalyzes enzymes:
A. Hexokinase and enolase
B. * Fructokinase and nonespecific hexokinase
C. Phosphofructokinase and enolase
D. Phosphofructokinase and phosphoglucoisomerase
E. Fructose-1-phosphate aldolase and hexokinase
82. Seven of the ten reactions in the glycolytic pathway have free energy values close to
zero. What does this tell us about those reactions?
A. They are near equilibrium reactions.
B. They are not control points for pathway regulation.
C. They are reversible reactions.
D. * All of the above.
E. None of the above.
83. Substrate level phosphorylation
A. Describes the conversion of ADP into ATP with the addition of inorganic
phosphate every place throughout the cell
B. Describes the formation of ADP by phosphoryl group transfer from 1,3
bisphosphoglycerate
C. None of the above
D. * Both a and b
84. Sucrase hydrolyses sucrose into:
A. glucose and galactose;
B. glucose and glucose;
C. * glucose and fructose;
D. glucose and mannose.
E. none of the above.
85. The accumulation of phosphorylated monosaccharides in the hepatocytes does not result
in the osmotic transfer of water and swelling of cells because glucose -6-phosphate
converts into insoluble:
A. Starch
B. Dekstrins
C. * Glycogen
D. Cellulose
E. Triacylglycerol
86. The activity of which glycolytic enzyme shown below is not used to control the rate of
glycolysis?
A. PFK-1
B. pyruvate kinase
C. * triose phosphate isomerase
D. hexokinase
E. glucokinase
87. The activity of which glycolytic enzyme shown below is not used to control the rate of
glycolysis?
A. PFK-1
B. pyruvate kinase
C. * triose phosphate isomerase
D. hexokinase
E. None of the above
88. The biological functions of glycolysis are the following, except:
A. getting energy by short way
B. * ATP formation by oxidative phosphorylation
C. ATP formation by substrate level phosphorylation
D. usage of intermediate metabolites for lipids synthesis
E. usage of metabolites fo glyconeogenesis
89. The concentration of glucose in the blood is normal. Which of the below-mentioned
figures proves this?
A. 3-10 mmol/l
B. * 3,3-5,5 mmol/l
C. 4-8 mmol/l
D. 10-15 g/l
E. 1-2 mmol/l
90. The conversion of pyruvate to ethanol also causes the ______________.
A. * oxidation of NADH
B. production of ADP
C. consumption of O2
D. generation of an ion gradient across mitochondrial membranes
E. oxidation of FADH2
91. The conversion of pyruvate to ethanol also causes the ______________.
A. * oxidation of NADH
B. production of ADP
C. consumption of O2
D. generation of an ion gradient across mitochondrial membranes
E. oxidation of FADH2
92. The conversion of pyruvate to ethanol also causes the ______________.
A. * oxidation of NADH
B. production of ADP
C. consumption of O2
D. generation of an ion gradient across mitochondrial membranes
E. reduction of NAD
93. The conversion of pyruvate to ethanol also causes the ______________.
A. * oxidation of NADH
B. production of ADP
C. consumption of O2
D. generation of an ion gradient across mitochondrial membranes
E. None of the above
94. The end product of anaerobic glycolysis is:
A. pyruvate
B. * lactic acid
C. ethanol
D. acetic acid
E. glycerine aldehyde
95. The end produtcs of aerobic glucose oxidation:
A. lactic acid
B. * Pyruvate
C. CO2 and H2O
D. ethanol
E. acetaldehyde
96. The enzyme that catalyzes the conversion of pyruvate to lactate is ______________.
A. lactate reductase
B. pyruvate kinase
C. lactoenolpyruvate
D. * lactate dehydrogenase
E. lactate oxidase
97. The enzyme that catalyzes the conversion of pyruvate to lactate is ______________.
A. lactate reductase
B. pyruvate kinase
C. lactoenolpyruvate
D. * lactate dehydrogenase
E. lactate oxidase
98. The enzyme that catalyzes the conversion of pyruvate to lactate is ______________.
A. lactate reductase
B. pyruvate kinase
C. lactoenolpyruvate
D. * lactate dehydrogenase
E. None of the above
99. The enzyme which the key regulatory step in glycogen biosynthesis is
A. * Glycogen synthase
B. Glycogenin
C. Branching enzyme
D. Phosphoglucomutase
E. UDP-glucose pyrophosphorylase
100.
The family of GluT (glucose transporters) is:
A. * the proteins embedded into the cell membrane and facilitating the glucose
transport across the membrane;
B. the proteins embedded into the cell membrane and inhibiting the glucose transport
across the membrane;
C. the cytoplasmic proteins transporting glucose in the cytoplasm of cell;
D. the proteins of blood plasma transporting glucose via the blood.
E. none of the above.
101.
The glycerophosphate shuttle system on one molecule of cytoplasmatic NADH
provides formation:
A. 15 ATP
B. 12 ATP
C. 8 ATP
D. * 2 ATP
E. 3 ATP
102.
The glycolytic pathway oxidizes glucose to two molecules of pyruvate and also
produces a net of two molecules of ATP. ATP allosterically inhibits the enzyme, PFK-1,
that catalyzes the third step of glycolysis. This is an example of _____________.
A. feed-forward activation
B. * feedback inhibition
C. negative cooperativity
D. competitive inhibition
E. None of the above
103.
The inhibitor of phosphofructikinase in the glycolysis is:
A. * adenosine triphosphate
B. guanosine monophosphate
C. cytidine monophosphate
D. guanosine triphosphate
E. uridine diphosphate
104.
The inhibitors of phosphofructokinase (which catalyses transformation of fructose
-6-phosphate into fructose -1, 6-biphosphate) are:
A. AMP and ADP
B. AMP and lactate
C. ADP and malonate
D. ADP and ATP
E. * ATP and citrate
105.
The innate undigestion of fructose is related to the genetic defect of enzyme:
A.
B.
C.
D.
E.
* Fructose-1-phosphate aldolase
Hexokinase
Phosphofruktokinase
Phosphoglucoisomerase
Enolase
106.
The key intermediate product of carbohydrate metabolism – glucose-6-phosphate
– can transformed in a liver by such ways, except:
A. From glucose-6-phosphate glycogen is synthesed
B. Glucose-6-phosphate under influence of specific phosphates split to free glucose
C. Spliting by glycolisis to pyruvate
D. Spliting to CO2 and H2O with selection energy of ATP
E. * Transforms in uracyl
107.
The main hormones that regulate the synthesis and decomposition of glycogen
are:
A. insulin, glucocorticoids, thyroxin;
B. glucagons, glucocorticoids, vasopressin;
C. * insulin, glucagons, epinephrine;
D. glucocorticoids, glucagons, epinephrine.
E. all of the above.
108.
The major regulatory enzyme of glycolysis is phosphofructokinase, the activators
of which are:
A. ATP and citrate
B. * AMP and ADP
C. Lactate and AMP
D. Malonate and ATP
E. Ions of Zn2+ and Na+
109.
The major regulatory step of the pentose phosphate pathway is catalyzed by which
enzyme?
A. transaldolase
B. phosphofructokinase-1
C. * glucose 6-phosphate dehydrogenase
D. ribose 5-phosphate isomerase
E. hexokinase
110.
The molecule 1,3-bisphosphoglycerate can be converted to 3phosphoenolpyruvate two ways. One way is catalyzed by the glycolytic pathway enzyme
phosphoglycerate kinase. The other route is a two step reaction sequence that uses the
enzymes bisphosphoglycerate mutase and 2,3-bisphosphoglycerate phosphatase. The
intermediate product of the two-step sequence is 2,3-BPG which is an allosteric inhibitor
of hemoglobin. What disadvantage would there be if the glycolytic pathway only used
the two-step reaction sequence?
A. * The step catalyzed by phosphoglycerate kinase is one of the ATP producing
steps of glycolysis. Using only the two-step reaction sequence would reduce the
number of ATP's produced.
B. There is no disadvantage to the two-step sequence other than having to use more
than one enzyme.
C. The molecule 2,3-BPG is also a potent inhibitor of PFK-1. Even transient
production of 2,3-BPG will significantly slow glycolysis.
D. Too much 2,3-BPG would be produced which would cause clumping of red blood
cells.
E. Both A and D
111.
The non-oxidative stage of the pentose phosphate pathway produces substances
that are intermediates of ___________.
A.
B.
C.
D.
E.
* glycolysis
the citric acid cycle
the Cori cycle
glycogenolysis
none of the above.
112.
The non-oxidative stage of the pentose phosphate pathway produces substances
that are intermediates of ___________.
A. * glycolysis
B. the citric acid cycle
C. the Cori cycle
D. glycogenolysis
E. B and C
113.
The non-oxidative stage of the pentose phosphate pathway __________.
A. produces NADPH and releases CO2
B. * consists entirely of near-equilibrium reactions
C. contains two reactions whose enzymes are allosterically inhibited by NADPH
D. consumes four ATP molecules
E. produces FADH2 and releases CO2
114.
The non-oxidative stage of the pentose phosphate pathway produces substances
that are intermediates of ___________.
A. * glycolysis
B. the citric acid cycle
C. the Cori cycle
D. glycogenolysis
E. a and b
115.
The overall ‘G for glycolysis is -72 kJ/mol in erythrocytes. Which statement
below is true?
A. The value of ‘G0' is also -72 kJ/mol since the cytosol pH is close to 7.
B. The free energy of glycolysis is found as the sum of the standard free energy
changes for the individual pathway reactions.
C. * The negative sign of ‘G shows that this pathway will proceed toward product
(pyruvate) under normal cellular conditions.
D. All of the above.
E. None of the above
116.
The pentose phosphate pathway consists of _____ phases:__________________ .
A. three; oxidative, hydrolytic and reductive;
B. four; oxidative, nonoxidative, hydrolytic and reductive;
C. * two; oxidative and nonoxidative;
D. two; oxidative and reductive.
E. none of the above.
117.
The pentose phosphate pathway has two primary products. They are _________.
A. ATP and NADPH
B. oxaloacetate and acetyl CoA
C. sorbitol and fructose
D. * ribose-5-phosphate and NADPH
E. ribose-5-phosphate and FADH2
118.
The pentose phosphate pathway has two primary products. They are _________.
A. ATP and NADPH
B. oxaloacetate and acetyl CoA
C. sorbitol and fructose
D. * ribose-5-phosphate and NADPH
E. a and d
119.
The possible metabolic pathways for pyruvate in anaerobic conditions are:
A. conversion to lactate or acetyl CoA;
B. conversion to ethanol or acetyl CoA;
C. * conversion to lactate or ethanol;
D. conversion to lactate or ethanol or acetyl CoA.
E. all of the above.
120.
The preparatory stage of glycolysis ends with formation of glyceraldehyde-3phosphate and dioxyacetone phosphate. This reaction catalyzed by an enzyme:
A. Phosphoglyceratekinase
B. Hexokinase
C. Enolase
D. * Aldolase
E. Phosphoglyceratemutase
121.
The process of glucose oxidation in anaerobic condition is:
A. * glycolisis
B. glyconeogenesis
C. glycogenolysis
D. glycogenogenesis
E. lipolysis
122.
The process of polysaccharides digestion in the gastrointestinal tract is catalized
by:
A. sucrase;
B. lactase;
C. maltase;
D. * D-amilase.
E. none of the above
123.
The product of aerobic glycolysis is:
A. 1,3- biphosphoglycerate
B. * Pyruvate
C. Acetyl-CoA
D. Enolphosphopyruvate
E. Lactate
124.
The products of lactose decomposition in the intestine are:
A. * glucose, galactose
B. glucose, fructose
C. fructose, galactose
D. maltose, ribose
E. ribose, glucose
125.
The products of sucrose decomposition in the intestine are:
A. glucose, galactose
B. * glucose, fructose
C. fructose, galactose
D. maltose, ribose
E. ribose, glucose
126.
The products, which are formed by the decomposition of fructose-1,6-diphosphate
in the glycolysis:
A. glycerol, pyruvate
B. * dihydroxyacetone phosphate, 3-phosphoglycerine aldehyde
C. pyruvate, dihydroxyacetone phosphate
D. lactic acid, acetyl CoA
E. acetyl CoA, glycerin
127.
The reaction of ethanol formation at the alcohol fermentation is catalyzed by an
enzyme:
A. Pyruvate dehydrogenase
B. Enolase
C. * Alcohol dehydrogenase
D. Lactate dehydrogenase
E. Pyruvate decarboxylase
128.
The reaction, which determines the glycolysis speed:
A. lactate dehydrogenase
B. * pyruvate kinase
C. glucose-6-phosphatase
D. aldolase
E. glucose-6-phosphate isomerase
129.
The reactions of alcohol fermentation and glycolysis are same to the stage of
pyruvate which farther under action of pyruvatedecarboxylase converts into:
A. Ethanol
B. Lactate
C. Glycerol
D. Glyceraldehyde
E. * Acetaldehyde
130.
The reason of development of cataract of children with galactosemia is a decrease
of activity of such enzyme in the lens as:
A. Glyucokinase
B. * Galactose-1- phosphate uridiltransferase
C. Galactomutase
D. Aldolase
E. UDP-glyucuronil transferase
131.
The sequence of glucose oxidation to lactate in peripheral tissues, delivery of
lactate to the liver, formation of glucose from lactate in the liver, and delivery of glucose
back to peripheral tissues is known as the ______.
A. glyoxylate cycle
B. Kreb's cycle
C. * Cori cycle
D. gluconeogenesis cycle
E. None of the above
132.
The sequence of reactions of alcohol fermentation is the same, as well as sequence
of reactions of glycolysis to the stage of such product formation:
A. * Pyruvate
B. Lactate
C. 3- phosphoglycerate
D. 2-phosphoglycerate
E. Two trioses
133.
The slowing of glycolysis in the presence of oxygen is called the ____________
effect.
A. Bohr
B. Michaelis-Menton
C. * Pastuer
D. Pauling
E. Fisher
134.
The slowing of glycolysis in the presence of oxygen is called the ____________
effect.
A. Bohr
B.
C.
D.
E.
Michaelis-Menton
* Pastuer
Pauling
None of the above
135.
The three control sites in glycolysis are the reactions catalyzed by:
A. hexokinase, phosphofructokinase 1 and aldolase;
B. phosphofructokinase 1, aldolase and pyruvate kinase;
C. hexokinase, aldolase and pyruvate kinase;
D. * hexokinase, phosphofructokinase 1 and pyruvate kinase.
E. all of the above.
136.
The transport of hydrogen from cytoplasm NADH to the respiratory chain of
internal mitochondrial membrane takes place with a help of:
A. Carnytin
B. Simple diffusion
C. * Malate-aspartate shuttle system
D. Permeases
E. Ionophores
137.
There are four enzymes that can catalyze the first step of glycolysis. They are
hexokinases I, II and III and glucokinase (hexokinase IV). Hexokinases I, II and III have
KM values near 0.1 mM. Glucokinase has a KM of 2-5 mM. If after a heavy meal the
blood glucose level rises to 8 mM which statement will be true?
A. At this high blood glucose level all four hexokinases are saturated with substrate.
B. * Hexokinases I, II and III are catalyzing at their maximum rate, but glucokinase
can still respond to increases in blood glucose levels.
C. None of the enzymes is saturated. All of them help to increase the rate of
glycolysis.
D. The flux through glucokinase will be extremely low. The only significant
catalysis is done by hexokinases I, II and III.
E. A and B correct
138.
Transfer of a high-energy phosphoryl group to ADP, resulting in ATP occurs
when:
A. 1,3 Bisphosphoglycerate -> 3-phosphoglycerate
B. * Phosphoenolpyruvate (PEP) -> pyruvate
C. 3-Phosphoglycerate -> 2-phosphoglycerate
D. Both a and b
E. all of the above.
139.
Transfer of a high-energy phosphoryl group to ADP, resulting in ATP occurs
when:
A. 1,3 Bisphosphoglycerate -> 3-phosphoglycerate
B. Phosphoenolpyruvate (PEP) -> pyruvate
C. 3-Phosphoglycerate -> 2-phosphoglycerate
D. * Both a and b
E. None of the above
140.
Transfer of the phosphoryl group from PEP to ADP is an example of
A. A mutase reaction
B. Isomerization
C. A dehydrogenase
D. * None of the above
E. Oxidation
141.
Transfer of the phosphoryl group from PEP to ADP is an example of
A. A mutase reaction
B. Isomerization
C. A dehydrogenase
D. * None of the above
E. Oxidation
142.
Transformation: D- galactose -1-phosphate + UDP-glucose D-glucose-1phosphate + UDP-1- galactose, catalyses an enzyme:
A. Hexokinase
B. Phosphoglucomutase
C. * Galactose-1- phosphate uridiltransferase
D. Glucokinase
E. UDP-galactopyrophosphorilase
143.
Transketolase and transaldolase are the enzymes of _______________ phase of
pentose phosphate pathway and they catalyze the ___________________________.
A. * nonoxidative; the exchange of two- and three-carbon fragments between sugar
phosphates;
B. oxidative; the conversion of glucose to ribulose 5-phosphate;
C. nonoxidative; the conversion of glucose to ribulose 5-phosphate;
D. oxidative; the exchange of two- and three-carbon fragments between sugar
phosphates
E. None of the above.
144.
Under starvation conditions, about how long does it take in humans for the body
to deplete the glycogen store in the liver?
A. 10 minutes
B. * 4 hours
C. 24 hours
D. 1 week
E. 2 weeks
145.
What are the effects of protein kinase A on PFK-2 and pyruvate kinase?
A. phosphorylates PFK-2; dephosphorylated pyruvate kinase; both enzymes are
inhibited
B. * phosphorylates both enzymes; inhibits both enzymes
C. dephosphorylates both enzymes; inhibits both enzymes
D. dephosphorylates PFK-2; phosphorylates pyruvate kinase; activates PFK-2;
inhibits pyruvate kinase
E. Both A and D
146.
What chemical species activates the GLUT4 protein to transport glucose into
cells?
A. adrenaline
B. * insulin
C. protein kinase A
D. PFK-2
E. all of the above.
147.
What chemical species activates the GLUT4 protein to transport glucose into
cells?
A. adrenaline
B. * insulin
C. protein kinase A
D. PFK-2
E. glucagon
148.
What chemical species activates the GLUT4 protein to transport glucose into
cells?
A. adrenaline
B. * insulin
C. protein kinase A
D. PFK-2
E. None of the above
149.
What enzyme catalyzes transformation of fructose -6-phosphate into glucose -6phosphate?
A. Phosphofructokinase
B. Phosphoglucomutase
C. * Phosphoglucoisomerase
D. Triosophosphateisomerase
E. Enolase
150.
What enzyme transform glucose of meal in a liver into glucose-6-phosphate?
A. Phosphoglucomutase
B. Phosphatase
C. * Hexokinase
D. Fructokinase
E. Glucose isomerase
151.
What from the below mentioned functions is not proper to glycolysis?
A. Providing of organism energy at the hypoxia
B. Formation of dihydroxyacetone phosphate that is reduced to the glycerophosphate
C. Formation of pyruvate and lactate for their transformations in aerobic conditions
D. Energy source for malignant cells
E. * Synthesis ATP by the oxidative phosphorylation
152.
What is the function of pyruvate decarboxylase?
A. * The conversion of pyruvate to carbon dioxide and acetaldehyde.
B. To completely oxidize pyruvate to three molecules of carbon dioxide.
C. To convert phosphoenolpyruvate to pyruvate.
D. To regulate PFK-1.
E. The conversion of pyruvate to acetyl CoA
153.
What is the function of pyruvate decarboxylase?
A. * The conversion of pyruvate to carbon dioxide and acetaldehyde.
B. To completely oxidize pyruvate to three molecules of carbon dioxide.
C. To e. None of the above convert phosphoenolpyruvate to pyruvate.
D. To regulate PFK-1.
E. None of the above
154.
What is the prosthetic group of transketolase?
A. * thiamine pyrophosphate
B. biotin
C. pyridoxal phosphate
D. NAD+
E. FAD
155.
What types of reactions are involved in the two-step conversion of glucose to
fructose?
A. * reduction followed by oxidation
B. two sequential hydrolysis reactions
C. hydrolysis followed by isomerization (rearrangement)
D. phosphorylation followed by dephosphorylation
E. phosphorylation followed by reduction
156.
Where in the cell the glycolysis reactions are localized?
A. mitochondria
B. * cytoplasm
C. lysosomes
D. microsomes
E. nucleus
157.
Which amino acid's residue plays a role in the phosphoglycerate mutase reaction
in glycolysis for muscle and yeast?
A. Leucine
B. Lysine
C. Alanine
D. Histidine
E. All of the above
158.
Which amino acid's residue plays a role in the phosphoglycerate mutase reaction
in glycolysis for muscle and yeast?
A. Leucine
B. Lysine
C. Alanine
D. Histidine
E. All of above
159.
Which applies to fructose 1,6-bisphosphate?
A. inhibitor of pyruvate kinase
B. product of PFK-1 catalyzed step in glycolysis
C. isomer of glucose 1,6-bisphosphate
D. * all of the above
E. None of the above
160.
Which is a reasonable concentration for glucose in the blood?
A. 0.5 mM
B. * 5 mM
C. 50 mM
D. 5 M
E. 3.5 M
161.
Which is an intermediate formed in the conversion of glucose to fructose?
A. glucose-1-phosphate
B. * sorbitol
C. ribose
D. aldose reductase
E. pyruvate
162.
Which is not a function of the main products of the pentose phosphate pathway?
A. To maintain the reduced form of iron in hemoglobin.
B. To provide reducing power for the synthesis of fatty acids.
C. To serve as precursors in the biosynthesis of RNA and DNA.
D. * To raise the concentration of cAMP.
E. none of the above.
163.
Which is not a function of the main products of the pentose phosphate pathway?
A. To maintain the reduced form of iron in hemoglobin.
B. To provide reducing power for the synthesis of fatty acids.
C. To serve as precursors in the biosynthesis of RNA and DNA.
D. * To raise the concentration of cAMP.
E. D and C
164.
Which is not a possible fate of mitochondrial pyruvate?
A. conversion to oxaloacetate by pyruvate carboxylase
B. * conversion to phosphoenolpyruvate by pyruvate kinase
C. conversion to acetyl CoA
D. conversion to citrate for fatty acid synthesis
E. a, b and c
165.
Which is not among the possible fates of pyruvate after glycolysis?
A.
B.
C.
D.
E.
conversion to lactate
further reduction by the citric acid cycle
conversion to ethanol
* used in the biosynthesis of alanine
all of the above.
166.
Which is not among the possible fates of pyruvate after glycolysis?
A. conversion to lactate
B. * further reduction by the citric acid cycle
C. conversion to ethanol
D. used in the biosynthesis of Alanine
E. None of the above
167.
Which is not among the possible fates of pyruvate after glycolysis?
A. conversion to lactate
B. further reduction by the citric acid cycle
C. conversion to ethanol
D. used in the biosynthesis of alanine
E. * all of the above
168.
Which metabolites link pentose phosphate pathway with glycolysis?
A. 3-phosphoglyserate, 2-phosphoglycerate;
B. phosphoenolpyruvate, pyruvate
C. fructose 1,6-biphosphate, phosphoenolpyruvate;
D. * fructose 6-phosphate, glucose 6-phosphate.
E. none of the above.
169.
Which molecule cannot be directly transported from the mitochondria to the
cytosol?
A. * oxaloacetate
B. acetate
C. malate
D. phosphoenolpyruvate
E. a and c
170.
Which of the following elements is required for the enzymatic reaction which
produces phosphoenolpyruvate?
A. Potassium
B. Calcium
C. * Magnesium
D. Manganese
E. Sodium
171.
Which of the following elements is required for the enzymatic reaction which
produces phosphoenolpyruvate?
A. Potassium
B. Calcium
C. * Magnesium
D. Manganese
E. Sodium
172.
Which of the following enzymatic reactions is/are control points for glycolysis:
A. Glucose 6-phosphate isomerase
B. Aldolase
C. Both a and b
D. * Neither a nor b
E. Mutase
173.
Which of the following is not a metabolically irreversible enzymatic reaction of
glycolysis?
A.
B.
C.
D.
E.
Pyruvate kinase reaction
PFK-1 reaction
Hexokinase/Glucokinase reaction
* None of the above
All of the above
174.
Which of the following is not a metabolically irreversible enzymatic reaction of
glycolysis?
A. Pyruvate kinase reaction
B. PFK-1 reaction
C. Hexokinase/Glucokinase reaction
D. * None of the above
E. All of these
175.
Which of the following is not a metabolically irreversible enzymatic reaction of
glycolysis?
A. Pyruvate kinase reaction
B. PFK-1 reaction
C. Hexokinase/Glucokinase reaction
D. None of the above
E. * Aldolase reaction
176.
Which of the following is not regulated in glycolysis?
A. Pyruvate kinase
B. * Phosphoglycerate kinase
C. Hexokinase
D. PFK-1
E. all of the above.
177.
Which of the following is not regulated in glycolysis?
A. Pyruvate kinase
B. * Phosphoglycerate kinase
C. Hexokinase
D. PFK-1
E. None of the above
178.
Which of the following is not regulated in glycolysis?
A. Pyruvate kinase
B. * Phosphoglycerate kinase
C. Hexokinase
D. PFK-1
E. None of above
179.
Which of the following is not regulated in glycolysis?
A. Pyruvate kinase
B. * Phosphoglycerate kinase
C. Hexokinase
D. PFK-1
E. Glucokinase
180.
Which of the following mutases catalyze the formation of a 2,3 BPG
intermediate?
A. Muscle phosphoglycerate mutases
B. Plant phosphoglycerate mutases
C. Yeast phosphoglycerate mutases
D. * a and c only
E. All of the above
181.
Which of the following mutases catalyze the formation of a 2,3 BPG
intermediate?
A.
B.
C.
D.
E.
Muscle phosphoglycerate mutases
Plant phosphoglycerate mutases
Yeast phosphoglycerate mutases
* a and c only
All of the above
182.
Which substance causes muscles to ache during strenuous exercise?
A. pyruvic acid
B. lactose dehydrogenase
C. lactate ion
D. * lactic acid
E. acetyl CoA
183.
Which substance causes muscles to ache during strenuous exercise?
A. pyruvic acid
B. lactose dehydrogenase
C. lactate ion
D. * lactic acid
E. None of the above
184.
Which substance is not needed for the conversion of glucose to fructose?
A. NADPH
B. Aldose reductase
C. * QH2
D. NAD+
E. b and c
185.
Which way transformation galactose in a liver on glucose-6-phosphate?
A. By specific hecsoisomerase
B. Glicolitic to pyruvat, and farther – by gluconeogenesis
C. By glucomutase
D. By cytidiltransferase
E. * Galactose galactose-1-phosphatepglucose-1-phosphatepglucose-6-phosphate
186.
Why would it be desirable to slow glycolysis when the demand for ATP is high?
A. Inhibition of PFK-1 allows for the complete oxidation of pyruvate via the citric
acid cycle.
B. * Slowing glycolysis slows the rate of decrease in pH. A low pH can be harmful
and potentially fatal.
C. The less active form of PFK-1 is a potent allosteric activator of creatine, so even
though glycolysis is slowed, ATP production is actually increased by the
activation of creatine.
D. As PFK-1 is inhibited, its isozyme, PFK-2 is activated. PFK-2 is functional at a
much lower pH than PFK-1.
E. Slowing glycolysis slows the rate of increase in pH. A high pH can be harmful
and potentially fatal.
187.
Yeast will normally convert pyruvate to ethanol. Why is this better for the yeast
than a conversion to lactate?
A. Conversion to ethanol releases more NAD+ per mole than the conversion to
lactate.
B. The carbon atoms are more oxidized in ethanol than in lactate.
C. * Ethanol is neutral, but lactate production is accompanied by a sharp decrease in
pH.
D. Ethanol production is not better. Yeast normally produces ethanol and lactate in
equilmolar amounts.
E. A and B only
188.
Gluconeogenesis
A. Is concerned with synthesis of glycogen
B. Refers to the conversion of glycogen into glucose
C. * Refers to the ability of the liver to form precursors for glucose using fats and
proteins
D. Is a process resulting from deamination of amino acids
E. Is an essential part of the emulsification of fats
189.
A lack of blood glucose would have the greatest effects on
A. * The brain.
B. The heart.
C. The liver.
D. Skeletal muscles
E. Cardiac muscle
190.
A regulator of the enzyme Glycogen synthase is
A. Citric acid
B. 2, 3 bisphosphoglycerate
C. * Pyruvate
D. GTP
E. GDP
191.
ADP and AMP ___________________ gluconeogenesis.
A. Stimulate
B. * Inhibit
C. Do not effect
D. ADP stimulates and AMP inhibits
E. ADP inhibits and AMP stimulates
192.
Aerobic resynthesis of ATP occurs:
A. In the mitochondria in a process called glycogenolysis
B. * In the mitochondria in a process called oxidative phosphorylation
C. In the cytosol
D. In the sarcoplasmic reticulum
E. Only in Type I muscle fibres
193.
After glycogen has been used up in the postabsorptive state, which of the
following is usually used as the next source of energy?
A. * Glucose
B. Lipids
C. Proteins
D. Amino acids
E. Only B
194.
After the hepatic threshold for glycogen formation has occurred any excess
glucose in the blood is:
A. * Converted to lipid and stored as fat
B. Excreted by the kidney
C. Converted to urea
D. Is used in protein synthesis
E. B and c
195.
After the hepatic treshhold for glycogen formation has occurred, any excess
glucose in the blood is
A. * Converted to lipid and stored as fat
B. Excreted by the kidneys
C. Converted to urea
D. Is used in protein synthesis
E. B and C are correct
196.
All are true for ?-amylase EXCEPT:
A.
B.
C.
D.
E.
Catalyzes hydrolysis of ? (1 4) linkages in starch.
Is an important component of saliva and pancreatic juice.
* Catalyzes hydrolysis of ? (1 6) linkages in amylopectin.
Catalyzes hydrolysis of ? (1 4) linkages in glycogen.
Activity is reduced in highly branched regions of polysaccharides and stops four
residues from any branch point.
197.
All of the following enzymes involved in the flow of carbon from glucose to
lactate (glycolysis) are also involved in the reversal of this flow (gluconeogenesis)
except:
A. 3-phosphoglycerate kinase.
B. Aldolase.
C. Enolase.
D. * Phosphofructokinase-1.
E. Phosphoglucoisomerase.
198.
All of the molecules below are examples of polymers of glucose subunits except:
A. * Chitin
B. Glycogen
C. Cellulose
D. Starch
E. Sucrose
199.
All of the molecules below are examples of polysaccharides except:
A. Glycogen
B. Cellulose
C. Starch
D. * Maltose
E. All of the above
200.
Amylose (a form of starch) and cellulose are both plant carbohydrate polymers.
What is the main structural difference between them?
A. Cellulose has many more branches than amylose
B. * Cellulose has ?-(1,4) glycosidic links and amylose has ?-(1,4) glycosidic links
C. Cellulose has ?-(1,4) glycosidic links and amylose has ?-(1,4) glycosidic links
D. Cellulose is a polymer of galactose, and amylose is a polymer of glucose
E. Cellulose is a polymer of glucose, and amylose is a polymer of galactose
201.
An enzyme involved in both glycolysis and gluconeogenesis is:
A. Hexokinase
B. Glucose-6-phosphatase
C. * 3-phosphoglycerate kinase
D. Phosphofructokinase-1
E. Pyruvate kinase
202.
An essential for converting Glucose to Glycogen in Liver is
A. Lactic acid
B. GTP
C. CTP
D. * UTP
E. FMN
203.
Anaerobic metabolism of glucose results in an oxygen debt that is the amount of
oxygen needed to metabolize the ______ that is produced.
A. Carbon dioxide
B. * Lactic acid
C. Glycogen
D. Fatty acid
E. Cholesterol
204.
Anaerobic metabolism refers to the generation of ATP:
A. Without the involvement of ADP
B. Without the use of glycogen
C. * Without the use of oxygen
D. In the absence of available oxygen
E. By the conversion of pyruvate to lactate
205.
Anaerobic respiration
A. * Is the primary metabolic pathway of cardiac muscle.
B. Is induced by ischemia.
C. Produces water and carbon dioxide as products.
D. Produces a maximum of 30 ATP
E. Produces oxygen as products
206.
Any compound that can be converted to __ can be a precursor for
gluconeogenesis.
A. Citrate
B. Pyruvate
C. Oxaloacetate
D. All of the above
E. * b and c
207.
Approximately how much energy is liberated when one gram of carbohydrate is
completely oxidised?
A. 4 kJ
B. 8 kJ
C. * 16 kJ
D. 24 kJ
E. 38 kJ
208.
As a result of anaerobic respiration in humans, glucose is converted to
A. * Pyruvic acid.
B. Lactic acid.
C. Citric acid.
D. Acetyl CoA.
E. Vitamin C
209.
As a result of glycolysis, linking step and the Krebs cycle each molecule of
glucose produces:
A. * 2 molecules of lactic acid
B. 6 molecules of carbon dioxide
C. 3 molecules of water
D. 3 molecules of carbon dioxide
E. 2 molecules of O2
210.
Blood glucose is actually:
A. * Absorbed from GI tract
B. Released by the muscles into the blood when it isn’t needed as energy source for
contraction
C. Glycogen from the liver
D. Glucagon which can be converted to sugar when glucose is needed for energy
source
E. The kinetic energy of the blood
211.
Cori cycle is
A. Synthesis of glucose
B. Reuse of glucose
C. Uptake of glycose
D. * Both (A) & (B)
E. Only C
212.
Cori’s cycle transfers
A. Glucose from muscles to liver
B. * Lactate from muscles to liver
C. Lactate from liver to muscles
D. Pyruvate from liver to muscles
E. Nonbe of the above
213.
Deficit of which enzyme is the base for the I type glycogenosis?
A. Glucose-6-phosphatase
B. * Hexokinase
C. Aldolase
D. Glycogen synthetase
E. Glucose-6-phosphate dehydrogenase
214.
Dietary fructose normally enters the glycolytic pathway by which of the following
mechanisms?
A. Phosphorylation to F-6-P by hexokinase.
B. Phosphorylation to F-1-P by fructokinase, cleavage by F-1-P aldolase.
C. Phosphorylation to F-1-P, isomerization to F-6-P by phosphofructomutase.
D. * A or B
E. A, B, or C
215.
Differentiate between the carbohydrate metabolic processes glycolysis and
gluconeogenesis. Name the following pathways: Releases glucose from glycogen
A. Glycolysis
B. Gluconeogenesis
C. Glycogenesis
D. * Glycogenolysis
E. Pentose phosphate pathway
216.
E 3,3 mmol/l
A. * 8,5 mmol/l
B. 2 mmol/l
C. 4,5 mmol/l
D. 5 mmol/l
217.
E The normal degradation of triglycerides into FFAs by adipose tissue is
impaired.
A. * The normal suppression of gluconeogenesis by insulin in the liver is impaired.
B. The normal uptake of glucose by skeletal muscle is impaired.
C. The normal glucose disposal, which includes glucose oxidation and glycogen
synthesis, by skeletal muscle, is impaired.
D. Excess free fatty acids (FFAs) impair c-cell function.
218.
Each of the following lines is the name of a disaccharide.
A. Galactose
B. Mannose
C. Glucose
D. * Maltose
E. Fructose
219.
Energy released by oxidation of glucose is stored as
A. concentration gradient across a membrane.
B. * TP
C. ADH.
D. DP.
E. oth b and c are correct.
220.
Ethanol decreases gluconeogenesis by
A.
B.
C.
D.
E.
Inhibiting glucose-6-phosphatase
Inhibiting PEP carboxykinase
* Converting NAD+ into NADH and decreasing the availability of pyruvate
Converting NAD+ into NADH and decreasing the availability of lactate
Only B
221.
Excess glucose after a meal will first form glycogen in a process called
A. Glycolysis.
B. * Glycogenesis.
C. Lipogenesis.
D. Lipolyosis.
E. Gluconeogenesis
222.
Excessive intake of ethanol increases the ratio:
A. * NADH : NAD+
B. NAD+ : NADH
C. FADH2 : FAD
D. FAD : FADH2
E. Cyt. B; cyt. C
223.
For glycogenesis, Glucose should be converted to
A. Glucuronic acid
B. Pyruvic acid
C. * UDP glucose
D. Sorbitol
E. None of the above
224.
Fructose metabolism is not well regulated in the liver because:
A. * Fructose-1-phosphate aldolase allows fructose to bypass phosphofructokinase
B. More ATP is derived from glucose than fructose.
C. More ATP is derived from fructose than glucose.
D. lucose is the preferred substrate for hexokinase.
E. GLUT4 transports glucose, but not fructose into the liver.
225.
Glucocorticoids increase the level of glucose in the blood activating:
A. lycogenogenesis
B. Glycogenolysis
C. Gluconeogenesis
D. Ketogenesis
E. * Glycolysis
226.
Glucokinase and hexokinase are different from one another in all of the following
ways except:
A. Their location in the tissues
B. Their Km
C. Their Vmax
D. * The reaction they catalyze
E. Their inhibition by product
227.
Gluconeogenesis is increased in the following condition:
A. Diabetes insipidus
B. * Diabetes Mellitus
C. Hypothyroidism
D. Liver diseases
E. All of the above
228.
Glucose breakdown in certain mammalian and bacterial cells can occur by
mechanisms other than classic glycolysis. In most of these, glucose 6-phosphate is
oxidized to 6-phosphogluconate, which is then further metabolized by:
A. An aldolase-type split to form glyceric acid and glyceraldehyde 3-phosphate.
B.
C.
D.
E.
229.
A.
B.
C.
D.
E.
230.
A.
B.
C.
D.
E.
231.
to
A.
B.
C.
D.
E.
An aldolase-type split to form glycolic acid and erythrose 4-phosphate.
Conversion to 1,6-bisphosphogluconate.
* Decarboxylation to produce keto- and aldopentoses.
Oxidation to a six-carbon dicarboxylic acid.
Glucose enters muscle cells mostly by:
Simple diffusion
* Facilitated diffusion using a specific glucose transporter
Co-transport with sodium
Co-transport with amino acids
Active transport
Glucose enters muscle cells mostly by:
Simple diffusion
* Facilitated diffusion using a specific glucose transporter
Co-transport with sodium
Co-transport with amino acids
Active transport
Glucose molecules entering skeletal muscle fibers are "trapped'' when converted
Pyruvic acid
Glucose 1-phosphate
* Glucose 6-phosphate
Glycogen
Lactic acid
232.
Glucose not part of a diet may be provided by degradation of
A. * Stored glycogen
B. Starch
C. Dextrin
D. A and b
E. A, b and c
233.
Glucose tolerance is decreased in
A. * Diabetes mellitus
B. Hypopituitarisme
C. Addison’s disease
D. Hypothyroidism
E. Anemia
234.
Glucose tolerance is increased in
A. Diabetes mellitus
B. * Adrenalectomy
C. Acromegaly
D. Thyrotoxicosis
E. Riscets
235.
Glucose-6-phosphate can serve as:
A. An intermedicate in glycolysis.
B. A precursor for the synthesis of glycogen.
C. An intermediate in gluconeogenesis.
D. A product of collagen degredation.
E. * A, B, C
236.
Glucose, labeled with 14C in different carbon atoms, is added to a crude extract of
a tissue rich in the enzymes of the pentose phosphate pathway. The most rapid production
of 14CO2 will occur when the glucose is labeled in:
A. * C-1.
B. C-3.
C. C-4.
D. C-5.
E. C-6.
237.
Glycerol is converted to ___________ when it is used for gluconeogenesis.
A. Dihydroxyacetone phosphate
B. Phosphoenolpyruvate
C. Oxaloacetate
D. * 3-phosphoglycerate
E. None of the above.
238.
Glycogen breakdown in exercising muscle is activated by:
A. Insulin
B. Cortisol
C. Increased pH
D. Amylase
E. * None of the above
239.
Glycogen breakdown in exercising muscle is activated by:
A. Insulin
B. Cortisol
C. Increased pH
D. Amylase
E. * None of the above
240.
glycogen in skeletal muscle?
A. Phosphorylase
B. Debranching enzyme
C. * ?-1-6-amyloglucosidase
D. Amylase
E. Gucose-6-phosphatase
241.
Glycogen is converted to glucose-1-phosphate by
A. UDPG transferase
B. Branching enzyme
C. * Phosphorylase
D. Phosphatase
E. None of the above
242.
Glycogen is converted to monosaccharide units by:
A. Glucokinase.
B. Glucose-6-phosphatase
C. * Glycogen phosphorylase.
D. Glycogen synthase.
E. Glycogenase.
243.
Glycogen is present in all body tissues except
A. Liver
B. * Brain
C. Kidney
D. Stomach
E. Pancreas
244.
Glycogen synthesis in vertebrates requires _____________ to activate glucose 1phosphate.
A. ATP
B. ADP
C. UTP
D. UDP
E. * All of the above
245.
Glycogen synthetase activity is depressed by
A. Glucose
B. Insulin
C. * Cyclic AMP
D. Fructokinase
E. Pyruvatdehydrogenase
246.
Glycogen while being acted upon by active phosphorylase is converted first to
A. Glucose
B. Glucose 1-phosphate and Glycogen with 1 carbon less
C. * Glucose-6-phosphate and Glycogen with 1 carbon less
D. 6-Phosphogluconic acid
E. None of the above
247.
Glycogenolysis is promoted by
A. * Glucagon
B. Growth hormone
C. Insulin
D. Cortisol (corticosteroids)
E. Both A and D are correct
248.
Glycogenolysis refers to
A. * The formation of glycogen
B. The formation of urea
C. Conversion of fat and/ or protein to glucose
D. The anaerobic metabolism of glucose
E. Conversion of glycogen to glucose
249.
Glycolysis
A. Requires the presence of oxygen
B. * Is the first step of cellular respiration
C. Produces carbon dioxide and water.
D. Pilizes FAD as an electron acceptor
E. Requires the presence of CO2
250.
Glycolysis in the erythrocyte produces pyruvate that is further metabolized to:
A. O2.
B. Ethanol.
C. Glucose.
D. Hemoglobin.
E. * Lactate.
251.
Glycolysis is inhibited by high concentrations of
A. Glucose.
B. Oxygen.
C. ADP.
D. * ATP.
E. All of above
252.
Glycolysis is the name given to the pathway involving the conversion of:
A. Glycogen to glucose-6-phosphate
B. Glycogen or glucose to fructose
C. * Glycogen or glucose to pyruvate or lactate
D. Glycogen or glucose to pyruvate or acetyl CoA
E. Glucose or fatty acids to pyruvate or acetyl CoA
253.
How certain carbohydrate pathways are reciprocally affected by insulin and
glucagon. The areas of focus will be:
A. Regulation of glycolysis by phosphorylation/dephosphorylation of key enzymes;
B. Regulation of glycogen formation vs. breakdown, again by phosphorylation/
dephosphorylation;
C. * How this regulation is coordinated so as to avoid futile cycling and trapping of
intermediates
D. How this regulation fits with the overall function of insulin vs. glucagon
E. None of the above
254.
How many grams of glucose is it necessary to eat for a patient to perform a
method of sugary loading
A. * 1 g per 1 kg of weight
B. 100 g
C. 20 g
D. 50 g
E. 10 g
255.
How many substrate level phosphorylation reactions are in glycolysis?
A. 1
B. * 2
C. 3
D. 4.
E. 0
256.
Hydrogen ions are formed when:
A. Glycogen becomes depleted
B. Phosphocreatine breakdown occurs
C. Pyruvate is converted to lactate
D. Pyruvate is converted to acetyl CoA
E. * lycolysis is being used as a major means of resynthesising ATP
257.
If excess glucose is present in the body, the glucose first will be stored as
__________ in muscle and the liver.
A. Starch
B. * Glycogen
C. Cellulose
D. Fat
E. Vitamin C
258.
If glucose labeled with 14C in C-3 is metabolized to lactate via fermentation, the
lactate will contain 14C in:
A. All three carbon atoms.
B. Only the carbon atom carrying the OH.
C. * Only the carboxyl carbon atom.
D. Only the methyl carbon atom.
E. The methyl and carboxyl carbon atoms.
259.
If oxygen is unavailable, the pyruvate produced by glycolysis is converted into:
A. Alcohol
B. * Lactic acid
C. Carbon dioxide
D. Acetyl CoA
E. Water
260.
Immediately after exhaustive exercise, carbohydrate should be consumed to:
A. Restore liver glycogen
B. * Restore muscle glycogen
C. Provide the muscle with an energy source during recovery
D. CHO should not be consumed post exercise
E. None of the above
261.
In a eukaryotic cell, the enzymes of glycolysis are located in the:
A.
B.
C.
D.
E.
Plasma membrane.
Inner mitochondrial membrane.
* Cytosol.
Mitochondrial matrix.
Intermembrane space.
262.
In a tissue that metabolizes glucose via the pentose phosphate pathway, C-1 of
glucose would be expected to end up principally in:
A. * Carbon dioxide.
B. Glycogen.
C. Phosphoglycerate.
D. Pyruvate.
E. Ribulose 5-phosphate.
263.
In addition to control of activity by phosphorylation, phosphorylase kinase is
activated by
A. Calmodulin
B. * Ca++
C. cAMP
D. Inhibitor-1
E. Na+
264.
In addition to energy (ATP), what is (are) the final product(s) of aerobic
respiration?
A. O2 and CO2
B. * CO2 and H2O
C. O2 and H2O
D. CO2 only
E. O2 only
265.
In an anaerobic muscle preparation, lactate formed from glucose labeled in C-2
would be labeled in:
A. All three carbon atoms.
B. * Only the carbon atom carrying the OH
C. Only the carboxyl carbon atom.
D. Only the methyl carbon atom.
E. The methyl and carboxyl carbon atoms.
266.
In anaerobic respiration, lactic acid is released into the blood from skeletal
muscle. When oxygen is available, most of the lactic acid is converted back to pyruvic
acid and glucose in the
A. Liver.
B. * Skeletal muscle
C. Heart.
D. Lung.
E. Kidney
267.
In comparison with the resting state, actively contracting human muscle tissue has
a:
A. Higher concentration of ATP.
B. * Higher rate of lactate formation.
C. Lower consumption of glucose.
D. Lower rate of consumption of oxygen
E. Lower ratio of NADH to NAD+.
268.
In general, the higher the intensity of exercise, the greater the proportional
contribution of:
A. Aerobic energy production
B. * Anaerobic energy production
C. The TCA cycle (Krebs’ cycle) to the production of ATP
D. The electron transfer chain to the production of ATP
E. Fat oxidation
269.
In glycolysis, glucose must be activated with the use of how many ATP
molecules?
A. 2
B. 3
C. 10
D. * 1
E. 5
270.
In glycolysis, how is ATP formed?
A. Through deamination
B. Through oxidative phosphorylation
C. * Through substrate level events
D. Through transfer of electrons
E. Through chemiosmosis
271.
In humans, gluconeogenesis:
A. * Helps to increase blood glucose during starvation.
B. Is used to convert fatty acids into glucose.
C. Is stimulated by the hormone insulin.
D. Requires the enzyme hexokinase.
E. Produces more ATP than it consumes.
272.
In the human skeletal muscle and brain cells, the energy yield per molecule of
glucose in aerobic respiration is ________ times higher than the energy yield in
anaerobic respiration.
A. * 2
B. 16
C. 18
D. 19
E. 20
273.
In the liver, kidney, and heart cells, the total number of ATP molecules generated
per glucose in aerobic respiration is
A. 2
B. 30
C. 36
D. * 38
E. 10
274.
Insulin decreases the level of blood glucose by:
A. * Gluconeogenesis activation
B. Gluconeogenesis oppression
C. Glucose phosphorylation oppression
D. Increase of glucose excretion with urine
E. Adrenalin excretion decrease
275.
Insulin facilitates energy storage in liver. Which enzymes of carbohydrate
metabolism are coordinately regulated in liver in response to insulin signaling?
A. Glycogen synthase
B. * Glycogen phosphorylase
C. Phosphofructokinase-1 (PFK-1)
D. Phosphofructokinase-2 (PFK-2)
E. Pyruvate kinase
276.
Lactic acid
A. Is produced as a result of aerobic metabolism of glucose.
B.
C.
D.
E.
* Is one of the normal end products of glycolysis
Is a common end product of red blood cells.
None of the above
All of the above
277.
Liver glycogen breakdown is stimulated by:
A. Insulin
B. Glucagon
C. Adrenaline
D. Both (a) and (b)
E. * Both (b) and (c)
278.
Liver glycogen breakdown is stimulated by:
A. Insulin
B. Glucagon
C. Adrenaline
D. Both (a) and (b)
E. * Both (b) and (c)
279.
Many people who are lactose intolerant can eat yogurt, which is prepared from
milk curdled by bacteria, with no problems. Why is this possible?
A. It is not possible (the statement in the question is not true); if one is lactose
intolerant, it is a genetic defect and cannot be altered
B. The bacteria convert all the lactose (milk sugar) to glucose
C. The bacteria convert all the lactose (milk sugar) to fructose
D. The bacteria convert all the lactose (milk sugar) to galactose
E. * The bacteria convert the disaccharide lactose into its component
monosaccharides, glucose and galactose; these resulting monosaccharides can be
tolerated
280.
Muscle and liver glycogen stores in a well nourished athlete would be sufficient to
sustain approximately how many minutes of submaximal exercise (if this were the only
energy source used)? The exercise is club level marathon pace.
A. 30 minutes
B. * 90 minutes
C. 180 minutes
D. 210 minutes
E. 300 minutes
281.
Muscle lactate production increases when:
A. Oxygen is readily available
B. Pyruvate cannot be formed from glucose breakdown
C. The pH of the muscle falls
D. * Glycolysis is activated at the onset of exercise
E. Muscle glycogen becomes depleted
282.
Name the substance which is the main energy source for brain:
A. Glycogen
B. Fructose
C. Lactic acid
D. Fatty acids
E. * Glucose
283.
Of the enzymes of carbohydrate metabolism listed below, which are
(de)phosphorylated in liver in response to insulin signaling?
A. Glycogen synthase
B. * Glycogen phosphorylase
C. Phosphofructokinase-1 (PFK-1)
D. Phosphofructokinase-2 (PFK-2)
E. Pyruvate kinase
284.
Once glycogen stores are filled, glucose and amino acids are used to synthesize
A. * Glycoproteins.
B. Proteins.
C. Lipids.
D. Lactic acid.
E. None of the above
285.
One of the following enzymes does not change glycogen synthase a to b.
A. Glycogen synthase kinases 3, 4, 5
B. Ca2+ calmodulin phosphorylase kinase
C. Ca2+ calmodulin dependent protein kinase
D. * Glycogen phosphorylase a
E. None of the above
286.
One of the net results of glycolysis is that:
A. Two FAD's are reduced
B. Two FAD's are oxidized
C. Two NAD to power of (exponent)'s are reduced
D. * D. Two NAD to power of (exponent)'s are oxidized
E. None of the above
287.
Pathway for synthesis of glycogen (from glucose)
A. Glycolysis
B. Gluconeogenesis
C. * Glycogenesis
D. Glycogenolysis
E. Pentose phosphate pathway
288.
Physiological glycosuria is met with in
A. Renal glycosuria
B. * Alimentary glycosuria
C. Diabetes Mellitus
D. Alloxan diabetes
E. None of the above
289.
Possible fates of pyruvate produced by glycolysis include all of the following (not
necessarily in the same organism), except:
A. Oxidation to CO2 and water through the citric acid cycle.
B. Reduction to lactate by lactate dehydrogenase.
C. * Oxidation to ethanol by alcoholic fermentation.
D. Re-synthesis into glucose by gluconeogenesis.
E. Transamination to alanine.
290.
Pyruvate is converted to lactate during anaerobic metabolism. This reaction
produces one
A. H2O
B. ATP
C. FAD.
D. * NAD+.
E. None of the above.
291.
reactions involved in anaerobic glycolysis of glucose to lactate?
A. FAD/FADH2
B. * NAD+/NADH
C. glyceraldehyde-3-phosphate
D. ATP
E. ADP
292.
Select the molecule that is glucose and galactose connected by a covalent bond:
A.
B.
C.
D.
E.
Maltose
* Sucrose
Cellulose
Lactose
Starch
293.
Select the reaction that is catabolic:
A. * Conversion of glucose into glycogen
B. Synthesis of hemoglobin
C. Conversion of sucrose into glucose and fructose
D. Synthesis of nucleic acids
E. Synthesis of fatty acid
294.
Select the sugar that is an important component of nucleotides:
A. Fructose
B. * Ribose
C. Galactose
D. Glucose
E. Sucrose
295.
Skeletal muscle and brain cells using aerobic respiration produce ____________
molecules of NADH which generates __________ molecules of ATP total.
A. * 2; 4
B. 10; 28
C. 10; 30
D. 10; 36
E. 1: 10
296.
Starch and glycogen are polymers of
A. Fructose
B. Mannose
C. * ?-?D-Glucose
D. Galactose
E. Cellulose
297.
Stimulation of adrenergic receptors has the effect of
A. Degrading glycogen
B. Activating protein kineses A and C
C. Lowering glycogen synthesis
D. * Reducing the effects of insulin
E. All of the above
298.
The ____ is the organ with an absolute requirement for blood glucose as its major
energy source.
A. * Brain
B. Heart
C. Skeletal muscle
D. Liver
E. Kedney
299.
The 4 rate limiting enzymes of gluconeogenesis are
A. Glucokinase, Pyruvate carboxylae phosphoenol pyruvate carboxykinase and
glucose-6-phosphatase
B. * Pyruvate carboxylase, phosphoenol pyruvate carboxykinase, fructose1,6
diphosphatase and glucose-6-phosphatase
C. Pyruvate kinase, pyruvate carboxylase, phosphoenol pyruvate carboxykinase and
glucose-6-phosphatase
D. Phospho fructokinase, pyruvate carboxylase, phosphoenol pyruvate
carboxykinase and fructose 1, 6 diphosphatase
E. None of the above
300.
The active form of glycogen ___ is phosphorylated; the active form of glycogen
___ is dephosphorylated.
A. Hydrolase; dehydrogenase
B. Dehydrogenase; hydrolase
C. Hydrolase; semisynthase
D. * Phosphorylase; synthase
E. Synthase; phosphorylase
301.
The aldolase reaction has a very unfavorable ?G' value (+23.8 kJ/mol). How can
this reaction be maintained as an easily reversible reaction in the cell?
A. * The concentration of F-1,6-BP is usually very high relative to products.
B. Elevation of body temperature to 37?C reverses the equilibrium.
C. Hydrolysis of the 1-phosphate from F-1,6-BP makes the reaction favorable.
D. A and B
E. A, B and C
302.
The appropriate sequence for the transaldolase catalyzed reaction would be (select
a capital letter from A to E):
A. * Reactive enamine attack on the aldehyde group of an aldose
B. beta-cleavage of Schiff base intermediate to release an aldose
C. Formation of Schiff base intermediate of a new ketose
D. Active site lysine forms Schiff base with keto-carbonyl group
E. Schiff base hydrolysed to ketose and enzyme
303.
The blood sugar raising action of the hormones of suprarenal cortex is due to
A. * Gluconeogenesis
B. Glycogenolysis
C. Glucagon-like activity
D. Due to inhibition of glomerular filtration
E. Glycoyisis
304.
The brain prefers to use _______ for fuel.
A. Vitamins
B. Protein
C. Lipids
D. Carbohydrates
E. All of the above
305.
The branching enzyme acts on the glycogen when the glycogen chain has been
lengthened to between glucose units:
A. 1 and 6
B. 2 and 7
C. 3 and 9
D. * 6 and 11
E. 1 and 2
306.
The breakdown of glycogen to form glucose occurs
A. * In the liver by phosphorolysis.
B. In the muscles by phosphorolysis
C. In the liver by hydrolysis.
D. In the muscles by hydrolysis.
E. The first two choices are both correct.
307.
The cause of insulin dependent diabetes (type I) is:
A. * Malfunction of structure and insulin receptors function
B. Decreased insulin synthesis
C. Increased insulin synthesis
D. Increased epinephrine synthesis
E. Decreased epinephrine synthesis
308.
The characteristic enzymes of gluconeogensis are found in the cytosol, except for
A. Pyruvate carboxlyase, which is in the mitrochondria.
B. Fructose-1,6-bisphosphatase, which is in the mitochondria
C. * Glucose-6-phosphatase, which is in the mitrochondria.
D. Fructose-1,6-bisphosphatase, which is in the glycogen granule.
E. Pyruvate carboxylase, which is in the glycogen granule
309.
The complete oxidation of glucose ultimately yields as products:
A. Acetyl CoA
B. Carbon dioxide and water
C. * Lactic acid
D. Pyruvate
E. O2
310.
The complex carbohydrate (polysaccharide) that is digested to the
monosaccharide, glucose, and is found in vegetables, fruits, and grains and is called
A. Maltose.
B. Starch.
C. Glycogen.
D. * Cellulose.
E. None of the above
311.
The conversion of glucose and oxygen into carbon dioxide and water is both a:
A. * Catabolic and reduction reaction
B. Anabolic and oxidation reaction
C. Catabolic and oxidation reaction
D. Anabolic and reduction reaction
E. Only B
312.
The conversion of noncarbohydrate molecules into glucose, is an example of
A. Glycogenolysis.
B. Glycogenesis.
C. * Gluconeogenesis
D. Glycolysis
E. Krebs cycle
313.
The conversion of one molecule of glucose to two molecules of pyruvate results in
the net formation of:
A. Six molecules of water
B. * Two molecules of ATP
C. Three molecules of ATP
D. Thirty-eight molecules of ATP
E. Thirty-nine molecules of ATP
314.
The conversion of oxaloacetate to malate by mitochondrial malate dehydrogenase
can be important in gluconeogenesis because:
A. It's the only way that carbons from oxaloacetate can get out of the mitochondrion.
B. * It is a reduction, so it allows carbons from oxaloacetate and electrons from
C. NADH to get out of the mitochondrion.
D. It is an oxidation, so it allows exchange of oxaloacetate carbons from
theImitochondrion for electrons from NADH in the cytosol.
E. Malate is an obligatory intermediate in gluconeogenesis. This is so confusing that
there is no logical answer to this question.
315.
The disaccharide that most people think of as table sugar is
A. * Sucrose.
B. Lactose.
C. Maltose.
D. Fructose.
E. Only B
316.
The end product of sugar metabolism is/are?
A. Maltose, lactose and sucrose
B. Carbon dioxide, water and heat and energy
C. Urea
D. * Glucose and other simple sugars
E. Glycerol
317.
The energy charge of the cell is:
A. The difference between the charge on the outside and inside of a cell
B. Generated by the sodium-potassium ATPase
C. The overall rate of energy use by the cell
D. * The extent to which the total adenine nucleotide pool is phosphorylated
E. The sum of the ATP, ADP and AMP concentrations in the cell
318.
The energy currency of the cell is
A. Starch
B. * Glycogen.
C. Glucose.
D. ATP.
E. None of the above
319.
The enzymes involved in Phosphorylation of glucose to glucose 6- phosphate are
A. Hexokinase
B. Glucokinase
C. Phosphofructokinase
D. * Both (A) and (B)
E. None of the above
320.
The enzymes of glycolysis are located in the:
A. Mitochondrion
B. Nucleus
C. * Cytoplasm
D. Lysosomes
E. Interstitial fluid
321.
The enzymes of glycolysis are located in the:
A. Mitochondrion
B. Nucleus
C. * Cytoplasm
D. Lysosomes
E. Interstitial fluid
322.
The glycolytic pathway (Glucose --> 2 Pyruvate) is found
A. * In most living organisms.
B. In all living organisms.
C. Primarily in animals.
D. Only in eukaryotes.
E. Only in yeast.
323.
The initiation of glycogen synthesis (ie the very first glucose residue added )
occurs by transfer of glucose from UDP-glucose to the:
A. * 4-position of free glucose.
B. 2-position of fructose.
C. Non-reducing end of maltose.
D. Serine-OH group of glycogen synthase.
E. Tyrosine-OH group of glycogenin.
324.
The linking reaction between glycolysis and Krebs cycle is:
A.
B.
C.
D.
E.
The conversion of pyruvate into lactic acid
The conversion of pyruvate into acetyl-CoA
* The conversion of NAD to power of (exponent) into NADH
The conversion of 1,6 fructose phosphate to 3 phosphoglycerate
None of the above
325.
The low intake of carbohydrate in the diet:
A. Does not influence exercise performance in events lasting less than 10 minutes
B. Affects the resting muscle pH
C. * May impair high intensity exercise performance
D. Results in greater reliance on muscle glycogen during exercise
E. Is associated with a metabolic alkalosis
326.
The major source of carbohydrate in a typical Western diet is:
A. * Starch
B. Cellulose
C. Glycogen
D. Sucrose
E. Saccharin
327.
The metabolic function of the pentose phosphate pathway is:
A. Act as a source of ADP biosynthesis.
B. * Generate NADPH and pentoses for the biosynthesis of fatty acids and nucleic
acids.
C. Participate in oxidation-reduction reactions during the formation of H2O.
D. Provide intermediates for the citric acid cycle.
E. Synthesize phosphorus pentoxide.
328.
The metabolic pathway that results in the splitting of a glucose molecule into two
pyruvate molecules is:
A. * Glycolysis
B. Oxidative phosphorylation
C. Krebs cycle
D. Cori cycle
E. Decarboxilation of pyruvat
329.
The monosaccharide that can be considered the most important source of energy
in the body is:
A. * Glucose
B. Glycogen
C. Starch
D. Fructose
E. Galactose
330.
The most rapid method to resynthesise ATP during exercise is through:
A. Glycolysis
B. * Phosphocreatine breakdown
C. Tricarboxylic acid cycle (Krebs’ cycle)
D. Glycogenolysis
E. Gluconeogenesis
331.
The net production of ATP via substrate-level phosphorylation in glycolysis is:
A. * 2 from glucose and 3 from glycogen
B. 2 from glucose and 4 from glycogen
C. 3 from glucose and 4 from glycogen
D. 3 from glucose and 2 from glycogen
E. 3 from glucose and 3 from glycogen
332.
The normal glucose tolerance curve reaches peak is
A. 15 min
B.
C.
D.
E.
* 1 hr
2 hrs
2. hrs
13 min.
333.
The oxidation of 3 mol of glucose by the pentose phosphate pathway may result in
the production of:
A. 2 mol of pentose, 4 mol of NADPH, and 8 mol of CO2.
B. 3 mol of pentose, 4 mol of NADPH, and 3 mol of CO2.
C. * 3 mol of pentose, 6 mol of NADPH, and 3 mol of CO2.
D. 4 mol of pentose, 3 mol of NADPH, and 3 mol of CO2.
E. 4 mol of pentose, 6 mol of NADPH, and 6 mol of CO2.
334.
The patient L., 46 years, complains on the dry mouth, thirst, often urination,
general weakness. Biochemical blood investigation showed hyperglycemia,
hyperketonemia. In the urine: glucose, ketone bodies. Possible diagnosis is:
A. * Diabetes mellitus
B. Aimentary hyperglycemia
C. acute pancreatitis
D. diabetes insipidus
E. ischemic heart disease
335.
The patient with diabetes mellitus has hypoglycemic coma. The reasons for this
can be:
A. * Overdosage of insulin
B. Starvation
C. Dehydration of the organism
D. Hyperproduction of epinephrin
E. Addison's disease
336.
?The precursor of glycogen in the glycogen synthase reaction is
A. Glucose-1-P
B. Glucose-6-P
C. * DP-glucose
D. TP-glucose
E. None of the above
337.
The process in which glucose is converted to a storage form
A. * Is called glycogenolysis
B. Requires vitamin A.
C. Results in beta oxidation
D. Occurs in the liver.
E. Is very energy consuming
338.
The production of glucose-breakdown products from protein is called
A. Glycolysis
B. * Gluconeogenesis
C. Glycogenesis
D. Glycogenolysis
E. None of the above
339.
The rate of blood lactate accumulation is determined by:
A. The rate of muscle lactate production and the rate of muscle lactate efflux
B. The rate of anaerobic glycolysis
C. The rate of muscle glucose uptake
D. The rate of muscle glycogen depletion
E. * The difference between the rate of lactate appearance and the rate of lactate
clearance
340.
The regulation of normal blood sugar level is accomplished by
A.
B.
C.
D.
E.
* Insulin, glucagons and adrenalin
Cell tissue absorption of glucose from the blood
The breakdown of glycogen by the liver
Glycogenesis and glycogenolysis
All of the above
341.
The regulation of the glycolytic pathway involves
A. * Feedback inhibition by ATP.
B. Allosteric inhibition by ATP.
C. Allosteric stimulation by ADP.
D. All three are correct.
E. Only a) and b) are correct.
342.
The steps of glycolysis between glyceraldehyde 3-phosphate and 3phosphoglycerate involve all of the following except:
A. ATP synthesis.
B. Catalysis by phosphoglycerate kinase.
C. * Oxidation of NADH to NAD+.
D. The formation of 1,3-bisphosphoglycerate.
E. Utilization of Pi.
343.
The synthesis of glucose from lactate, glycerol, or amino acids is called:
A. Glycogenolysis
B. Glycolysis
C. Lipolysis
D. * Gluconeogenesis
E. Transamination
344.
The synthesis of glucose from pyruvate by gluconeogenesis
A. * Requires the participation of biotin
B. Occurs exclusively in the cytosol
C. Is inhibited by elevated level of insulin
D. Requires oxidation/reduction of FAD
E. None of the above
345.
The synthesis of glycogen from glucose subunits is an example of this type of
reaction:
A. Hydrolysis
B. Oxidation
C. * Phosphorylation
D. Condensation
E. None of the above
346.
The tissues with the highest total glycogen content are
A. Muscle and kidneys
B. Kidneys and liver
C. * Liver and muscle
D. Brain and Liver
E. All of the above
347.
The total glycogen content of the body is about ______ gms.
A. 100
B. 200
C. * 300
D. 500
E. 230
348.
The two glycolytic intermediates that directly link glucose metabolism to the
metabolism of triglycerides, and thereby linking carbohydrate to fat metabolism are
A. Pyruvic acid and phosphoglyceraldehyde
B.
C.
D.
E.
349.
A.
B.
C.
D.
E.
Acetyl CoA and pyruvic acid
* Phosphoglyceraldehyde and acetyl CoA
Glucose and pyruvic
A and B
The ultimate electron acceptor in the fermentation of glucose to ethanol is:
* Acetaldehyde.
Acetate.
Ethanol.
NAD+.
Pyruvate.
This is the property of carbohydrates that make them readily dissolvable into
350.
water:
A. The electronegativity of the carbon atom
B. Their ability to form polymers
C. * The presence of hydroxyl groups
D. The long carbon chains they form
E. None of the above
351.
To the resuscitation unit was delivered an infant with the following symptoms:
vomiting, diarrhoea, growth and development malfunction, cataract, mental deficiency.
The diagnosis was galactosemia. The deficit of which enzyme occure?
A. Hexose-6-phosphatase
B. * Galactose-1-phosphate uridiltransferase
C. Uridine diphosphate-glucose-4-epimerase
D. Uridine diphosphate-glucose-pyrophosphorylase
E. Glucose-6-phosphate dehydrogenase
352.
To which additional adrenergic receptor subtype does epinephrine bind in order to
further activate glycogen phosphorylase in liver?
A. Alpha-1
B. Alpha-2
C. Beta-1
D. * A novel beta-3 subtype
E. None of the above
353.
Transfer of high-energy phosphates and/or electrons between carbohydrate
intermediates and other compounds (e.g. ATP, NADH) occurs in the reactions catalyzed
by all of the following enzymes except:
A. Hexokinase
B. * Phosphohexose isomerase
C. Phosphofructokinase-1
D. Glyceraldehyde-3-phosphate dehydrogenase
E. Phosphoglycerate kinase
354.
Transketolase is an enzyme that catalyzes:
A. Transfers of three-carbon units from a ketose to an aldose.
B. Transfer of ketoses into aldoses.
C. * Transfer of xylulose-5-phosphate into ribulose-5-phosphate.
D. TPP-dependent transfer of 2-carbon units to the recipient aldose.
E. Transfer of keto groups from C-2 to C-3.
355.
Two conditions in which gluconeogenesis is increased are
A. Diabetes mellitus and atherosclerosis
B. Fed condition and thyrotoxicosis
C. * Diabetes mellitus and Starvation
D. Alcohol intake and cigarette smoking
E. Riscets and obesyti
356.
A.
B.
C.
D.
E.
357.
A.
B.
C.
D.
E.
UDP-Glucose is converted to UDPGlucuronic acid by
ATP
* GTP
NADP+
NAD+
Glutation
Value of kidney threshold for glucose:
5-7 mmol/l
8-10 mmol/l
* 10-15 mmol/l
2-3 mmol/l
15-20 mmol/l
What chemical species activates the GLUT4 protein to transport glucose into
358.
cells?
A. Adrenaline
B. * Insulin
C. Protein kinase A
D. PFK-2
E. None of the above
359.
When lactose is digested, it yields two monosaccharides called
A. Glucose and glucose.
B. Maltose and glucose.
C. Glucose and fructose.
D. * Glucose and galactose
E. None of the above
360.
When liver glycogen levels are inadequate to supply glucose, especially to the
brain, amino acids and glycerol are used to produce ___________ in a process called
A. Glycogen; glycogenesis
B. Glucose; gluconeogenesis
C. Lipids; lipogenesis
D. * Glucose-6-phosphate; glycogenolys
E. None of the above
361.
When the body metabolizes nutrients for energy, fats yield about _______ times
the energy as carbohydrates or proteins.
A. 1/2
B. 2
C. 4
D. * 6
E. 3
362.
Which coenzymes are reduced in both glycolysis and the Krebs' cycle?
A. FAD
B. Coenzyme A
C. Manganese
D. * NAD
E. Vitamin A
363.
Which method of the measurement of glucose in urine is widely used?
A. Gess
B. Tromer
C. Feling
D. Selivanov
E. * Altgausen
364.
Which of the below-mentioned diseases can be found by the method of sugary
loading?
A. Ishemia
B. Hepatitis
C. * Diabetes mellitus
D. Titania
E. Anemia
365.
Which of the below-mentioned processes is the source of NADPH2 in the
organism?
A. Gluconeogenesis
B. * Pentose-phosphate cycle
C. Glycolysis
D. Glycogenolysis
E. Glycogenogenesis
366.
Which of the following enzymes acts in the pentose phosphate pathway?
A. * 6-phosphogluconate dehydrogenase
B. Aldolase
C. Glycogen phosphorylase
D. Phosphofructokinase-1
E. Pyruvate kinase
367.
Which of the following generates free glucose during the enzymatic breakdown of
glycogen in skeletal muscle?
A. Phosphorylase
B. Debranching enzyme
C. * ?-1-6-amyloglucosidase
D. Amylase
E. Glucose-6-phosphatase
368.
Which of the following groups contain only water soluble vitamins?
A. A , B
B. * B, C
C. C, D
D. D, E
E. A, D
369.
Which of the following is a cofactor in the reaction catalyzed by glyceraldehyde
3-phosphate dehydrogenase?
A. ATP
B. Cu2+
C. Heme
D. * NAD+
E. NADP+
370.
Which of the following is FALSE with regards to GNG
A. Glycerol, lactate, pyruvate, alanine are precursors for GNG
B. The rate of GNG is highest post exercise
C. * GNG occurs in muscle and liver
D. Glucose 6-phosphotase by passes hexokinase
E. One of the major regulators of flux through GNG is substrate availability
371.
Which of the following is likely to be the most important in terms of regulating
hepatic glucose production?
A. Direct innervation of the liver
B. Increase in catecholamines
C. Feedforward mechanisms from the CNS
D. * Increase in glucagon to insulin ratio
E. A only
372.
Which of the following is not a precursor for gluconeogenesis?
A. Alanine
B. Glycerol
C. Oxaloacetate
D. * Acetate
E. Only B
373.
Which of the following is not a true statement?
A. Muscle glycogen is broken down enzymatically to glucose-1-phosphate
B. Elite endurance runners have a high proportion of Type I fibres in their leg
muscles
C. Liver glycogen is important in the maintenance of the blood glucose
concentration
D. * Insulin promotes glucose uptake by all tissues in the body
E. Glucagon has generally antagonistic actions to those of insulin
374.
Which of the following is true concerning glycolysis?
A. It is an aerobic process.
B. A net of 2 molecules of ATP are produced.
C. A total of 38 ATP are produced.
D. * 4 NADH are produced.
E. None of the above
375.
Which of the following processes converts lactic acid to glucose?
A. Citric acid cycle
B. Krebs cycle
C. * Cori cycle
D. None of the above.
E. All of the above
376.
Which of the following processes describes the formation of glycogen from
excess glucose in the blood?
A. Ketogenesis
B. Glycogenolysis
C. Lipogenesis
D. * Gluconeogenesis
E. Only A
377.
Which of the following processes is involved in converting amino acids into
glucose?
A. Glycogenolysis
B. Lipogenesis
C. * Gluconeogenesis
D. Transamination
E. None of the above
378.
Which of the following reaction gives lactose?
A. * UDP galactose and glucose
B. UDP glucose and galactose
C. Glucose and Galactose
D. Glucose, Galactose and UTP
E. UDP fructose and glucose
379.
Which of the following reactions is catalyzed by a transketolase in the pentose
phosphate pathway? (select a capital letter from A to E): І. Fructose-6-P +
glyceraldehyde-3-P . xylulose-5-P + erythrose-4-P II. Erythrose-4-P + fructose-6-P I
sedoheptulose-7-P + glyceraldehyde-3-P III. Sedoheptulose-7-P + glyceraldehyde-3-P I
ribose-5-P + xyulose-5-P
A.
B.
C.
D.
E.
* I only
II and III
II only
I and III
I and II
380.
Which of the following reactions is unique to gluconeogenesis?
A. Lactate Pyruvate
B. Phosphoenol pyruvate pyruvate
C. * Oxaloacetate phosphoenol pyruvate
D. Glucose-6-phosphate Fructose-6-phosphate
E. None of the above
381.
Which of the following statements about glycolysis is false?
A. It results in the formation of two molecules of pyruvic acid.
B. It results in the net gain of four ATP molecules.
C. It can occur with or without oxygen present.
D. * It is an exergonic reaction.
E. Only C
382.
Which of the following statements about pyruvate kinase is true?
A. Active in both glycolysis and gluconeogenesis
B. * Activated by phosphorylation
C. Stimulated by ATP
D. Inhibited by fructose-1,6-bisphosphate
E. None of the above are true
383.
Which of the following statements about the pentose phosphate pathway is
incorrect?
A. It generates CO2 from C-1 of glucose.
B. It involves the conversion of an aldohexose to an aldopentose.
C. It is prominant in lactating mammary gland.
D. It is principally directed toward the generation of NADPH.
E. * It requires the participation of molecular oxygen.
384.
Which of the following statements is true?
A. ATP stimulates phosphofructokinase-1 and inhibits fructose-1,6-bisphosphatase.
B. AMP stimulates both phosphofructokinase-1 and fructose-1,6-bisphosphatase.
C. * AMP stimulates phosphofructokinase-1 and inhibits fructose-1,6bisphosphatase.
D. Citrate stimulates phosphofructokinase-1 and inhibits fructose-1,6bisphosphatase.
E. Citrate stimulates phosphofructokinase-1 and glucose-6-phosphatase.
385.
Which of the following steps from glycolysis is performed by a different enzyme
in gluconeogenesis?
A. Isomerase
B. * Phosphofructokinase (PFK)
C. Glyceraldehyde-3-P dehydrogenase
D. Enolase
E. Aldolase
386.
Which of the following substances are NOT stored in the body?
A. * Amino acids
B. Glycogen
C. Triglycerides
D. Cholesterol
E. None of the above
387.
Which of the following substances is produced in limited quantities in glycolysis
as a result of the metabolism of excessive amounts of alcohol (ethanol)?
A. * NADH
B. Lipids
C. Glucose
D. Glycogen
E. All of the above
388.
Which of these cofactors participates directly in most of the oxidation-reduction
reactions in the fermentation of glucose to lactate?
A. ADP
B. ATP
C. FAD/FADH2
D. Glyceraldehyde 3-phosphate
E. * NAD+/NADH
389.
Which of these does not occur for about four hours after a meal?
A. Glucose enters cells and is either used or stored as glycogen or fat.
B. * Fatty acids and glycerol combine to form fat, which when absorbed is deposited
in adipose tissue.
C. Amino acids are used in protein synthesis, some are used for energy and some are
converted to fats and carbohydrates.
D. Glycerol is converted to glucose, fatty acids are converted to acetyl-CoA, and
acetyl-CoA is used to produce ketone bodies.
E. All of the above
390.
Which one of the following is a rate limiting enzyme of gluconeogenesis?
A. Hexokinase
B. Phsophofructokinase
C. * Pyruvate carboxylase
D. Pyruvate kinase
E. Glucokinase
391.
Which one of the following is NOT a major factor in pancreatic o-cell dysfunction
in patients with type 2 diabetes mellitus?
A. Environmental factors.
B. Genetic factors.
C. Lipotoxicity.
D. Glucotoxicity.
E. * Insulin toxicity.
392.
Which one of the following would not be a characteristic of the postabsorptive
state which occurs late in the morning, late in the afternoon or early in the morning
before breakfast?
A. * Glycogen is broken down to produce glucose.
B. Amino acids are used to synthesize proteins.
C. Triglycerides in adipose tissue break down to glycerol and fatty acids
D. Lactic acid is produced and then converted to glucose in the liver.
E. None of the above
393.
Which statement is true regarding the ‘lactate/anaerobic threshold’ phenomenon?
A. An increase in ventilation is caused by an decrease in both H+ and pCO2
B. It is a poor predictor of marathon performance
C. The power output at which it occurs is decreased with endurance training
D. It is a good indicator of the onset of anaerobic metabolism
E. * Theoretically, a sharp increase in blood lactate, ventilation, and VCO2/VO2 all
occur at the lactate threshold
394.
Which substance causes muscles to ache during strenuous exercise?
A.
B.
C.
D.
E.
Pyruvic acid
Lactose dehydrogenase
Lactate ion
* Lactic acid
None of the above
395.
Which two factors regarding sports drinks affect gastric emptying the most:
A. * Volume, amount of glucose
B. Temperature; carbonation
C. Volume; temperature
D. Amount of glucose; carbonation
E. None of the above
396.
Cerebonic acid is present in
A. Triglycerides
B. * Cerebrosides
C. Esterified cholestrol
D. Sphingomyelin
E. Glycerophospholipids
397.
Energetic function in the organism play the following lipids, except:
A. tryacylglycerol
B. saturated fatty acids
C. diacylglycerol
D. monounsaturated fatty acids
E. * cholesterol
398.
In glycerophospholipids, a polyunsaturated fatty acid is commonly attached to
which of the following carbon atom of glycerol?
A. Carbon 1
B. * Carbon 2
C. Both (A) and (B)
D. Carbon 3
E. None of these
399.
The highest phospholipids content is found in
A. Chylomicrons
B. VLDL
C. LDL
D. * HDL
E. B and D
400.
Which of the following is not essential fatty acid?
A. * Oleic acid
B. Linoleic acid
C. Arachidonic acid
D. Linolenic acid
E. Both (A) and (B)
401.
_____ are the simplest lipids but they may be a part of or a source of many
complex lipids.
A. Triglycerols
B. Carbohydrates
C. Terpenes
D. * Fatty acids
E. Waxes
402.
1 molecule of palmitic acid on total oxidation to CO2 will yield molecules of ATP
(as high energy bonds):
A. * 129
B.
C.
D.
E.
403.
A.
B.
C.
D.
E.
404.
A.
B.
C.
D.
E.
154
83
25
229
9 indicates a double bond between carbon atoms of the fatty acids:
8 and 9
* 9 and 10
9 and 11
9 and 12
None of the above
A digestive secretion that does not contain any digestive enzyme is
Saliva
Gastric juice
Pancreatic juice
* Bile
A and D
A fatty acid which is not synthesized in human body and has to be supplied in the
405.
diet:
A. Palmitic acid
B. Oleic acid
C. * Linoleic acid
D. Stearic acid
E. All of these
406.
A reserve fats in the fat depot mainly are:
A. * tryacylglycerols
B. phospholipids
C. cholesterol
D. fatty acids
E. glycolipids
407.
Acetyl CoA carboxylase regulates fatty acid synthesis by which of the following
mechanism?
A. Allosteric regulation
B. Covalent modification
C. Induction and repression
D. * All of these
E. None of these
408.
acids into mitochondria
A. It can be synthesized in the human body
B. It can be synthesized from methionine and lysine
C. * It is required for transport of short chain fatty
D. Its deficiency can occur due to haemodialysis
E. A and B
409.
After entering cytosol, free fatty acids are bound to
A. Albumin
B. Globulin
C. * Z-protein
D. All of these
E. None of these
410.
All of the following occur during synthesis of ether lipids except:
A. Esterification of an acyl group from fatty acyl CoA to dihydroxyacetone
phosphate
B. Displacement of 1-acyldihydroxyacetone phosphate by a fatty alcohol
411.
412.
413.
414.
415.
416.
417.
418.
C. * Oxidation of the keto group of 1-acyldihydroxyacetone by NADPH
D. Esterification of 1-alkylglycero-3-phosphate to produce 1-alkyl-2-acylglycero-3phosphate
E. None of the above
All the following can be oxidized by ?-oxidation except
A. Palmitic acid
B. * Phytanic acid
C. Linoleic acid
D. Fatty acids having an odd number of carbon atoms
E. B and D
All the following have 18 carbon atoms except
A. Linoleic acid
B. Linolenic acid
C. * Arachidonic acid
D. Stearic acid
E. B and C only
All the following statements about brown adipose tissue are true except
A. It is rich in cytochromes
B. It oxidizes glucose and fatty acids
C. * Oxidation and phosphorylation are tightly coupled in it
D. Dinitrophenol has no effect on it
E. C and D
Amphiphatic lipids are
A. Hydrophilic
B. Hydrophobic
C. * Both (A) and (B)
D. Lipophilic
E. All of these
Arachidonic acid contains the number of double bonds:
A. 2
B. 3
C. * 4
D. 5
E. 1
Bile acids are metabolic products of :
A. * cholesterol
B. phospholipids
C. tryglycerins
D. glycogen
E. glycolipids
Bile is produced by
A. * Liver
B. Gall-bladder
C. Pancreas
D. Intestine
E. All of the above
Biological functions of lipids include
A. Source of energy
B. Insulating material
C. Maintenance of cellular integrity
D. * All of these
E. A and C only
419.
C22 and C24, fatty acids required for the synthesis of sphingolipids in brain are
formed by
A. De novo synthesis
B. * Microsomal chain elongation
C. Mitochondrial chain elongation
D. All of these
E. A and B
420.
Carnitine is required for the transport of
A. Triglycerides out of liver
B. Triglycerides into mitochondria
C. Short chain fatty acids into mitochondria
D. * Long chain fatty acids into mitochondria
E. C and D
421.
Cephalin consists of
A. Glycerol, fatty acids, phosphoric acid and choline
B. * Glycerol, fatty acids, phosphoric acid and ethanolamine
C. Glycerol, fatty acids, phosphoric acid and inositol
D. Glycerol, fatty acids, phosphoric acid and Serine
E. Glycerol, fatty acids, hydrochloric acid and ethanolamine
422.
Ceramide is a precursor to which of the following compounds?
A. Phosphatidyl serine
B. * Sphingomyelin
C. Phosphatidyl glycerol
D. Phosphatidyl choline
E. Phosphatidyl ethanolamine
423.
Cerebrosides are composed of
A. Sphingosine, fatty acids, glycerol and phosphoric acid
B. * Sphingosine, fatty acids, galactose
C. Glycerol, fatty acids, galactose
D. Glycerol, fatty acids, galactose, sphingol
E. Both (A) and (B)
424.
Current concepts concerning the intestinal absorption of triacylglycerols are that
A. They must be completely hydrolysed before the constituent fatty acids can be
absorbed
B. * They are hydrolysed partially and the material absorbed consists of free fatty
acids, mono and diacyl glycerols and unchanged triacyl glycerols
C. Fatty acids with less than 10 carbon atoms are absorbed about equally via lymph
and via portal blood
D. In the absence of bile the hydrolysis of triacyl glycerols is absorbed
E. B and D
425.
Depot fats of mammalian cells comprise mostly of
A. Cholesterol
B. Cholesterol esters
C. * Triacyl glycerol
D. Phospholipids
E. All of these
426.
Dietary triacylglycerols are digested as a result of
A. Lipase action
B. Bile salts
C. Micelle formation
D. Diffusion and absorption by intestinal cells
E. * All of the above
427.
Dipalmitoyl lecithin acts as
A. Platelet activating factor
B. Second messenger for hormones
C. * Lung surfactant
D. Anti-ketogenic compound
E. All of the above
428.
During each cycle of ? -oxidation
A. One carbon atom is removed from the carboxyl end of the fatty acid
B. One carbon atom is removed from the methyl end of the fatty acid
C. * Two carbon atoms are removed from the carboxyl end of the fatty acid
D. Two carbon atoms are removed from the methyl end of the fatty acid
E. Three carbon atoms are removed from the carboxyl end of the fatty acid
429.
During each cycle of ?-oxidation of fatty acid, all the following compounds are
generated except
A. NADH
B. * H2O
C. FAD
D. Acyl CoA
E. Both B and C
430.
During fatty acid metabolism in humans, coenzyme A (CoA) is different from
acyl carrier protein (ACP) in which one of the following ways?
A. Binding of malonic acid with a phosphopantetheine
B. Binding of fatty acids
C. * Function in fatty acid oxidation
D. Function in the cytosol
E. Function in fatty acid synthesis
431.
During the fed state, which of the following occur(s)?
A. Insulin inhibition of stored triacylglycerols' hydrolysis
B. Insulin stimulation of malonyl CoA formation
C. Allosteric inhibition of carnitine acyltransferase I
D. * All of the above
E. A and C only
432.
During the fed state, which of the following occur(s)?
A. Insulin inhibition of stored triacylglycerols' hydrolysis
B. Insulin stimulation of malonyl CoA formation
C. Allosteric inhibition of carnitine acyltransferase I
D. * All of the above
E. A and C only
433.
Fatty acids are activated to acyl CoA by the enzyme thiokinase:
A. NAD+
B. NADP+
C. * CoA
D. FAD+
E. CoQ
434.
Fatty acids are oxidized by
A. ??-oxidation
B. ??-oxidation
C. ??-oxidation
D. * All of these
E. None of these
435.
Fatty acids are oxidized in the ___________.
A. * mitochondrial matrix
B.
C.
D.
E.
cytosol
endoplasmic reticulum
mitochondrial inner membrane space
nucleus
436.
Fatty acids having chain length of 10 carbon atoms enter the
A. * Portal ciruclation
B. Lacteals
C. Systemic circulation
D. Colon
E. Both (A) and (c)
437.
Fatty acids required in the diet of mammals are called
A. Important
B. Dietary
C. Saturated
D. * Essential
E. Esters
438.
Fatty liver may be caused by
A. Deficiency of methionine
B. Puromycin
C. Chronic alcoholism
D. * All of these
E. A and C
439.
Fatty liver may be prevented by all of the following except
A. Choline
B. Betaine
C. Methionine
D. * Ethionine
E. A and C
440.
Fluidity of membranes is increased by the following constituent except
A. * Polyunsaturated fatty acids
B. Saturated fatty acids
C. Integral proteins
D. Cholesterol
441.
For every 2 mol of free glycerol released by lipolysis of triacylglycerides in
adipose tissue
A. 2 mol of triacylglycerides is released
B. 2 mol of free fatty acids is released
C. * 1 mol of glucose can be synthesized in gluconeogenesis
D. 1 mol of triacylglyceride is released
E. 3 mol of acyl CoA is produced
442.
For extramitochondrial fatty acid synthesis, acetyl CoA may be obtained from
A. * Citrate
B. Isocitrate
C. Oxaloacetate
D. Succinate
E. Succsinil-CoA
443.
For the activation of long chain fatty acids the enzyme thiokinase requires the
cofactor:
A. Mg++
B. * Ca++
C. Mn++
D. K+
E. Fe++
444.
Free fatty acids are transported in the blood
A. * Combined with albumin
B. Combined with fatty acid binding protein
C. Combined with ??-lipoprotein
D. In unbound free salts
E. Combined with ??-lipoprotein
445.
Glycerol is transformed into active form with the enzyme:
A. phosphatidate phosphatase
B. * glycerol kinase
C. glycerol phosphate dehydrogenase
D. glycerol phosphate acyltransferase
E. diglyceride-acyl-transferase
446.
Glycerol released from adipose tissue by hydrolysis of triglycerides is mainly
A. * Taken up by liver
B. Taken up by extrahepatic tissues
C. Reutilised in adipose tissue
D. Excreted from the body
E. All of the above
447.
Glycosphingolipids are a combination of
A. * Ceramide with one or more sugar residues
B. Glycerol with galactose
C. Sphingosine with galactose
D. Sphingosine with phosphoric acid
E. None of the above
448.
Hepatic lipogenesis is stimulated by:
A. cAMP
B. Glucagon
C. Epinephrine
D. * Insulin
E. B and D
449.
Hepatoenteral circulation of bile acids has such aim as:
A. fats resynthesis
B. formation of chylomicrons
C. * economical usage of bile acids
D. cholesterol absorption
E. glycerol absorption
450.
Hepatoenteral circulation of bile acids is important for:
A. fat resynthesis
B. formation of chylomicrons
C. * economical usage of bile acids
D. cholesterol absorption
E. glycerin absorption
451.
Higher alcohol present in waxes is
A. Benzyl
B. Methyl
C. Ethyl
D. * Cetyl
E. None of the above
452.
How many cycles of ? -oxidation are required to completely process a saturated
C18 fatty acid?
A. 6
B.
C.
D.
E.
*8
9
18
10
453.
How many cycles of ?-oxidation are required to completely process a saturated
C18 fatty acid?
A. 6
B. * 8
C. 9
D. 18
E. 7
454.
How many enzymes are there in the palmitate synthase multienzyme complex?
A. 5
B. 2
C. 3
D. 10
E. * 7
455.
How many QH2 and NADH are produced by one round of the ?-oxidation
pathway?
A. * 1 each
B. 1 QH2 and 2 NADH
C. 2 each
D. 2 QH2 and 1 NADH
E. all of the above
456.
Human desaturase enzyme system cannot introduce a double bond in a fatty acid
beyond
A. * Carbon 9
B. Carbon 6
C. Carbon 5
D. Carbon 3
E. Carbon 7
457.
Hydrolysis of fats by alkali is called
A. Saponification number
B. * Saponification
C. Both (A) and (B)
D. Lipolysis
E. None of these
458.
If the carbonyl carbon of a fatty acyl CoA molecule with a C10 chain is labeled
with 14C, how many rounds of ?-oxidation are required to produce a radioactively
labeled acetyl CoA?
A. * one
B. two
C. four
D. five
E. six
459.
In adipose tissue, glycerol-3-phosphate required for the synthesis of triglycerides
comes mainly from
A. Hydrolysis of pre-existing triglycerides
B. Hydrolysis of phospholipids
C. * Dihydroxyacetone phosphate formed in.glycolysis
D. Free glycerol
E. All of the above
460.
In cephalin, choline is replaced by
A. Serine
B. * Ethanolamine
C. Betaine
D. Sphingosine
E. Both B and C
461.
In humans, a dietary essential fatty acid is
A. Palmitic acid
B. Stearic acid
C. Oleic acid
D. * Linoleic acid
E. All of the above
462.
In humans, the formation of the fatty acid C-18-?9,?12 can be derived from which
of the following?
A. C-18 cis-?9
B. C-18 cis- ? 6
C. C-18
D. C-16 cis- ? 6, ? 9
E. * C-18 cis- ? 9, ? 12
463.
In mammals, the major fat in adipose tissues is
A. Phospholipid
B. Cholesterol
C. Sphingolipids
D. * Triacylglycerol
E. All of the above
464.
In neutral fats, the unsaponificable matter includes
A. * Hydrocarbons
B. Triacylglycerol
C. Phospholipids
D. Cholsesterol
E. Fatty acids
465.
In oxidation of fatty acids with odd number of carbon atoms propionyl-CoA is
transformed to:
A. malonyl-CoA
B. * succinyl-CoA
C. enoil-CoA
D. butyryl-CoA
E. acetacetyl-CoA
466.
In synthesis of Triglyceride from Glycerol 3-phosphate and acetyl CoA, the first
intermediate formed is
A. ?-diacyl glycerol
B. Acyl carnitine
C. Monoacyl glycerol
D. * Phosphatidic acid
E. None of the above
467.
In the construction of liquid lipids prevail:
A. saturated fatty acids
B. * unsaturated fatty acids
C. dicarboxylic acids
D. keto acid
E. hydroxide acids
468.
In the extra mitochondrial synthesis of fatty acids, CO2 is utilized
A.
B.
C.
D.
To keep the system anaerobic and prevent regeneration of acetyl CoA
In the conversion of malonyl to CoA hydroxybutyryl CoA
* In the conversion of acetyl CoA to malonyl CoA
In the formation of acetyl CoA from 1 carbon intermediates
469.
In ??-oxidation of fatty acids which of the following are utilized as co-enzymes?
A. NAD+ and NADP+
B. FAD H2 and NADH + H+
C. FAD and FMN
D. * FAD and NAD+
E. All of these
470.
Isoprenoids are lipids which do not contain nor are derived from fatty acids and
include
A. Steroids
B. Waxes
C. Terpenes
D. * a and c
E. a, b, and c
471.
Lecithin belongs to:
A. neutral fat
B. * phospholipids
C. waxes
D. steroids
E. glycolipids
472.
Lecithins are composed of
A. * Glycerol + Fatty acids + Phosphoric acid + Choline
B. Glycerol + Fatty acids + Phosphoric acid + Ethanolamine
C. Glycerol + Fatty acids + Phosphoric acid + Serine
D. Glycerol + Fatty acids + Phosphoric acid + Beaine
E. All of these
473.
Like plasmologens, sphingolipids are found in relative abundance in
A. Bacteria
B. Plant cells
C. * Nerve cells
D. Intestinal cells
E. All of the above
474.
Lipase can act only at pH:
A. 2.5–4
B. 3.5–5
C. 4 to 5
D. * 5–7
E. 8-9
475.
Lipase present in the stomach cannot hydrolyze fats owing to
A. Alkalinity
B. Acidity
C. * High acidity
D. Neutrality
E. None of the above
476.
Lipid stores are mainly present in
A. Liver
B. Brain
C. Muscles
D. * Adipose tissue
E. A and D
477.
Lipids are stored in the body mainly in the form of
A. Phospholipids
B. Glycolipids
C. * Triglycerides
D. Fatty acids
E. Cholesterol
478.
Lipids have the following properties:
A. Insoluble in water and soluble in fat solvent
B. High energy content
C. Structural component of cell membrane
D. * All of these
E. A and C only
479.
Lipids may be either hydrophobic or
A. Hydrophilic
B. Organic
C. Inorganic
D. * Amphipathic
E. Soluble in polar solvents
480.
Lisophospholipids, formed after action of phospholipase A2, have the following
feature:
A. activate lipase
B. * hemolytic
C. form conjugates with bile acids
D. participate in phospholipids resynthesis
E. accompany absorption of fatty acids
481.
Long chain acyl CoA penetrates mitochondria in the presence of
A. Palmitate
B. * Carnitine
C. Sorbitol
D. DNP
E. cAMP
482.
Long chain fatty acids are first activated to acetyl-CoA in
A. * Cytosol
B. Microsomes
C. Nucleus
D. Mitochondria
E. EPR
483.
Long chain fatty acids are first activated to acyl CoA in the
A. * Cytosol
B. Mitochodria
C. Ribosomes
D. Microsome
E. EPR
484.
Long chain fatty acids penetrate the inner mitochondrial membrane
A. Freely
B. As acyl-CoA derivative
C. * As carnitine derivative
D. Requiring Na dependent carrier
E. As lipoproteins
485.
Lysolecithin is formed from lecithin by removal of
A. Fatty acid from position 1
B.
C.
D.
E.
* Fatty acid from position 2
Phosphorylcholine
Choline
None of these
486.
Malonyl-CoA reacts with the central
A. * —SH group
B. —NH2 group
C. —COOH group
D. —CH2OH group
E. C and D
487.
Maximum energy produced by
A. * Fats
B. Carbohydrates
C. Proteins
D. Nucleic acids
E. All of these
488.
Mitochondrial lipogenesis requires
A. bicarbonate
B. biotin
C. acetyl CoA carboxylase
D. * NADPH
E. Both (A) and (B)
489.
Mitochondrial membrane is permeable to
A. * Short chain fatty acids
B. Medium chain fatty acids
C. Long chain fatty acids
D. All of these
E. A and B
490.
Mitochondrial thiokinase acts on
A. * Short chain of fatty acids
B. Medium chain fatty acids
C. Long chain fatty acids
D. A and B
E. All of these
491.
Most animal tissues contain appreciable amounts of lipid, when in the form of
depot fat it consists largely of
A. Cholesterol ester
B. Phosphatides
C. Chylomicrons
D. * Triacylglycerol
492.
Most of the reducing equivalents utilized for synthesis of fatty acids can be
generated from
A. * The pentose phosphate pathway
B. Glycolysis
C. The citric acid cycle
D. Mitochondrial malate dehydrogenase
E. Citrate lyase
493.
NADPH required for fatty acid synthesis can come from
A. Pentose phosphate shunt
B. Oxidative decarboxylation of malate
C. Extramitochondrial oxidation of isocitrate
D. * All of these
E. A and B
494.
Net ATP generation on complete oxidation of stearic acid is
A. 129
B. 131
C. * 146
D. 148
E. 120
495.
Net energy generation on complete oxidation of linoleic acid is
A. 148 ATP equivalents
B. 146 ATP equivalents
C. 144 ATP equivalents
D. * 142 ATP equivalents
E. 138 ATP equivalents
496.
Net generation of energy on complete oxidation of palmitic acid is
A. 129 ATP equivalents
B. 131 ATP equivalents
C. 146 ATP equivalents
D. * 148 ATP equivalents
E. 118 ATP equivalents
497.
Net generation of energy on complete oxidation of a 17-carbon fatty acid is
A. Equal to the energy generation from a 16-carbon fatty acid
B. Equal to the energy generation from an 18-carbon fatty acid
C. * Less than the energy generation from a 16-carbon fatty acid
D. In between the energy generation from a16-carbon fatty acid and an 18-carbon
fatty acid
E. None of the above
498.
Niemann-Pick disease results from deficiency of
A. Ceramidase
B. * Sphingomyelinase
C. Arylsulphatase A
D. Hexosaminidase A
E. Lipase
499.
Normal fat content of liver is about _______ gms %.
A. * 5
B. 8
C. 10
D. 15
E. 55
500.
Number of carbon atoms in cholesterol is
A. 17
B. 19
C. * 27
D. 30
E. 25
501.
One functional sub-unit of multi-enzyme complex for de novo synthesis of fatty
acids contains
A. One —SH group
B. * Two —SH groups
C. Three —SH groups
D. Four —SH groups
E. Five —SH groups
502.
Oxidation of fatty acids occurs
A.
B.
C.
D.
E.
In the cytosol
* In the matrix of mitochondria
On inner mitochondrial membrane
On the microsomes
In the EPR
503.
Pancreatic lipase converts triacylglycerols into
A. 2, 3-Diacylglycerol
B. 1-Monoacylglycerol
C. * 2-Monoacylglycerol
D. 3-Monoacylglycerol
E. B and C only
504.
Pancreatic lipase requires for its activity:
A. Co-lipase
B. Bile salts
C. Phospholipids
D. * A and b only
E. All of these
505.
Pancreatic lipose is an enzyme which hydrolyzes fats. It acts as a/an
A. peptidase
B. * hydrolase
C. carbohydrates
D. dehydrogenase
E. None of the above
506.
Phosphatidic acid on hydrolysis yields
A. Glycerol, fatty acids, phosphoric acid, choline
B. * Glycerol, fatty acids, phosphoric acid
C. Glycerol, fatty acids, phosphoric acid, glucose
D. Sphingol, fatty acids, phosphoric acid
E. Both (A) and (B)
507.
Phospholipase A2 is an enzyme which removes a fatty acid residue from lecithin
to form
A. Lecithin fragments
B. Phosphotidic acid
C. Glyceryl phosphate
D. * Lysolecithin
E. Both A and D
508.
Phospholipids are important cell membrane components because
A. They have glycerol
B. They can form bilayers in water
C. * They have both polar and non polar potions
D. They combine covalently with proteins
E. All of these
509.
Phospholipids form bilipid layer of cellular membranes as a result of:
A. hydrophilic features
B. hydrophobic features
C. construction complexity
D. * amphiphilic features
E. molecules stability
510.
Phospholipids help the oxidation of
A. Glycerol
B. * Fatty acids
C. Glycerophosphates
D. Glucose
E. None of these
511.
Phospholipids in the organism perform such functions, except:
A. are a part of cellular membranes
B. * have atherosclerotic properties
C. have antiatherosclerotic activity
D. are lipotropic factors
E. participate in the formation of blood lipoproteins
512.
Plasma becomes milky
A. Due to high level of HDL
B. Due to high level of LDL
C. During fasting
D. * After a meal
E. All of the above
513.
Polar heads of glycerophospholipids may be
A. + charged
B. - charged
C. Neutral
D. A mixture of + and - charges, but not neutral
E. * All of the above
514.
Properties of acetyl CoA-carboxylase include:
A. Activation by fatty acyl CoA
B. Phosphorylation and inactivation by glucagon in adipocytes
C. Inactivation of its phosphorylation by epinephrine in the liver
D. All of the above
E. * None of the above
515.
Propionyl CoA formed oxidation of fatty acids having an odd number of carbon
atoms is converted into
A. Acetyl CoA
B. Acetoacetyl CoA
C. * D-Methylmalonyl CoA
D. Butyryl CoA
E. None of the above
516.
Propionyl CoA is formed on oxidation of
A. Monounsaturated fatty acids
B. Polyunsaturated fatty acids
C. * Fatty acids with odd number of carbon atoms
D. All of these
E. None of these
517.
Refsum’s disease results from a defect in the following pathway except
A. * Alpha-oxidation of fatty acids
B. Beta-oxidation of fatty acids
C. Gamma-oxidation of fatty acids
D. Omega-oxidation of fatty acids
E. None of these
518.
Release of free fatty acids from adipose tissue is increased by all of the following
except
A. Glucagon
B. Epinephrine
C. Growth hormone
D. * Insulin
E. C and D
519.
Reserve fat in the fat depot mainly belong to:
A. * tryacylglycerols
B. phospholipids
C. cholesterol
D. fatty acids
E. glycolipids
520.
Saliva contains a lipase which acts on triglycerides having
A. * Short chain fatty acids
B. Medium chain fatty acids
C. Long chain fatty acids
D. All of these
E. A and B only
521.
Salivary lipase converts dietary triglycerides into
A. * Diglycerides and fatty acids
B. Monoglycerides and fatty acids
C. Glycerol and fatty acids
D. All of these
E. A and b only
522.
Salivary lipase hydrolyses the ester bond at
A. Position 1 of triglycerides
B. Position 2 of triglycerides
C. * Position 3 of triglycerides
D. All of these
E. A and b only
523.
Salivary lipase is secreted by
A. Parotid glands
B. Submandibular glands
C. * Dorsal surface of tongue
D. None of these
E. A and B
524.
Sphingomyelins:
A. * Phospholipids
B. Complex proteins
C. Nitrolipids
D. Alcohols
E. None of these
525.
Sphingosine is synthesized from
A. Palmitoyl CoA and Choline
B. Palmitoyl CoA and ethanolamine
C. * Palmitoyl CoA and serine
D. Acetyl CoA and choline
E. None of above
526.
Splitting off acetyl-CoA in the final reaction of ?-oxidation of fatty acids is
performed by the enzyme:
A. dehydrogenase
B. enolase
C. hydrase
D. lipase
E. * thiolase (acetyl-CoA-acyltransferase)
527.
Sterols are steroids which have
A. A hydroxyl group at position C-17
B. * A hydroxyl group at position C-3
C. The ability to accumulate as plaques in blood vessels
D. 5 fused rings instead of 4 fused rings
E. Hydroxyl groups at both position C-3 and C-17
528.
The ‘free fatty acids’ (FFA) of plasma:
A. metabolically inert
B. mainly bound to alfa -lipoproteins
C. stored in the fat
D. * mainly bound to serum albumin
E. All of the above
529.
The acids which are irreplaceable (essential) for human organism:
A. lipoic acid, stearic acid, palmitic acid
B. oleic acid, linoleic acid, linolenic acid
C. palmitic acid, stearic acid, arachidonic acid
D. * arachidonic acid, linolenic acid, linoleic acid
E. butyric acid, oleic acid, linolenic acid
530.
The amount of total lipids in blood is:
A. * 3-8 g/l
B. 5-10 g/l
C. 2-3 g/l
D. 15-20 g/l
E. 7-12 g/l
531.
The bile acids in bile are in conjugated condition with:
A. cholesterol
B. bilirubin
C. glycine and alanine
D. * glycine and taurine
E. taurine and valine
532.
The carbon chain of fatty acids is shortened by 2 carbon atoms at a time. This
involves successive reactions catalysed by 4-enzymes. These act the following order:
A. Acetyl CoA dehydrogenase, ?-OH acyl CoA dehydrogenase, enoyl hydrase,
thiolose
B. * Acyl CoA dehydrogenase, thiolase, enoyl hydrase, ?-OH acyl CoA
dehydrogenase
C. Acyl CoA dehydrogenase, thiolose, enoyl hydrase, ?-OH acyl CoA
dehydrogenase
D. Enoyl hydrase, ?-OH acyl CoA dehydrogenase,acyl CoA dehydrogenase, thiolose
E. None of the above
533.
The citrate transport system is responsible for:
A. * Transporting from the mitochondrion to the cytosol.
B. Generating cytosolic NADPH from cytosolic NADP+.
C. Both a and b.
D. None of the above.
E. Transporting from the cytosol to the mitochondrion.
534.
The citrate transport system is responsible for:
A. Transporting acetyl CoA from the mitochondrion to the cytosol.
B. Generating cytosolic NADPH from cytosolic NADP+.
C. * Both a and b.
D. Generating cytosolic FADH2.
E. None of the above.
535.
The common precursor for the synthesis of triacylglycerols, phosphatidylcholine
and phosphatidyletanolamine is:
A. choline
B.
C.
D.
E.
CDP-choline
* 1,2-diacylglycerol
none of the above
citrate
536.
The common precursor for the synthesis of triacylglycerols, phosphatidylcholine
and phosphatidyletanolamine is:
A. choline
B. CDP-choline
C. * 1,2-diacylglycerol
D. none of the above
E. citrate
537.
The conversion of the fatty acid palmitate (C16) to carbon dioxide via ?-oxidation,
the citric acid cycle and oxidative phosphorylation yield approximately ______ ATP
equivalents.
A. 3
B. 32
C. * 106
D. 800
E. 48
538.
The desaturation and chain elongation system of polyunsaturated fatty acids are
greatly diminished in the absence of
A. * Insulin
B. Glycagon
C. Epinephrine
D. Thyroxine
E. C and A
539.
The elongation and desaturation of fatty acids take place:
A. * in the cytoplasm
B. in the matrix of mitochondria
C. in the inner membranes of mitochondria
D. in the endoplasmic reticulum
E. in the nucleus
540.
The elongation of fatty acids occurs in which of the diagrammatic structures
shown below?
A. Structure A
B. Structure B
C. * Structure C
D. Structure D
E. Structure E
541.
The end product of cytosol fatty acid synthetase in humans is
A. Oleic acid
B. Arachidonic acid
C. Linoleic acid
D. * Palmitic acid
E. Palmitoleic acid
542.
The energy yield from complete oxidation of products generated by second
reaction cycle of ?-oxidation of palmitoyl CoA will be
A. 5 ATP
B. 12 ATP
C. 17 ATP
D. * 34 ATP
E. 14 ATP
543.
The enzyme acyl-CoA synthase catalyses the conversion of a fatty acid of an
active fatty acid in the presence of
A. AMP
B. ADP
C. * ATP
D. GTP
E. GDP
544.
The enzymes of ? -oxidation are found in
A. * Mitochondria
B. Cytosol
C. Golgi apparatus
D. Nucleus
E. EPR
545.
The fatty acid synthase complex of mammals
A. Is a dimer of unsimilar subunits
B. Is composed of seven different proteins
C. Dissociates into eight different proteins
D. Catalyzes eight different enzymatic steps
E. * Is composed of covalently linked enzymes
546.
The fatty acids containing even number and odd number of carbon atoms as well
as the unsaturated fatty acids are oxidized by
A. ??-oxidation
B. * ??-oxidation
C. ??-oxidation
D. All of these
E. None of these
547.
The first oxidation in the alfa -oxidation of saturated fatty acids is catalyzed by
___________________ and is the conversion of____________________:
A. alfa -hydroxyacyl-CoA dehydrogenase; a primary alcohol to an aldehyde.
B. * acyl-CoA dehydrogenase; a saturated to an unsaturated carbon-carbon bond.
C. acyl-CoA dehydrogenase; a secondary alcohol to a ketone.
D. acyl-CoA dehydrogenase; an aldehyde to a ketone.
E. acyl-CoA dehydrogenase; an alcohol to a ketone.
548.
The first stage of acyl-CoA dehydrogenating in mitochondria causes the formation
of:
A. alfa-keto-acyl-CoA
B. acetyl-CoA
C. alfa-oxyacyl-CoA
D. * enoil-CoA
E. acylcarnitine
549.
The formation of lisophospholipids in the intestine is caused by the action of:
A. * phospholipase A1
B. phospholipase A2
C. phospholipase C
D. phospholipase D
E. phospholipase B
550.
The function of pentose-phosphate cycle in the synthesis of fatty acids and
cholesterol is the following:
A. main energy source
B. source of acetyl-CoA
C. source of malonyl-CoA
D. * source of NADPH2
E. source of NADH2
551.
The hormone, that depresses the lipolysis in the fat tissue:
A. * insulin
B. epinephrine
C. glucagon
D. thyroxin
E. adrenocorticotropin
552.
The importance of phospholipids as constituent of cell membrane is because they
possess
A. Fatty acids
B. * Both polar and nonpolar groups
C. Glycerol
D. Phosphoric acid
E. All of the above
553.
The largest lipoproteins are the __________.
A. * chylomicrons
B. VLDLs
C. LDLs
D. HDLs
E. none of the above.
554.
The level of free fatty acids in plasma is increased by
A. Insulin
B. * Caffeine
C. Glucose
D. Niacin
E. All of the above
555.
The main function of the bile salts is to __________.
A. * emulsify lipids in the intestin
B. act as counterions for the ionized forms of lipids
C. cleave the ester bonds in triacylglycerols to release free fatty acids
D. form the surface layer of chylomicrons
E. none of the above.
556.
The main function of the bile salts is to __________.
A. * transport lipids to the intestinal wall
B. act as counterions for the ionized forms of lipids
C. cleave the ester bonds in triacylglycerols to release free fatty acids
D. form the surface layer of chylomicrons
E. all of the above
557.
The main sources of NADPH for fatty acid biosynthesis is:
A. TCA cycle
B. oxidative phosphorylation
C. * the pentose phosphate pathway
D. glycolysis
E. All of the above.
558.
The major lipid in chylomicrons is
A. * Triglycerides
B. Phospholipids
C. Cholesterol
D. Free fatty acids
E. A and B
559.
The major storage form of lipids is
A. Esterified cholesterol
B.
C.
D.
E.
Glycerophospholipids
* Triglycerides
Sphingolipids
B and C only
560.
The maximum number of double bonds present in essential fatty acid is
A. 1
B. 2
C. 3
D. * 4
E. 7
561.
The nitrogenous base in lecithin is
A. Ethanolamine
B. * Choline
C. Serine
D. Betaine
E. None of above
562.
The oxidation of glycerol in aerobic conditions to CO2 and H2O causes the
formation of :
A. 9 ATP molucules
B. 1 ATP molucules
C. 3 ATP molucules
D. * 22 ATP molucules
E. 15 ATP molucules
563.
The product of lipids digestion are the following, except:
A. glycerol
B. * serine
C. phosphoric acid
D. fatty acids
E. methionine
564.
The rate of fatty acid oxidation is increased by
A. * Phospholipids
B. Glycolipids
C. Aminolipids
D. All of these
E. A and B
565.
The reaction, Palmitoyl-ACP--->Palmitate + HS-ACP proceeds via which of the
following enzymes?
A. * Thioesterase
B. Ketoacyl-ACP synthase
C. Transacylase
D. None of the above
E. Carboxylase
566.
The reaction, Palmitoyl-ACP--->Palmitate + HS-ACP proceeds via which of the
following enzymes?
A. * Thioesterase
B. Ketoacyl-ACP synthase
C. Transacylase
D. All of the above
E. None of the above
567.
The substrate for fatty acids synthesis is:
A. * acetyl-CoA
B. acyl-CoA
C. butiryl-CoA
D. propionil-CoA
E. succinyl-CoA
568.
The triacyl glycerol present in plasma lipoproteins are hydrolyzed by
A. Linqual lipase
B. Pancreatic lipase
C. Colipase
D. * Lipoprotein lipase
E. Both B and D
569.
There are four steps in the ?-oxidation pathway. Some reaction types are listed
below. Give the proper reaction types in the order that they occur in the ?-oxidation
pathway. 1. condensation 2. oxidation 3. reduction 4. thiolysis 5. hydration 6.
phosphorylation 7. rearrangement
A. 1,7,2,2
B. 6,3,4,2
C. 1,2,3,5
D. * 1,5,1,4
E. 1,3,1,4
570.
Triacyl glycerol lipase of fat tissue is activated by:
A. adenosine triphosphate
B. guanosine triphosphate
C. adenosine diphosphate
D. * cyclic adenosine monophosphate
E. guanosine diphosphate
571.
Triacylglycerols are not found in cell membranes because they are
A. Amphipathic
B. * Not amphipathic
C. Not abundant in cells
D. a and c
E. b and c
572.
Triacylglycerols cannot form lipid bilayers because they
A. Have hydrophobic tails
B. * Do not have polar heads
C. Cannot associate with cholesterol
D. Have polar heads
E. Cannot engage in hydrophobic interactions
573.
Triglycerides are
A. Heavier than water
B. Major constituents of membranes
C. * Non-polar
D. Hydrophilic
E. C and D only
574.
Tryacylglycerollipase of fatty tissue is activated by:
A. adenosine triphosphate
B. guanosine triphosphate
C. adenosine diphosphate
D. * cyclic adenosine monophosphate
E. guanosine diphosphate
575.
Waxes contain higher alcohols named as
A. Methyl
B. Ethyl
C. Phytyl
D. * Cetyl
E. None of the above
576.
What is the major role of phospholipase A2?
A. To cleave the phosphate group from phospholipids.
B. To phosphorylate the enzyme enoyl-CoA.
C. * To hydrolyze an ester bond in glycerophospholipids to form
lysophosphoglyceride.
D. To transport glycerophospholipids in the blood.
E. none of the above
577.
What is the sub cellular site for the ?- oxidation of fatty acids?
A. Nucleus
B. * Mitochondria
C. Lysosome
D. Cytosol
E. All of the above
578.
Where the resynthesis of lipids, proper for a human organism, takes place?
A. in the liver
B. in the kidney
C. in the muscles
D. * in the intestinal epithelium
E. in the spleen
579.
Which dietary lipid usually contains no ester bonds?
A. * cholesterol
B. triacylglycerides
C. glycerophospholipids
D. None of the above, all dietary lipids are esterified.
E. All of the above
580.
Which enzyme is needed for the oxidation of odd-chain saturated fatty acids that
is not needed for even-chain fatty acids?
A. methylmalonyl-CoA mutase
B. * propionyl-CoA carboxylase
C. methylmalonyl-CoA racemase
D. all of the above
E. none of the above.
581.
Which enzyme is needed for the oxidation of odd-chain saturated fatty acids that
is not needed for even-chain fatty acids?
A. methylmalonyl-CoA mutase
B. propionyl-CoA carboxylase
C. methylmalonyl-CoA racemase
D. * all of the above
E. none of the above
582.
Which enzyme requires adenosylcobalamin as a cofactor?
A. carnitine acyl transferase I
B. * methylmalonyl-CoA mutase
C. enoyl-CoA hydratase
D. propionyl-CoA carboxylase
E. none of the above
583.
Which lipid form is transported across the inner mitochondrial membrane before
alfa-oxidation?
A. * acylcarnitine
B. fatty acyl CoA
C. acetoacetyl CoA
D. lysophospholipid CoA
E. none of the above.
584.
Which lipid form is transported across the inner mitochondrial membrane before
alfa-oxidation?
A. * acylcarnitine
B. fatty acyl CoA
C. acetoacetyl CoA
D. lysophospholipid CoA
E. all of the above
585.
Which of the below-mentioned acids belongs to bile acids?
A. linoleic acid
B. arachidonic acid
C. oleic acid
D. * cholic acid
E. myristic acid
586.
Which of the below-mentioned metabolites is formed by oxidation of fatty acids
with odd amount of carbon atoms:
A. * propionyl-CoA
B. butyryl-CoA
C. malonyl-CoA
D. enoil-CoA
E. acetacetyl-CoA
587.
Which of the following can be oxidized by alfa -oxidation pathway?
A. Saturated fatty acids
B. Monounsaturated fatty acids
C. Polyunsaturated fatty acids
D. * All of these
E. A and B
588.
Which of the following can be synthesized in the human body if precurors are
available?
A. Oleic acid
B. Palmitoleic acid
C. Arachidonic acid
D. * A and B
E. All of these
589.
Which of the following does (do) not occur during the fasting state?
A. Increased insulin levels
B. Inhibition of lipolysis
C. An increase in the concentration of albumin-bound fatty acids in the blood
D. * a and b only
E. All of the above
590.
Which of the following is a polyunsaturated fatty acid?
A. Palmitic acid
B. Palmitoleic acid
C. * Linoleic acid
D. Oleic acid
E. All of these
591.
Which of the following is a true statement for fatty acid synthesis?
A. It occurs in the mitochondria.
B. The reducing power for synthesis is supplied by NAD and ubiquinone.
C. Both a and b.
D. * None of the above.
592.
593.
594.
595.
596.
597.
598.
599.
600.
E. It occurs in the lysosom.
Which of the following is a true statement for fatty acid oxidation?
A. It occurs in the cytosol.
B. Oxidation requires a three carbon substrate, which transfers a two-carbon unit to
the chain.
C. Both a and b.
D. * None of the above.
E. It occurs in the lysosom.
Which of the following is not a stage of fatty acid synthesis?
A. Condensation of precursors
B. Dehydration
C. Reduction
D. All of the above
E. * None of the above
Which of the following is not used in the synthesis of fatty acids?
A. Cobalamin (vitamin B12)
B. NADPH
C. AMP
D. * FADH2
E. HCO3
Which of the following is not used in the synthesis of fatty acids?
A. Cobalamin (vitamin B12)
B. NADPH
C. AMP
D. * FADH2
E. HCO3
Which of the following is omega-3 polyunsaturated fatty acid?
A. Linoleic acid
B. * Linolenic acid
C. Palmitoleic acid
D. Arachidonic acid
E. A and B
Which of the following is required as a reductant in fatty acid synthesis?
A. NADH
B. * NADPH
C. FADH2
D. FMNH2
E. A and B
Which of the following is the regulated step of fatty acid synthesis in eukaryotes?
A. * Carboxylation of acetyl CoA.
B. Transportation of mitochondrial acetyl CoA into the cytosol.
C. Assembly of the fatty acid chain.
D. All of the above.
E. None of the above.
Which of the following is the regulated step of fatty acid synthesis in eukaryotes?
A. * Carboxylation of acetyl CoA.
B. Transportation of mitochondrial acetyl CoA into the cytosol.
C. Assembly of the fatty acid chain.
D. All of the above.
E. None of the above
Which of the following is true for carboxylation of Acetyl CoA?
A. In animals and yeast, it requires three separate protein subunits.
B.
C.
D.
E.
It is a metabolically reversible reaction.
In bacteria, it is catalyzed by a bifunctional enzyme.
All of the above.
* None of the above.
601.
Which of the following lipid is absorbed actively from intestine?
A. Glycerol
B. Cholesterol
C. Monoacylglycerol
D. * None of these
E. A and B
602.
Which of the following statements correctly describes the enzyme thiokinase?
A. It yields acetyl CoA as a product
B. It yields ADP as a product
C. It yields CoA as a product
D. * It forms CoA thioesters as a product
E. It requires I-ketoacyl CoA as a substrate
603.
Which one of the following compounds is a key intermediate in the synthesis of
both triacylglycerols and phospholipids?
A. CDP-choline
B. * Phosphatidate
C. Triacylglyceride
D. Phosphatidylserine
E. CDP-diacylglycerol
604.
Which one of the following compounds is a key intermediate in the synthesis of
both triacylglycerols and phospholipids?
A. CDP-choline
B. * Phosphatidate
C. Triacylglyceride
D. Phosphatidylserine
E. CDP-diacylglycerol
605.
Which one of the following is not a phospholipid?
A. Lecithin
B. Plasmalogen
C. Lysolecithin
D. * Gangliosides
E. B and C
606.
Why are triacylglycerols able to provide more energy than carbohydrates (gram
for gram)?
A. The triacylglycerols have an extremely high group transfer potential.
B. * The carbohydrates are already in a more oxidized state than the triacylglycerols.
C. The carbohydrates contain fewer carbon-carbon bonds.
D. The triacylglycerols are less soluble in water than the carbohydrates.
E. all of the above
607.
Why is it undesirable to have high concentrations of free fatty acids and
lysophosphoglycerides in cells?
A. They are unstable, free radicals that can react to form toxic substances.
B. They polymerize easily and can cause the cytosol to become too gel-like.
C. * They are amphipathic and act as detergents that can degrade membranes.
D. They inhibit the uptake of pyruvate by mitochondria.
E. all of the above
608.
?-Oxidation of fatty acids requires all the following coenzymes except
A. CoA
B.
C.
D.
E.
FAD
NAD
* NADP
A and B
609.
?-Oxidation of odd-carbon fatty acid chain produces
A. Succinyl CoA
B. Propionyl CoA
C. Acetyl CoA
D. * Malonyl CoA
E. None of the above
610.
_______acts to suppress appetite by inhibiting _____ that is the most powerful
known appetite stimulant.
A. Growth hormone; lipase
B. Serotonin; adrenaline
C. Leptin; neuropeptide Y
D. * Insulin; glucagon
E. None of the above
611.
85% of blood cholesterol is produced
A. * In the liver.
B. By the breakdown of worn out RBCs in the spleen
C. By the ingestion of too much chocolate in the diet.
D. From the breakdown of fatty acids in the digestive system.
E. None of the above
612.
A Chylomicrons
A. VLDL
B. LDL
C. * HDL
D. Only B
613.
A compound normally used to conjugate bile acids is
A. Serine
B. * Glycine
C. Glucoronic acid
D. Fatty acid
E. Cholesterol
614.
A good source of monounsaturated fats is
A. Fat associated with meat.
B. Egg yolks.
C. Whole milk
D. Fish oil.
E. * Olive oil.
615.
A human physiology student had learned all about diabetes and its symptoms. She
was presented with a case of an obese, middle-aged patient, recently diagnosed with Type
2 Diabetes. Indicate which of the following symptoms does NOT apply to this patient:
A. * Hyperglycemia
B. Glucose present in the urine
C. Insulin resistance
D. Marked plasma insulin deficiency
E. Excessive urine production
616.
A hydrocarbon formed in cholesterol synthesis is
A. Mevalonate
B. HMG CoA
C. * Squalene
D. Zymosterol
E. Lanosterol
617.
A lipoprotein associated with high incidence of coronary atherosclerosis is
A. * LDL
B. VLDL
C. IDL
D. HDL
E. None of the above
618.
A lipoprotein inversely related to the incidence of coronary artherosclerosis is
A. VLDL
B. IDL
C. LDL
D. * HDL
E. B and C
619.
A low density lipoproteins would contain
A. A high protein content.
B. A high cholesterol content
C. A low lipid content.
D. A high lipid content.
E. A high triacylglycerol content.
620.
A major cause of atherosclerosis is:
A. * Plaque.
B. High density lipoproteins
C. Unsaturated fats.
D. Monounsaturated fats.
E. Very low density lipoproteins
621.
A metabolite which is common to pathways of cholesterol biosynthesis from
acetyl-CoA and cholecalciferol formation from cholesterol is
A. Zymosterol
B. Lumisterol
C. Ergosterol
D. * 7 Dehydrocholesterol
E. Lanosterol
622.
A total blood cholesterol concentration that is less than _____ mg/dl indicates a
low risk of developing CHD, whereas a concentration that is greater than _____ mg/dl
indicates a high CHD risk.
A. * 3; 9
B. 6; 6
C. 7; 8
D. 7,5; 5
E. None of the above
623.
Acetoacetate transforms into acetone by means of:
A. Dehydrogenating
B. * Decarboxylation
C. Hydration
D. Oxidation
E. Reduction
624.
Acetoacetate transforms into acetone by means of:
A. Dehydrogenating
B. * Decarboxylation
C. Hydration
D. Oxidation
625.
626.
627.
628.
629.
630.
631.
632.
633.
E. Reduction
Activated lecithin cholesterol acyl transferase is essential for the conversion of
A. VLDL remnants into LDL
B. * Nascent HDL into HDL
C. HDL2 into HDL3
D. HDL3 into HDL2
E. VLDL into HDL
Adiposogenital dystrophy is caused by the lack of secretion of:
A. Oxytocin
B. Vasopressin
C. Gonadotropic hormone
D. * Thyrotropic hormone
E. Melanotropic hormone
All of the following are factors that increase one's risk of hypertension EXCEPT:
A. Lack of exercise
B. Gender.
C. A high-salt diet
D. * Diabetes.
E. All of the above
All of the following are high in monounsaturated fatty acids EXCEPT:
A. Safflower oil
B. Canola oil.
C. Cashew nuts.
D. * Olive oil.
E. All of the above
All of the following are risk factors for cardiovascular disease EXCEPT:
A. * Low blood pressure
B. Being overweight
C. Diabetes mellitus
D. Smoking
E. None of the above
All of the following are true regarding HDL-cholesterol levels EXCEPT:
A. That regular physical activity increases HDL
B. That a high saturated fat diet decreases HDL
C. * That a diet high in omega-3 increases HDL
D. That a high fiber diet increases HDL.
E. All of the above
All of the following hormones would suppress appetite except
A. Leptin released by adipose cells
B. Ghrelin produced in the stomach.
C. Cholecystokinin produced during digestion
D. * Insulin.
E. Epinephrin
All of the following tissue are capable of using ketone bodies except
A. Brain
B. Renal cortex
C. * R.B.C.
D. Cardiac muscle
E. Sceletal muscle
All the following can be oxidized by oxidation except
A. Palmitic acid
B. * Phytanic acid
C. Linolic acid
D. Fatty acids having an odd number of carbon atoms
E. Linolenic
634.
All the following statements correctly describe ketone bodies except
A. They may result from starvation
B. They are present at high levels in uncontrolled diabetes
C. They include—OH ?-butyrate and acetone
D. * They are utilized by the liver during long term starvation
E. Only A
635.
An enzyme required for the synthesis of ketone bodies as well as cholesterol is
A. Acetyl CoA carboxylase
B. * HMG CoA synthetase
C. HMG CoA reductase
D. HMG CoA lyase
E. None of the above
636.
Answer both questions a) and b). a) Is the following triacylglycerol chiral? b) Are
any of its hydrolysis products chiral?
A. A) no b) no
B. A) yes b) no
C. * A) yes b) yes
D. A) no b) yes
E. None of the above
637.
Antioxidant have the ability to:
A. * Level the processes of free-radical oxidation of lipids
B. Depress ?-oxidation of fatty acids
C. Activate fat decomposition
D. Activate the processes of peroxidation
E. Depress ketogenesis
638.
Arachidonic acid is classified as essential because it:
A. Is needed as a source of energy
B. Is needed as a source of ?-3 acids
C. * Is needed for synthesis of eicosanoids
D. Can be converted to all of the other fatty acids
E. Readily acts as an antioxidant
639.
Beta-oxidation results in the formation of:
A. Pyruvate.
B. * Acetyl-CoA.
C. Carbon monoxide.
D. Oxaloacetate.
E. None of the above
640.
By observation of a sick child were discovered phenomena of xanthomatosis, liver
increase, retinal hyperlipemia, pain in the stomach, hyperchilomicronemia. What enzyme
activity malfunction possibly caused such pathology?
A. Lipoprotein lipase
B. * Lecithin cholesterol-acyltransferase
C. Tissue tryglycerol lipase
D. Tissue dyglycerol lipase
E. Pancreatic lipase
641.
By oxidation of fatty acids with paired amount of carbohydrate atoms propionilCoA is transformed to:
A. Malonyl-CoA
B. Succinyl-CoA
C. Enoil-CoA
D. Butyryl-CoA
E. Acetacetyl-CoA
642.
Can Acetyl-CoA directly be transported from the mitochondrial matrix to the
cytosol?
A. No, oxaloacetate goes over to the cytosol and then acetyl-CoA is produced
B. No, oxaloacetate and acetyl-CoA forms citrate which travels out to the cytosol
C. Yes, acetyl-CoA is directly transferred into the cytosol
D. Yes, oxaloacetate goes over to the cytosol and then acetyl-CoA is produced
E. None of the above
643.
Cardiovascular function is most commonly impaired by which disease?
A. Arteriosclerosis
B. Hypertension
C. Coronary heart disease
D. Stroke
E. * All of the above
644.
Cholesterol is a
A. Animal sterol
B. M.F. C27 H46O
C. 5 methyl groups
D. * All of these
E. Non produse energy
645.
Cholesterol is a precursor for each of the following, except:
A. Bile salts
B. Vitamin D
C. Testradiol
D. * Vitamin C
E. Lipoproteins
646.
Cholesterol is a precursor in the biogenesis of
A. Vitamin A
B. * Vitamin D
C. Vitamin E
D. None of these
E. Vitamin K
647.
Cholesterol is essential for normal membrane functions because it
A. Cannot be made by higher organisms, e.g. mammals
B. Spans the thickness of the bilayer.
C. Keeps membranes fluid.
D. *Catalyzes lipid flip-flop in the bilayer.
E. Plugs up the cardiac arteries of older men.
648.
Cholesterol is present in all of the following except
A. Egg
B. Fish
C. Milk
D. * Pulses
E. Liver
649.
Cholesterol is transported from liver to extrahepatic tissues by
A. Chylomicrons
B. VLDL
C. HDL
D. * LDL
E. Carnitin
650.
Cholesterol molecule has _______ carbon atoms.
A. * 27
B. 21
C. 15
D. 12
E. 2
651.
Cholesterol performs in the organism the following functions, but:
A. Is a part of cell membranes
B. Substratum for bile acids synthesis
C. Substratum for D vitamin synthesis
D. * Is a source of energy
E. Substratum for steroid hormones synthesis
652.
Cholesterol, bile salts, vitamin D and sex hormones are
A. Mucolipids
B. Glycolipids
C. Phospholipids
D. * Isoprenoid lipids
E. Milk lipids
653.
Chylomicron remnants are catabolised in
A. Intestine
B. Adipose tissue
C. Liver
D. * Liver and intestine
E. Lung
654.
Chylomicron, intermediate density lipoproteins (IDL), low density lipoproteins
(LDL) and very low density lipoproteins (VLDL) all are serum lipoproteins. What is the
correct ordering of these particles from the lowest to the greatest density?
A. LDL, IDL, VLDL, Chylomicron
B. * Chylomicron, VLDL, IDL, LDL
C. VLDL, IDL, LDL, Chylomicron
D. Chylomicron, IDL, VLDL, LDL
E. IDL, Chylomicron, VLDL, LDL
655.
Contributory, or secondary, risk factors for heart disease include
A. Stress, obesity, diabetes.
B. * Heredity, age, high blood pressure
C. Gender, physical inactivity, age
D. Diabetes, smoking, high cholesterol
E. Only C
656.
Drugs called statins are now used to decrease plasma cholesterol levels by
inhibiting the rate-limiting step of cholesterol synthesis. This step is catalyzed by
A. Lipoprotein lipase
B. * ?-ketothiolase
C. Acyl carrier protein
D. HMG-CoA reductase
E. Transacylase
657.
During the absorptive state, the primary source of energy for liver cell functions is
A. Keto acids derived from amino acids.
B. * Ketone bodies derived from fatty acids.
C. Lactic acid derived from muscle glycogen.
D. Glucose derived from intestinal absorption.
E. Pyruvate derived from glycolysis.
658.
Elevated plasma level of the following projects against atherosclerosis:
A.
B.
C.
D.
E.
Chylomicrons
VLDL
HDL
* LDL
A and B
659.
Enzyme lecithin cholesterol acyltransferase (LCAT) performs the following
function:
A. * Removal of cholesterol from perifery tissues and its transportation to the liver
B. Formation of chylomicrons
C. Formation of ?-lipoproteins
D. Resynthesis of lipids
E. Formation of cell membranes structure
660.
Exercise can help reduce the risk of heart disease by:
A. Reducing the levels of HDL
B. * Increasing the levels of HDL
C. Increasing the levels of LDL
D. Increasing blood pressure
E. Increasing the levels of VLDL
661.
Familial hypercholesterolemia is associated with loss of function due to defects in
:
A. * Hydroxymethylglutaryl (HMG)-CoA reductase.
B. Chylomicrons.
C. Hormone-sensitive lipase.
D. Receptors for LDL.
E. Mevalonatereductase
662.
Fatty acid oxidation occurs mostly within mitochondria, but fatty acids can't easily
cross the mitochondrial membrane. How do they pass?
A. Attached to alcohols
B. * As esters of carnitine
C. As amides of glycine
D. Attached to esters of alcohols
E. As bile acids or salts
663.
Fatty acids are activated to acyl-CoAs and the acyl group is further transferred to
carnitine because:
A. Acyl-CoAs easily cross the mitochondrial membrane, but the fatty acids
themselves will not.
B. Fatty acids cannot be oxidized by FAD unless they are in the acyl-carnitine form.
C. Carnitine is required to oxidize NAD+ to NADH.
D. * Acyl-carnitines readily cross the mitochondrial inner membrane, but acyl-CoAs
do not.
E. None of the above is true.
664.
Fatty acids from hyaloplasm to mitochondria are transported with the help of:
A. * Albumins
B. Chylomicrons
C. Alfa- lipoproteins
D. Pre-D-lipoproteins
E. E- lipoproteins
665.
Fluidity of membranes is increased by the following constituent except
A. * Polyunsaturated fatty acids
B. Saturated fatty acids
C. Integral proteins
D. Cholesterol
E. Lanosterol
666.
For extramitochondrial fatty acid synthesis, acetyl CoA may be obtained from
A. * Citrate
B. Isocitrate
C. Oxaloacetate
D. Succinate
E. Lactat
667.
For long-term storage glucose is converted to ______ , while for short-term
storage, glucose is converted to ______.
A. * Fat, glycogen
B. Glycogen, ketone bodies
C. Glycogen, protein
D. Pyruvic acid, fat
E. Fat, pyruvic acid
668.
HDL is synthesized and secreted from
A. Pancreas
B. * Liver
C. Kidney
D. Muscle
E. Lung
669.
HDL is synthesized in
A. Adipose tissue
B. * Liver
C. Intestine
D. Liver and intestine
E. Muscle tissue
670.
High cholesterol can be reduced by all of the following EXCEPT:
A. Proper nutrition.
B. Drug treatment.
C. * Exercise.
D. Increasing saturated fat
E. None of the above
671.
Hormone, which increase the activity of lipolysis in fatty tissue:
A. * Epinephrine
B. Parathyroid hormone
C. Insulin
D. Vasopressin
E. Prostaglandins
672.
How many enzymes are there in the palmitate synthase multiferment complex?
A. * 5
B. 2
C. 3
D. 10
E. 7
673.
In order to estimate your 10-year risk for cardiovascular disease, you need to
know all of the following EXCEPT:
A. Your systolic blood pressure
B. Your weight.
C. * Your age.
D. Your total cholesterol.
E. Your total glucose
674.
In terms of CHD risk, which of the following is undesirable?
A.
B.
C.
D.
E.
* High HDL
Low HDL
Both low HDL and high LDL together
High LDL
Low VLDL
675.
In the blood of a patient is decreased amount of phospholipids, increased
concentration of cholesterol, observed the symptoms of prostaglandin lack. What can
cause such events?
A. Limited usage of carbohydrate
B. * Limited usage of polysaturated fat acids
C. D hypovitaminosis
D. Excess of lipids in food
E. Excess of carbohydrates in food
676.
In the organism of a patient as a result of vitamin B12 deficiency has evolved fatty
hepatosis. What is the reasone for this?
A. Lecithin synthesis violation
B. Haemopoiesis violation
C. Excessive usage of butter
D. * Lack of inner Castle's factor
E. Ketone bodies synthesis violation
677.
In the process of ketogenesis and cholesterol genesis is formed general metabolite.
Name it:
A. Mevalonic acid
B. Malonyl KoA
C. Succinil KoA
D. * D-oxy---?metil-glutaryl CoA
E. E-oxy---?citryl-glutaryl CoA
678.
In those cases were the original fatty acid consists of an uneven number of carbon
atoms, the last cycle of the ?-oxidation will yield a molecule of
A. Acetyl-CoA
B. Glucose
C. * Propionyl-CoA_Succionyl-CoA
D. Oxaloacetate
E. Lactat
679.
Ketogenesis is stimulated by all of the following except
A. Glucagon.
B. Epinephrine.
C. Thryoxine.
D. * Cortisol.
E. Only B
680.
Ketone bodies are formed in
A. Kidney
B. * Liver
C. Heart
D. Intestines
E. Muscles
681.
Ketone bodies are formed in:
A. * Liver
B. Muscles
C. Stomach
D. Intestines
E. Kidney
682.
Ketone bodies are mainly formed (where)?
A. In the brain
B. In the heart
C. * In the liver
D. In kidney
E. None of the above
683.
Ketone bodies are synthesized from fatty acid oxidation products by which of the
following organs?
A. * Liver
B. Skeletal muscles
C. Kidney
D. Brain
E. Heard
684.
Ketosis in partly ascribed to
A. Over production of Glucose
B. * Under production of Glucose
C. Increased carbohydrate utilization
D. Increased fat utilization
E. None of the above
685.
LCAT activity is associated with which of the lipo-protein complex?
A. VLDL
B. Chylomicrones
C. IDL
D. * HDL
E. LDL
686.
leaflet of the membrane lipid bilayer?
A. * Choline phosphoglycerides
B. Ethanolamine phosphoglycerides
C. Inositol phosphoglycerides
D. Serine phosphoglycerides
E. Only C
687.
Lipid stores are mainly present in
A. Liver
B. Brain
C. Muscles
D. * Adipose tissue
E. Lung
688.
Lipids are stored in the body mainly in the form of
A. Phospholipids
B. Glycolipids
C. * Triglycerides
D. Fatty acids
E. Cholesterol
689.
Lipids are:
A. * Stored primarily as triglycerides.
B. Synthesized by beta-oxidation.
C. Broken down by oxidative deamination.
D. All of the above.
E. None of the above
690.
Lipolysis is accelerated by:
A. Insulin
B. * Glucagon
C. Water
D. Testosterone
E. Sucrose
691.
Lipoproteinlipase of tissue capillary is activated by:
A. * Bile acids
B. Heparin
C. Trypsin
D. Epinephrine
E. Ions of Ca
692.
Lipoprotiens may be identified more accurately by means of
A. * Electrophoresis
B. Ultra centrifugation
C. Centrifugation
D. Immunoelectrophoresis
E. Only C
693.
Mark all forms of ketone bodies, except:
A. Acetoacetate
B. Acetone
C. C-hydroxybutyrate
D. * Cellulosa
694.
Mark important molecules and steps directly involved in the oxidation of fatty
acids
A. Hexokinase/Glucokinase.
B. Shuttling of Acetyl-CoA with Carnitin
C. Lactate
D. * The process starts with the activation of a fat molecule, in which a thioester
bond is formed between the carboxyl group of the fatty acid and the thiol group of
coenzyme A (CoA-SH)
E. Acyl-CoA-synthetase and acyl-CoA
695.
Mark the lipid, which usage is to be decreased in the daily food allowance:
A. Cholesterol
B. * Oleic acid
C. Lecithin
D. Monooleateglyceride
E. Phosphatidyl serine
696.
Micelles of fatty acids in water are organized such that the ___ face the solvent
and the ___ are directed toward the interior.
A. Hydrophilic heads; hydrophobic tails
B. Carboxylic acid groups; hydrocarbon chains
C. Hydrocarbon chains; carboxylic acid groups
D. Hydrophobic tails; hydrophilic heads
E. The first and second answers are both correct.
697.
Mitochondrial lipogenesis requires
A. Bicarbonate
B. Biotin
C. Acetyl CoA carboxylase
D. * NADPH
E. Carnitin
698.
Most products of fat digestion are transported in the lymph as
A. Glycerol.
B. Fatty acids.
C. * Chylomicrons
D. Cholesterol.
E. HDL
699.
Normal concentration of ketone bodies in blood is:
A. 2-3 mg/l.
B. 5-10 mg/l.
C. 50-70 mg/l.
D. * 10-20 mg/l.
E. 30-40 mg/l.
700.
Normal range of plasma total phospholipids is
A. 0.2–0.6 mmol/L
B. 0.9–2.0 mmol/L
C. * 1.5–3.6 mmol/L
D. 2.8–5.3 mmol/L
E. 9,2-11.6 mmol/L
701.
Number of carbon atoms in cholesterol is
A. 17
B. 19
C. * 27
D. 30
E. 18
702.
Old-time physicians often diagnosed illness on the basis of the odor of patients.
Untreated diabetics sometimes have breath with a "fruity" odor due to the presence of a
volatile ketone body. Which one?
A. ?-Hydroxybutyric acid
B. Pyruvic acid
C. Stearic acid
D. Acetoacetic acid
E. * Acetone
703.
Oxidation of fatty acids occurs
A. In the cytosol
B. * In the matrix of mitochondria
C. On inner mitochondrial membrane
D. On the microsomes
E. on the lysosoms
704.
Pancreatic lipase converts triacylglycerols into
A. 2, 3-Diacylglycerol
B. 1-Monoacylglycerol
C. * 2-Monoacylglycerol
D. 3-Monoacylglycerol
E. 5-Monoacylglycerol
705.
Pancreatic lipase requires for its activity:
A. Co-lipase
B. Bile salts
C. Phospholipids
D. * All of these
E. Monoacylglicerols
706.
Phospholipids are important cell membrane components because
A. They have glycerol
B. They can form bilayers in water
C. They have both polar and non polar potions
D. * They combine covalently with proteins
E. None of these
707.
Products of ?-oxidation of palmitoyl CoA include:
A. Propionyl CoA, NADH and FADH2.
B. Acetyl CoA, NADPH and FAD.
C. * Acetyl CoA, FADH2 and NADH.
D. Propionyl CoA, NADH and acetyl CoA.
E. Pyruvat and lactat
708.
Serum cholesterol is decreased in
A. Endemic goitre
B. Thyrotoxicosis
C. Myxoedema
D. Cretinism
E. Pellsgra
709.
Some triacylglycerols are exogenous, and others are endogenous (synthesized
within the body). Endogenous triacylglycerols are typically synthesized in:
A. Erythrocytes
B. Liver
C. Heart tissue
D. Muscle tissue
E. * Adipose tissue
710.
Some ways to reduce the risk of cardiovascular disease are:
A. Quit smoking and reduce saturated fat intake.
B. Reduce exercise to prevent stress on the heart.
C. Obtain 50% or more of total calorie intake from fat and decrease carbohydrate
intake.
D. * All of the above.
E. None of the above
711.
Stress is considered a risk factor for heart disease because it can cause all of the
following EXCEPT:
A. An increase in blood pressure.
B. An increase in HDL levels.
C. * An increase in smoking.
D. An increase in total blood cholesterol
E. Only D
712.
The ‘Committed step’ in the biosynthesis of cholesterol from acetyl CoA is
A. Formation of acetoacetyl CoA from acetyl CoA
B. Formation of mevalonate from HMG CoA
C. * Formation of HMG CoA from acetyl CoA and acetoacetyl CoA
D. Formation of squalene by squalene synthetase
E. Only A
713.
The action of bile in lipid digestion is to:
A. * Emulsify fats by a detergent action
B. Chemically degrade triglycerides to fatty acids and glycerol
C. Chemically convert triglycerides to diglycerides
D. Chemically reduce cholesterol esters to cholesterol
E. A and B
714.
The amount of cholesterol in blood plasma is normal:
A. * 3-7 mmol/l
B. 10-15 mmol/l
C. 20-25 mmol/l
D. 1-3 mmol/l
E. 5-7 mmol/l
715.
The amount of total lipids in blood plasma is normal:
A.
B.
C.
D.
E.
4-8 g/l
5-10 g/l
2-3 g/l
15-20 g/l
7-12 g/l
716.
The anabolism/synthesis of lipids mainly take place in the
A. * Cytosol
B. Mitochondrial matrix
C. Nucleus
D. Cell membrane
E. Lysosoms
717.
The biggest risk factor for sudden cardiac death is:
A. Heredity.
B. Smoking
C. Hipertension.
D. * High blood cholesterol levels
E. Low blood cholesterol levels
718.
The bile salts are:
A. Enzymes for digesting food in the small intestines
B. * Detergents for breaking up large fat globules to small ones
C. Stimulants to pancreatic secretion of enzymes
D. The major ingredients of gallstones
E. Reabsorbed primarily by the gallbladder
719.
The catabolism/breakdown of lipids (?-oxidation) mainly take place in the
A. Cytosol
B. * Mitochondrial matrix
C. Nucleus
D. Cell membrane
E. Lysosoms
720.
The complete h-oxidation of 1 mole of palmitic acid to CO2 and H2O yields
_______ net moles of ATP.
A. 92
B. 94
C. 106
D. 108
E. * 129
721.
The correct sequence of events that occurs when fatty acids are used to generate
ATP is:
A. Beta-oxidation, electron transport chain, deamination.
B. Electron transport chain, beta-oxidation, glycolysis.
C. * Beta-oxidation, TCA cycle, electron transport chain.
D. Glycolysis, beta-oxidation, TCA cycle.
E. Glycolysis, electron transport chain, deamination
722.
The cyclization of squalene causes the formation of:
A. * Lanosterol
B. B-oxy---methylglutaryl-CoA
C. Phosphomevalonate
D. Acetoacetyl-CoA
E. Cholesterol
723.
The fatty degeneration of liver is prevented by lipotropic matters. Which of the
belowmentioned matters belongs to them?
A. * Methionine
B.
C.
D.
E.
Cholesterol
Bilirubin
Glycine
Glucose
724.
The fatty degeneration of liver is prevented by lipotropic matters. Which of the
belowmentioned matters belongs to them?
A. * Methionine
B. Cholesterol
C. Bilirubin
D. Glycine
E. Glucose
725.
The following contains the least cholesterol:
A. * Milk
B. Meat
C. Butter
D. Cheese
E. Suet
726.
The function of pentose-phosphate cycle in the synthesis of fatty acids and
cholesterol is the following:
A. Main energy source
B. Source of acetyl-CoA
C. Source of malonyl-CoA
D. * Source of NADPH2
E. Source of NAPH2
727.
The highest phospholipids content is found in
A. Chylomicrons
B. VLDL
C. LDL
D. * HDL
E. None of the above
728.
The hormone, that depresses lipolysis in fatty tissue:
A. * Insulin
B. Epinephrine
C. Glucagon
D. Thyroxine
E. Adrenocorticotropin
729.
The hormone, that depresses lipolysis in fatty tissue:
A. * Insulin
B. Epinephrine
C. Glucagon
D. Thyroxine
E. Adrenocorticotropin
730.
The hormone, which favours oxidation of cholesterol side chain and its excretion
with bile:
A. Parathyroid hormone
B. * Epinephrine
C. Insulin
D. Thyroxine
E. Vasopressin
731.
The key intermediate in the synthesis of cholesterol following the formation of
acetoacetyl CoA are. Any one of the following:
A. Hydroxymethylglutaryl CoA or HMG CoA
B.
C.
D.
E.
* Mevalonate (or mevalonic acid)
Isoprenoid
Squalene
Lanosterol
732.
The key intermediate in the synthesis of cholesterol following the formation of
acetoacetyl CoA are. Any one of the following:
A. Hydroxymethylglutaryl CoA or HMG CoA
B. * Mevalonate (or mevalonic acid)
C. Isoprenoid
D. Squalene
E. Lanosterol
733.
The major lipid in chylomicrons is
A. * Triglycerides
B. Phospholipids
C. Cholesterol
D. Free fatty acids
E. Lanosterol
734.
The major product of the fatty acid synthase reaction is:
A. Propionyl CoA.
B. * Palmitate.
C. Palmitoyl CoA.
D. Acetyl CoA.
E. Lactat
735.
The major source of cholesterol in arterial smooth muscle cells is from
A. IDL
B. * LDL
C. HDL
D. Chylomicrons
E. IDL
736.
The major storage form of lipids is
A. Esterified cholesterol
B. Glycerophospholipids
C. * Triglycerides
D. Sphingolipids
E. All of the above
737.
The normal content of cholesterol in blood is:
A. 5-10 mmol/l.
B. 10-15 mmol/l.
C. * 3-8 mmol/l.
D. 2-5 mmol/l.
E. 3,3-5,5 mmol/l
738.
The official medical measurement of obesity is the
A. Weight of a person compared to their height
B. * Body mass index.
C. Proportion of bone density to weight.
D. Basal metabolic rate.
E. Only C
739.
The oxidation of glycerin in aerobic conditions to CO2 and H2O taking into
account energy spending causes the formation of :
A. 9 ATP molucules
B. 1 ATP molucules
C. 3 ATP molucules
D. * 22 ATP molucules
E. 15 ATP molucules
740.
The patient has hyperfunctioning of thyroid gland. What changes of lipid
metabolism may take place?
A. Hypercholesterolemia
B. Hypophospholipidemia
C. * Decrease of amount of free fatty acids in blood
D. Steatorrhea
E. Adiposity
741.
The patient has hyperfunctioning of thyroid gland. What changes of lipid
metabolism may take place?
A. Hypercholesterolemia
B. Hypophospholipidemia
C. * Decrease of amount of free fatty acids in blood
D. Steatorrhea
E. Adiposity
742.
The presence of ketone bodies in the urine indicates increased metabolism of:
A. * Amino acids.
B. Lactic acid.
C. Fatty acids.
D. Nucleic acids
E. Vitamins
743.
The presence of ketone bodies in the urine indicates increased metabolism of:
A. * Amino acids.
B. Lactic acid.
C. Fatty acids.
D. Nucleic acids
E. Vitamins
744.
The result of hyperketonemia is:
A. Fatty liver degeneration
B. * Acidosis
C. General adiposity
D. Emaciation
E. Atherosclerosis
745.
The sources of NADPH for synthesis of fatty acids and cholesterol are:
A. Malate dehydrogenase and glycolysis
B. Lactate dehydrogenase and oxidative branch of the pentose phosphate pathway.
C. Malate dehydrogenase and non-oxidative branch of the pentose phosphate
pathway.
D. * Malic enzyme and oxidative branch of the pentose phosphate pathway.
E. None of the above
746.
The substratum for fatty acids synthesis is:
A. * Acetyl-CoA
B. Acyl-CoA
C. Malonyl-CoA
D. Propionil-CoA
E. Succinyl-CoA
747.
The synthesis of 1 mole of palmitic acid from 8 moles of acetyl CoA requires
_____ moles of ATP.
A. 2
B. 4
C. 6
D. * 7
E. 8
748.
This interferes with cholesterol absorption
A. Lipoprotein lipase
B. Creatinase
C. ?-sitosterol
D. * 7-dehydrocholesterol
E. None of the above
749.
Uncontrolled type I diabetes mellitus
A. * Stimulates increased tissue ketone utilization.
B. If untreated leads to convulsions and death.
C. Stimulates hepatic glycogen synthesis.
D. Results in hypotension.
E. All of the above
750.
Vasodilators that may mediate local myocardial control of coronary vascular tone
A. Adenosine
B. * Bradykinin
C. Prostaglandins
D. Potassium ions
E. Hydrogen ions
751.
Very low density lipoproteins are also known as
A. ?-lipoproteins
B. * Pre B--lipoproteins
C. C-lipoproteins
D. None of these
E. Pre-?-lipoproteins
752.
VLDL remnant may be converted into
A. VLDL
B. * LDL
C. HDL
D. Chylomicrons
E. None of the above
753.
What are the steps you can take to reduce your high blood cholesterol?
A. Follow the TLC Diet (low saturated fat, low cholesterol).
B. Be more physically active.
C. Lose weight if you are overweight.
D. * Take your cholesterol lowering medication if prescribed.
E. All of the above.
754.
What changes in lipid metabolism will be caused by insufficiency of somatotropin
excretion?
A. Emaciation
B. * Fatty liver degeneration
C. Atherosclerosis
D. Adiposity
E. Ketonemia
755.
What is the state called when there is a high level of ketone bodies in the blood?
A. * Ketosis/ketoacidosis
B. Hyperglycemia
C. Hypothermia
D. Hyperaminoacidemia
E. Hypoglycemia
756.
What is the sub cellular site for the W- oxidation of fatty acids?
A.
B.
C.
D.
E.
ucleus
* Mitochondria
Lysosome
Cytosol
Nucleus
757.
What is true regarding the production of ketone bodies:
A. It involves a number of reactions
B. It’s a single reaction with only one enzyme
C. The first step is a condensation reaction in which two molecules acetyl-CoA are
transformed into acetyl-CoA
D. The second step is another condensation reaction and HMG-CoA is formed
E. * Acetoacetate can be either reduced to ?–hydroxybutyrate or through a
decarboxylation reaction it can form acetone
758.
What is/are true for lipids?
A. They are polar and soluble in water.
B. They are nonpolar and soluble in water.
C. * Steroids, glycolipids, fatty acids, triacylglycerols, phosphoacylglycerols a
sphingolipids are all examples of lipids.
D. Pyruvate, glucose and lactate are all examples of lipids.
E. None of the above
759.
What is/are true?
A. The synthesis of fatty acids produces energy in the form of ATP
B. The catabolism of fatty acids costs ATP
C. The synthesis of fatty acids costs energy in the form of ATP
D. * The catabolism of fatty acids produces ATP
E. Only C
760.
Where in the cell is performed the synthesis of palmitic acid?
A. * Cytoplasm
B. Nucleus
C. Lysosomes
D. Mitochondria
E. Microsomes
761.
Which are the cholesterol esters that enter cells through the receptor-mediated
endocytosis of lipoproteins hydrolyzed?
A. Endoplasmin reticulum
B. * Lysosomes
C. Plasma membrane receptor
D. Mitochondria
E. Nucleus
762.
Which compounds/coenzymes belong to fatty acid synthesis and not to the
catabolism of the same?
A. FAD
B. NAD+
C. NADH
D. Coenzyme A
E. * Acyl carrier protein (ACP)
763.
Which food does not have a lot of cholesterol?
A. Steak
B. * Banana
C. Liver.
D. Butter
E. Egg.
764.
A.
B.
C.
D.
765.
A.
B.
C.
D.
E.
766.
A.
B.
C.
D.
E.
767.
A.
B.
C.
D.
E.
768.
A.
B.
C.
D.
E.
769.
A.
B.
C.
D.
E.
770.
A.
B.
C.
D.
E.
771.
A.
B.
C.
D.
E.
772.
A.
B.
C.
D.
Which is not a ketone body?
* Dihydroxyacetone
Acetoacetate
Acetone
D-hydroxybutyrate
Which lipoprotein contains high levels of cholesterol?
HDL
Chylomicron
VLDL
IDL
* LDL
Which of the following are considered major risk factors for CHD?
Obesity, gender, heredity
* Hypertension, high cholesterol, sedentary lifestyle
Diabetes, obesity, stress
Smoking, diabetes, stress
Only C
Which of the following does NOT characterize Type 1 diabetes mellitus?
Increased glucose utilization due to excessive glucose availability
Excessive urination due to the osmotic effect of glucose in the urine
Acidosis due to excessive levels of ketones (ketone bodies) in the blood.
* Dehydration.
Hyperglycemia
Which of the following effect of insulin is correct?
Activates the oxidation of fatty acids.
Activates the lipolysis.
Inhibits the synthesis of lipids.
* Enhances the synthesis of lipids.
None of the above
Which of the following has the highest cholesterol content?
Meat
Fish
* Butter
Milk
Cheese
Which of the following has the highest cholesterol content?
* Egg yolk
Egg white
Meat
Fish
Milk
Which of the following is not a lipoprotein?
Chylomicron
* Creatinine
LDL
HDL
VLDL
Which of the following is NOT a long-term consequence of untreated diabetes?
Osteoporosis
Heart disease
Blindness
Kidney disorders leading to kidney failure
E. * Peripheral neuropathy
773.
Which of the following is not specifically required in the synthesis of fatty acids?
A. Biotin
B. Acetyl -CoA
C. Malonyl-CoA
D. NADH
E. * HCO3 - (CO2)
774.
Which of the following is not true of the reaction in which malonyl-CoA is
produced during fatty acid synthesis?
A. It requires CO2(or bicarbonate)
B. The cofactor biotin
C. It requires the acyl carrier protein(ACP)
D. * It is stimulated by citrate
E. None of the above
775.
Which of the following lipoproteins would contribute to a measurement of plasma
cholesterol in a normal person following a 12 hr fast?
A. * High density lipoprotiens
B. Low density lipoproteins
C. Chylomicron
D. Chylomicron remnants
E. All of the above
776.
Which of the following molecules will be formed from the carbon skeleton of a
ketogenic amino acid?
A. * Acetyl-CoA
B. Oxaloacetate
C. Pyruvate
D. Acetoacetyl-CoA
E. All of the above
777.
Which of the following statement is correct about membrane cholesterol?
A. The hydroxyl group is located near the centre of the lipid layer
B. Most of the cholesterol is in the form of a cholesterol ester
C. * The steroid nucleus form forms a rigid, planar structure
D. The hydrocarbon chain of cholesterol projects into the extracellular fluid
E. None of the above
778.
Which of the following statements about genetic influences on obesity is FALSE?
A. * A family history of obesity increases your chances of becoming obese by 25-30
percent.
B. Fifty percent of children with two obese parents are also obese.
C. Genes play a significant role in how your body balances calories and energy.
D. Genes influence body fat and fat distribution.
E. Only C
779.
Which of the following statements about lipoproteins is TRUE?
A. VLDL transports cholesterol from liver to adipose tissue.
B. LDL transports cholesterol from peripheral tissues to the liver.
C. HDL transports cholesterol from the liver to the peripheral tissues.
D. * Chylomicrons transport triacylglycerols from the small intestine to the liver and
adipose tissue.
E. All the above are true.
780.
Which of the following statements about membrane lipids is true?
A. Lecithin (phosphatidylcholine), which is used as an emulsifier in margarine and
chocolate, is a sphingolipid.
B. * Glycerophospholipids contain fatty acids linked to glycerol through amide
bonds.
C. Some sphingolipids include oligosaccharides in their structure.
D. Glycerophospholipids are found only in the membranes of plant cells.
E. All of the above
781.
Which of the following statements regarding the biosynthesis of cholesterol is
FALSE?
A. Cholesterol has the same carbon skeleton as its biosynthetic precursor, lanosterol.
B. * Cholesterol is derived from the dimerization of farnesyl pyrophosphate in a
head-to-head manner.
C. Cyclization of squalene to cholesterol is initiated by acid-catalyzed ring opening
of an epoxide.
D. Cholesterol has three fewer carbon atoms than lanosterol.
E. Only C
782.
Which of these foods does NOT contain cholesterol?
A. Liver
B. Butter
C. Cheese
D. Eggs
E. * Molasses
783.
Which one of the following pairs of metabolic pathways occur in mitochondria?
A. Fatty acid synthesis, ketogenesis
B. * Citric acid cycle, C-oxidation of saturated fatty acids
C. Ketogenesis, cholesterol synthesis
D. Pentose phosphate pathway, glycolysis
E. Gluconeogenesis, pyrimidine synthesis
784.
Which one of the following statements about lipids is FALSE?
A. A fatty acid group is present in acylglycerols and sphingolipids.
B. Vitamin A functions in vision.
C. Most phospholipids can by degraded by phospholipases A1, A2, C and D.
D. The synthesis of cholesterol requires acetyl CoA, ATP and NADPH.
E. * The more double bonds present in a fatty acid the higher the melting point.
785.
Which risk factors place an individual at risk for cardiovascular disease?
A. Gender
B. Socioeconomic status
C. Ethnicity
D. Family history
E. * Age
786.
Which substrate is used for the activation of acetoacetate in peripheral tissues?
A. * Succinyl-CoA.
B. Acetyl-CoA.
C. Acetoacetyl-CoA.
D. CoA-SH.
E. Aceton
787.
Which vitamin is derived from cholesterol?
A. A
B. B12
C. * D
D. K
E. E
788.
With the urine daily healthy human organism excretes the ketone bodies in
amount of:
A.
B.
C.
D.
E.
* 40 mg
80 mg
100 mg
10 mg
50 mg
789.
With the urine daily healthy human organism excretes the ketone bodies in
amount of:
A. * 40 mg
B. 80 mg
C. 100 mg
D. 10 mg
E. 50 mg
790.
А. Lactic acid
A. Glucose
B. Stearic acid
C. Acetoacetic acid
D. * Acetic acid
791.
7 years old child was carried into the hospital in the state of allergic shock, which
evolved after wasp's sting. The increased concentration of histamine was detected in the
blood. This amine is formed as a result of reaction of:
A. Dehydration
B. Deamination
C. Reduction
D. * Decarboxylation
E. Hydrooxidation
792.
Chymotrypsin in the small intestine hydrolyzes peptide linkages containing
A. Alanine
B. * Pheynl alanine
C. Valine
D. Methionine
E. B and C
793.
Coenzyme of amino acids decarboxylases is:
A. Thymidine diphosphate
B. Flavin adenine dinucleotide
C. Flavin mononucleotide
D. HS-CoA
E. * PLP
794.
Free ammonia is released during
A. Oxidative deamination of glutamate
B. Catabolism of purines
C. Catabolism of pyrimidines
D. * All of these
E. Both A and B
795.
Hyperchlorhydria is:
A. A.Increase of gastric juices general acidity
B. B.Increase of bonded hydrochloric acid in gastric juice
C. * Increase of free hydrochloric acid in gastric juice
D. Increase of lactic acid in gastric juice
E. Increase of pyruvic acid in gastric juice
796.
In small intestine trypsin hydrolyzes peptide linkages containing
A. * Arginine
B. Histidine
C. Serine
D. Aspartate
E. A and D
797.
Injuries and burns cause the development of negative nitric balance that is the
result of increase of:
A. Glycolysis
B. Lipolysis
C. * Proteolysis
D. Phosphorolysis
E. Fibrinolysis
798.
Intrinsic factor is chemically a:
A. Protein
B. * Glycoprotein
C. Mucopolysaccaride
D. Peptide
E. Amino acid
799.
Most amino acids are substrates for transamination except
A. Alanine
B. * Threonine
C. Serine
D. Valine
E. A and B
800.
Name the compound in the urine, which is a test of protein putrefaction processes
in the intestine:
A. Urates
B. Urea
C. Creatinine
D. * Indican
E. Lactic acid
801.
Oxidative conversion of many amino acids to their corresponding –ketoacids
occurs in mammalian:
A. * Liver and kidney
B. Adipose tissue
C. Pancreas
D. Intestine
E. Stomach
802.
The amino acids abstracted from the liver are not utilized for repair or special
synthesis but are broken down to
A. Keto acids
B. Sulphur dioxide
C. Water
D. * Ammonia
E. A and D
803.
The amount of general acidity in gastric juice of a patient P. is normal. Which of
the listed numbers is true in this case?
A. A.30-40 mM/l
B. * B.40-60 mM/l
C. 20-30 mM/l
D. 30-50 mM/l
E. 20-40 mM/l
804.
The example of chromoprotein:
A. Salmine
B.
C.
D.
E.
* Catalase
Zein
Gliadin
All of these
805.
The metabolism of all proteins ingested over and above the essential requirements
is called
A. * Exogenous metabolism
B. Endogenous metabolism
C. Both (A) and (B)
D. Metabolism
E. None of these
806.
The transaminase activity needs the coenzyme:
A. ATP
B. * B6 – PO4
C. FAD+
D. NAD+
E. TPP
807.
The unwanted amino acids abstracted from the tissues are either used up by the
tissue or in the liver converted into
A. Ammonia
B. * Urea
C. Ammonium salts
D. Uric acid
E. B and C
808.
Transamination is a
A. Irreversible process
B. * Reversible process
C. Both (A) and (B)
D. Linear pathway
E. None of these
809.
A limiting amino acid is an essential amino acid
A. * That is most deficient in proteins
B. That is most excess in proteins
C. That which increases the growth
D. That which increases the weight gain
E. None of these
810.
A positive nitrogen balance occurs
A. * In growing infant
B. Following surgery
C. In advanced cancer
D. In kwashiorkor
E. A and D
811.
Absorption of Vitamin B12 requires the presence of:
A. Pepsin
B. Hydrochloric acid
C. Intrinsic factor
D. * Boh (B) and (C)
E. Rennin
812.
Achylia gastrica is said to be when absence of
A. Pepsin only
B. * Both pepsin and HCl
C. HCl only
D. Bile
E. None of these
813.
Achylia gastrica is said to be when absence of
A. Pepsin only
B. * Both pepsin and HCl
C. HCl only
D. Bile
E. All of these
814.
Achylia is lack of:
A. Gastric juices acidity
B. Pepsin in gastric juices
C. * Pepsin and hydrochloric acid in gastric juices
D. Free hydrochloric acid in gastric juices
E. Lactic acid in gastric juices
815.
Active trypsin formation by the action of enteropeptidase can be viewed as the
master activation step because
A. It occurs first
B. * It can activate its own zymogen
C. Trypsin activates other pancreatic zymogens
D. a and b
E. a, b and c
816.
After digestion amino acids
A. * Are absorbed into portal circulation
B. Are absorbed into lymph
C. Are excreted to the extent of 50%
D. Converted into glucose in the intestine
E. Both A and B
817.
After digestion amino acids
A. * Are absorbed into portal circulation
B. Are absorbed into lymph
C. Are excreted to the extent of 50%
D. Converted into glucose in the intestine
E. All of the above
818.
Alanine aminotransferase (ALT) transfers an amino group from alanine to:
A. Pyruvate
B. * alpha-ketoglutarate
C. Oxaloacetate
D. Methionine
E. Carbamoyl phosphate
819.
Alanine can be synthesized from
A. Glutamate and ?-ketoglutarate
B. * Pyruvate and glutamate
C. Pyruvate and ? -ketoglutarate
D. Asparate and ? –ketoglutarate
E. All of these
820.
All of the following are required for synthesis of alanine except
A. Pyruvate
B. * ? -ketoglutarate
C. Glutamate
D. Pyridoxal phosphate
E. Both a and C
821.
All of the following statements about aspartate are true except
A.
B.
C.
D.
E.
822.
A.
B.
C.
D.
E.
823.
A.
B.
C.
D.
E.
824.
A.
B.
C.
D.
E.
825.
A.
B.
C.
D.
E.
826.
A.
B.
C.
D.
E.
827.
A.
B.
C.
D.
E.
828.
A.
B.
C.
D.
E.
829.
A.
B.
C.
D.
It is non-essential amino acid
It is a dicarboxylic amino acid
* It can be synthesized from pyruvate and glutamate
It can be converted into asparagine
Both A and B
All the following are branched chain amino acids except
Isoleucine
* Alanine
Leucine
Valine
Both A and B
All the following are true about phenylketonuria except
Deficiency of phenylalanine hydroxylase
Mental retardation
* Increased urinary excretion of p-hydroxyphenyl pyruvic acid
Decrease serotonin formation
C and D
?All the following statements about pepsin are correct except
It is smaller than pepsinogen
It is formed by the action of HCl on its precursor
Its optimum pH is 1.0–2.0
* It hydrolyses the C-terminal and N-terminal peptide bonds of proteins
C and D
All the following statements about pepsin are correct except
It is smaller than pepsinogen
It is formed by the action of HCl on its precursor
Its optimum pH is 1.0–2.0
* It hydrolyses the C-terminal and N-terminal peptide bonds of proteins
Both A and D
Allosteric inhibitor of glutamate dehydrogenase is
* ATP
ADP
AMP
GMP
None of these
Allsoteric activator of glutamate dehydrogenase is
ATP
GTP
* ADP and GDP
AMP and GMP
AMP
Amino acids from N-end in the proteins are splittted off by
Dipeptidase
Carbooxypeptidase
* Aminopeptidase
Elastase
Endopeptidase
Amino acids provide the nitrogen for the synthesis of
* The bases of the phospholipids
Uric acid
Glycolipids
Chondroitin sulphates
830.
831.
832.
833.
834.
835.
836.
837.
838.
E. All of these
Amino acids provide the nitrogen for the synthesis of
A. * The bases of the phospholipids
B. Uric acid
C. Glycolipids
D. Chondroitin sulphates
E. None of these
Ammonia intoxication symptoms occur when brain ammonia levels are
A. Slightly diminished
B. Highly diminished
C. * Increased
D. Normal
E. All of these
Ammonia is transported from muscles to liver mainly in the form of
A. Free ammonia
B. Glutamine
C. Asparagine
D. * Alanine
E. Both B and D
An organ which is extremely sensitive to ammonia toxicity is
A. Liver
B. * Brain
C. Kidney
D. Heart
E. Both A and B
As a result of amino acids decarboxylation in the organism are formed:
A. Ammonia, urea, creatine
B. * Amines, diamines
C. Polypeptides, uric acid
D. Dipeptides, xanthine
E. Amines, indican
As a result of amino acids decarboxylation in the organism are formed:
A. Ammonia, urea, creatine
B. * Amines, diamines
C. Polypeptides, uric acid
D. Dipeptides, xanthine
E. Allantion, indican
Aspartate amino transferase uses the following for transamination:
A. Glutamic acid and pyruvic acid
B. * Glutamic acid and oxaloacetic acid
C. Aspartic acid and pyruvic acid
D. aspartic acid and keto adipic acid
E. None of these
Bioactive amines are formed in reaction of:
A. Transamination.
B. * Decarboxylisation.
C. Oxidation.
D. Deamination.
E. All of these
Biological value of a protein is
A. The percentage of ingested protein/nitrogen absorbed into circulation
B. * The percentage of ingested protein/nitrogen in the body
C. The percentage of ingested protein utilized for protein synthesis in the body
D. The gain in body weight (gm) per gm of protein ingested
E. Both A and B
839.
By overheating the nutritional value of cereal proteins is
A. Increased
B. * Decreased
C. Unchanged
D. None of these
E. D and C
840.
Carboxy peptidase B in the small intestine hydrolyzes peptides containing
A. Leucine
B. Isoleucine
C. * Arginine
D. Cysteine
841.
Carboxy peptidase B in the small intestine hydrolyzes peptides containing
A. Leucine
B. Isoleucine
C. * Arginine
D. Cysteine
E. C and D
842.
Carboxypeptidase, an enzyme of pancreatic juice, contains
A. Mn
B. * Zinc
C. Magnesium
D. Manganese
E. Iron
843.
Carboxypeptidase, an enzyme of pancreatic juice, contains
A. Mn
B. * Zinc
C. Magnesium
D. Manganese
E. All of the above
844.
Choose from the below mentioned substances the product, which concentration is
increased during putrefying of proteins:
A. * Indole, phenol
B. Creatinine, creatine
C. Cyanocobalamin, naphthoquinone
D. Thiamine, biotin
E. Tryptophane, arginine
845.
Chymotrypsin in the small intestine hydrolyzes peptide linkages containing
A. Alanine
B. * Pheynl alanine
C. Valine
D. Methionine
E. Both A and B
846.
Chymotrypsinogen is transformed to chymotrypsin in the effect of:
A. Renin, gastrixin
B. Elastase, enterokinase
C. Hydrochloric acid, pepsin
D. * Trypsin, chymotrypsin
E. Pepsin, enterokinase
847.
Clinical features of Kwashiorkor include all of the following except
A.
B.
C.
D.
E.
* Mental retardation
Muscle wasting
Oedema
Anaemia
Both A and D
848.
Conversion of tyrosine to dihydroxyphenylalanine is catalysed by tyrosine
hydroxylase which requires
A. NAD
B. FAD
C. ATP
D. * Tetrahydrobiopterin
E. PLP
849.
Daily excretion of nitrogen by an adult man is about
A. 15–20 mg
B. 1.5–2 gm
C. * 5–10 gm
D. 15–20 gm
E. 2-3 mg
850.
Deamination is ______ of amino group.
A. * Removal
B. Addition
C. Supplementation
D. None of these
E. Both B and C
851.
Deamination is ______ of amino group.
A. * Removal
B. Addition
C. Supplementation
D. Transfer
E. None of these
852.
DOPA is an intermediate in the synthesis of
A. Thyroid hormones
B. Catecholamines
C. Melanin
D. * Catecholamines and melanin
E. All of these
853.
During denaturation of proteins, all of the following are disrupted except
A. * Primary structure
B. Secondary structure
C. Tertiary structure
D. Quaternary structure
E. Both A and C
854.
Enzyme catalyzed hydrolysis of proteins produces amino acids of the form:
A. D
B. * L
C. DL
D. G
E. All of these
855.
Enzyme catalyzed hydrolysis of proteins produces amino acids of the form:
A. D
B. * L
C. DL
D. All of these
E. None of these
856.
From dietary protein as well as from the urea present in fluids secreted into the
gastrointestinal tract intestinal bacteria produce
A. Carbondioxide
B. * Ammonia
C. Ammonium sulphate
D. Creatine
E. A and D
857.
From dietary protein as well as from the urea present in fluids secreted into the
gastrointestinal tract intestinal bacteria produce
A. Carbondioxide
B. * Ammonia
C. Ammonium sulphate
D. Creatine
858.
From two amino acids peptide bond formation involves removal of one molecule
of
A. * Water
B. Ammonia
C. Carbon dioxide
D. Carboxylic acid
E. Both A and B
859.
GABA(gama amino butyric acid) is
A. Post-synaptic excitatory transmitter
B. * Post-synaptic inhibitor transmitter
C. activator of glia-cell function
D. inhibitor of glia-cell function
E. Both B and C
860.
Gastrin stimulates
A. Gastric motility
B. Gastric secretion
C. * Both (A) and (B)
D. None of these
E. Increase of gastric juice pH
861.
Histamine is formed from histidine by
A. Deamination
B. Dehydrogenation
C. * Decarboxylation
D. Carboxylation
E. Oxidation
862.
Histidine is converted to histamine through the process of
A. Transamination
B. * Decarboxylation
C. Oxidative deamination
D. Urea cycle
E. TCA
863.
If one amino acid is fed excess, the absorption of another is
A. Slightly accelerated
B. Moderately accelerated
C. Highly accelerated
D. * Retarded
E. Both C and D
864.
If one amino acid is fed excess, the absorption of another is
A. Slightly accelerated
B. Moderately accelerated
C. Highly accelerated
D. * Retarded
E. None of the above
865.
If the amino group and a carboxylic group of the amino acid are attached to same
carbon atom, the amino acid is called as
A. * Alpha
B. Beta
C. Gamma
D. Epsilon
E. Delta
866.
In a person increase in weight in gms per gm of protein consumption represents
A. * Protein efficiency ratio
B. Digestibility value of proteins
C. Biological value of proteins
D. Net protein utilisation
E. All of these
867.
In carcinoid syndrome the argentaffin tissue of the abdominal cavity overproduce
A. * Serotonin
B. Histamine
C. Tryptamine
D. Tyrosine
E. All of these
868.
In case of severe denaturation of protein, there is
A. Reversible denaturation
B. Moderate reversible denaturation
C. * Irreversible denaturation
D. B and C
E. None of these
869.
In glycoproteins the carbohydrate is in the form of disaccharide units, the number
of units are
A. 50–100
B. 200–300
C. 400–500
D. * 600–700
E. None of these
870.
In human and other ureotelic organisms, the end product of amino acid nitrogen
metabolism:
A. Bile acids
B. Ketone bodies
C. * Urea
D. Barium sulphate
E. Ammonia
871.
In humans, NH3 is detoxified in liver as
A. Creatinine
B. Uric acid
C. * Urea
D. Uronic acid
E. All of these
872.
In quaternary structure, subunits are linked by
A.
B.
C.
D.
E.
873.
A.
B.
C.
D.
E.
874.
A.
B.
C.
D.
E.
875.
A.
B.
C.
D.
E.
Peptide bonds
Disulphide bonds
Covalent bonds
* Non-covalent bonds
Both A and B
In small intestine trypsin hydrolyzes peptide linkages containing
* Arginine
Histidine
Serine
Aspartate
All of these
In the stomach act such proteolytic enzymes:
Trypsin, chymotrypsin
Pepsin, enterokinase
Trypsin, renin
* Pepsin, gastrixin, rennin
Chymotrypsin, enterokinase
In which part of polypeptide chain aminopeptidases break down peptides?
From the end of free carboxyl group
* From the end of free amino group
From the end of free imino group
Bonds between aminodicarbonic acids
Bonds between cyclic amino acids
Inactive zymogens are precursors of all the following gastrointestinal enzymes
876.
except
A. Carboxypeptidase
B. Pepsin
C. * Amino peptidase
D. Chymotrypsin
E. Both B and D
877.
Inactive zymogens are precursors of all the following gastrointestinal enzymes
except
A. Carboxypeptidase
B. Pepsin
C. * Amino peptidase
D. Chymotrypsin
E. A and C
878.
Insufficiency of which vitamin does cause the inhibition of activity of
aminotransferase’s and decarboxylase?
A. B3
B. B2
C. B6
D. B12
E. B15
879.
Kwashiorkor occurs when the diet is severely deficient in
A. Iron
B. Calories
C. * Proteins
D. Essential fatty acids
E. Both B and C
880.
Kwashiorkor usually occurs in
A. * The post-weaning period
B.
C.
D.
E.
Pregnancy
Lactation
Old age
Both B and C
881.
Maple syrup urine diseases is an inborn error of metabolism of
A. Sulphur-containing amino acids
B. Aromatic amino acids
C. * Branched chain amino acids
D. Dicarboxylic amino acids
E. All of these
882.
Marasmus differs from Kwashiorkor in the which of these following respect
A. Mental retardation occurs in kwashiorkor but not in marasmus
B. Growth is retarded in kwashiorkor but not in marasmus
C. Muscle wasting occurs in marasmus but not kwashiorkor
D. * Subcutaneous fat disappears in marasmus but not in kwashiorkor
E. All of these
883.
Marasmus occurs from deficient intake of
A. Essential amino acids
B. Essential fatty acids
C. * Calories
D. Zinc
E. All of these
884.
More than half of the protein of the liver and intestinal mucosa are broken down
and resynthesised in
A. * 10 days
B. 12 days
C. 15 days
D. 18 days
E. All of the above
885.
Most amino acids are substrates for transamination except
A. Alanine
B. * Threonine
C. Serine
D. Valine
E. Glutamate
886.
Most of the ammonia released from L-?-amino acids reflects the coupled action of
transaminase and
A. * L-glutamate dehydrogenase
B. L-amino acid oxidase
C. Histidase
D. Serine dehydratase
E. All of these
887.
Naturally occurring amino acids have
A. * L-Configuration
B. D-Configuration
C. DL-Configuration
D. F-Configuration
E. None of these
888.
Negative nitrogenous balance is observed in all listed cases, except:
A. Kidney disease
B. * Domination of animal proteins in food
C. Domination of vegetable proteins in food
D. In old age
E. Pancreatitis
889.
Net protein utilization depends upon
A. Protein efficiency ratio
B. Digestibility coefficient
C. Digestibility coefficient and protein efficiency ratio
D. * Digestibility coefficient and biological value
E. Both A and D
890.
NH3 is detoxified in brain chiefly as
A. Urea
B. Uric acid
C. Creatinine
D. * Glutamine
E. Alanine
891.
Number of amino acids present in the dietary proteins:
A. 22
B. 23
C. * 20
D. 19
E. 12
892.
Oxaloacetate is converted to aspartic acid by
A. Reductase
B. Oxidase
C. * Transminase
D. Catalase
E. Decarboxylase
893.
Oxidative conversion of many amino acids to their corresponding –ketoacids
occurs in mammalian:
A. * Liver and kidney
B. Adipose tissue
C. Pancreas
D. Intestine
E. All of these
894.
Pancreatic juice contains all of the following except
A. Trypsinogen
B. Lipase
C. * Cholecystokinin
D. Chymnotrypsinogen
E. Both C and D
895.
Pancreatic juice contains the precursors of all of the following except
A. Trypsin
B. Chymotrypsin
C. Carboxypeptidase
D. * Aminopeptidase
E. Both c and D
896.
Pancreatic juice contains the precursors of all of the following except
A. Trypsin
B. Chymotrypsin
C. Carboxypeptidase
D. * Aminopeptidase
E. C and D
897.
Patient has hyperplasia of G-cells of antral part of stomach. What changes in
gastric juice are most possible for this pathology?
A. * Hyperchlorhydria
B. Hypochlorhydria
C. Achlorhydria
D. Achylia
E. Presence of lactic acid
898.
Patient has positive nitrogenous balance. The reason of this can be:
A. Starvation
B. Tuberculosis
C. * Pregnancy
D. Cancer
E. AIDS
899.
Patient with enterocolitis has affected protein metabolism. What mechanism of
amino acids absorption is affected?
A. Pinocytosis
B. Primary active transport
C. Simple diffusion
D. * Secondary sodium-dependent transport
E. Phagocytosis
900.
Pentagastrin is a
A. Naturally occurring form of gastrin
B. Inactive metabolite of gastrin
C. Active metabolite of gastrin
D. * Synthetic form of gastrin
E. Both A and B
901.
Physiologically active configuration of amino acids:
A. * L
B. D
C. A
D. For some amino acids it is either of two
E. Neither L nor D
902.
Plasma proteins are isolated by
A. Salting out
B. Electrophoresis
C. Flourimetry
D. * Both (A) and (B)
E. All of the above
903.
Progressive transmethylation of ethanolamine gives
A. Creatinine
B. Choline
C. * Methionine
D. N-methyl nicotinamide
E. Adenine
904.
Protein anabolism is stimulated by
A. ACTH
B. * Testosterone
C. Glucagon
D. Epinephrine
E. All of these
905.
Protein anabolism is stimulated by
A. ACTH
B.
C.
D.
E.
906.
A.
B.
C.
D.
E.
907.
A.
B.
C.
D.
E.
908.
A.
B.
C.
D.
E.
909.
A.
B.
C.
D.
E.
910.
A.
B.
C.
D.
E.
911.
A.
B.
C.
D.
E.
912.
A.
B.
C.
D.
E.
913.
A.
B.
C.
D.
E.
* Testosterone
Glucagon
Epinephrine
Both B and C
Protein catabolism is stimulated by all below mentioned exept
ACTH
* Testosterone
Glucagon
Epinephrine
A and D
Proteins produce polypeptides from proteins by
Oxidizing
Reducing
* Hydrolyzing
All of these
None of these
Proteins produce polypeptides from proteins by
Oxidizing
Reducing
* Hydrolyzing
None of these
B and C
Proteins react with biuret reagent which is suggestive of 2 or more
Hydrogen bonds
* Peptide bonds
Disulphide bonds
Hydrophobic bonds
Both B and C
Pyridoxal phosphate is involved in which type of reaction?
oxidation of pyruvate
* production of new amino acids by transamination
phosphate-transfer to produce ATP from ADP
the regeneration of methionine from homocysteine
A and B
Pyridoxal phosphate is the active coenzyme form of vitamin:
B1.
B2.
B3.
* B6.
B12
Pyruvic acid can be obtained by transamination of alanine with
* ?- keto glutaric acid
Acetoacetic acid
OH butyric acid
Phosphoenol Pyruvic acid
Both A and C
Rennin acts on casein of milk in infants in presence of
Mg++
Zn++
Co++
* Ca++
Both C and D
914.
Rennin acts on casein of milk in infants in presence of
A. Mg++
B. Zn++
C. Co++
D. * Ca++
E. B and D
915.
Secretion of gastrin is evoked by
A. Entry of food into stomach
B. Vagal stimulation
C. Lower aliphatic alcohols
D. * All of these
E. None of these
916.
Sulphur containing amino acids after catabolism produces a substance which is
excreted:
A. SO2
B. HNO3
C. * H2SO4
D. H3PO4
E. All of these
917.
Sulphur-containing amino acid is
A. Glutathione
B. Chondroitin sulphate
C. * Homocysteine
D. Tryptophan
E. Alanine
918.
The acceptor of amino group in the processes of transamination is:
A. Pyruvate
B. Oxaloacetate
C. * CC-ketoglutarate
D. Fumarate
E. Dioxyacetonephosphate
919.
The acceptor of amino group in the processes of transamination is:
A. Pyruvate
B. Oxaloacetate
C. * C.? ?-ketoglutarate
D. Fumarate
E. Dioxyacetonephosphate
920.
The activator of trypsinogen is:
A. Hydrochloric acid
B. Chymotrypsin
C. * Enterokinase
D. Aminopeptidase
E. Carboxypeptidase
921.
The activity of mammalian L-amino acid oxidase, an FMN – flavo protein, is
quite
A. * Slow
B. Rapid
C. Both (A) and (B)
D. Sensitive
E. None of these
922.
The amino acid that undergoes oxidative deamination at significant rate is
A. Alanine
B.
C.
D.
E.
Aspartate
* Glutamate
Glutamine
All of these
923.
The amino acid which synthesizes many hormornes:
A. Valine
B. * Phenylalanine
C. Alanine
D. Histidine
E. Both B and C
924.
The amino acids abstracted from the liver are not utilized for repair or special
synthesis but are broken down to
A. Keto acids
B. Sulphur dioxide
C. Water
D. * Ammonia
E. Uric acid
925.
The amount of total acidity in gastric juice of a patient P. is normal. Which of the
listed numbers is true in this case?
A. A.30-40 mM/l
B. * B.40-60 mM/l
C. 20-30 mM/l
D. 30-50 mM/l
E. 20-40 mM/l
926.
The basic amino acids are
A. * Lysine
B. Bile acids
C. Glycine
D. Alanine
E. All of these
927.
The building up and breaking down of protoplasm are concerned with the
metabolism of
A. Carbohydrate
B. Lipid
C. * Protein
D. Minerals
E. C and D
928.
The chemical score of different proteins is calculated in terms of
A. * Egg proteins
B. Milk proteins
C. Fish proteins
D. Wheat proteins
E. All of these
929.
The end product of amino acid nitrogen metabolism in uricotelic organisms
(reptiles and birds) is
A. Bilirubin
B. Urea
C. * Uric acid
D. Biliverdin
E. Ammonia
930.
The essential amino acids
931.
932.
933.
934.
935.
936.
937.
938.
A. Must be supplied in the diet because the organism has lost the capacity to aminate
the corresponding ketoacids
B. * Must be supplied in the diet because the human has an impaired ability to
synthesize the carbon chain of the corresponding ketoacids
C. Are identical in all species studied
D. Are defined as those amino acids which cannot be synthesized by the organism at
a rate adequate to meet metabolic requirements
E. Both B and D
The example of phosphoprotein:
A. Mucin
B. * Ovovitellin
C. Ovomucoid
D. Tendomucoid
E. All of the above
The gain in body weight (gm) per gm of protein ingested is known as
A. Net protein utilisation
B. * Protein efficiency ratio
C. Digestibility coefficient
D. Biological value of protein
E. Both A and B
The half-life of antibody protein is about
A. 4 weeks
B. 3 weeks
C. * 2 weeks
D. 1 week
E. None of the above
The main enzymatic reaction requiring pyridoxal phosphate as a coenzyme is :
A. Decarboxilation
B. * Transamination
C. Deamination
D. Reamination
E. Oxidation
The main sites for oxidative deamination are
A. * Liver and kidney
B. Skin and pancreas
C. Intestine and mammary gland
D. Lung and spleen
E. All of these
The major constituent of the proteins of hair and keratin of skin:
A. Arginine
B. * Cysteine
C. Glycine
D. Arginine
E. None of these
The major site of urea synthesis is
A. Brain
B. Kidneys
C. * Liver
D. Muscles
E. Both B and C
The metabolism of protein is integrated with that of carbohydrate and fat through
A. * Oxaloacetate
B.
C.
D.
E.
939.
A.
B.
C.
D.
E.
940.
A.
B.
C.
D.
E.
941.
A.
B.
C.
D.
E.
942.
A.
B.
C.
D.
E.
943.
A.
B.
C.
D.
E.
944.
acid:
A.
B.
C.
D.
E.
945.
A.
B.
C.
D.
E.
946.
A.
B.
C.
D.
Citrate
Isocitrate
Malate
A and B
The metabolism of protein is integrated with that of carbohydrate and fat through
* Oxaloacetate
Citrate
Isocitrate
Malate
Succinate
The milk protein in the stomach in an adult is digested by
* Pepsin
Rennin
HCl
Chymotrypsinogen
Both A and B
The milk protein in the stomach of the infants is digested by
Pepsin
Trypsin
Chymotrypsin
* Rennin
All of the above
The milk protein in the stomach of the infants is digested by
Pepsin
Trypsin
Chymotrypsin
* Rennin
A and D
The milk protein in the stomach of theinfants is digested by
Pepsin
Trypsin
Chymotrypsin
* Rennin
Both A and D
The negative nitric balance can be caused by the deficit of the following amino
Serine
Tyrosine
* Leucine
Glycine
Alanine
The neutral amino acids for absorption need
TPP
* B6 – PO4
NAD+
NADP+
FAD
The neutral amino acids for absorption need
TPP
* B6 – PO4
NAD+
NADP+
947.
948.
949.
950.
as
E. FAD
The only correct statement about chymotrypsin is
A. It is formed from trypsin
B. Carboxypeptidase converts trypsin into chymotrypsin
C. * Its optimum pH is around 7
D. It hydrolyses peptide bonds involving basic amino acids
E. Its optimum pH is around 4
The only correct statement about chymotrypsin is
A. It is formed from trypsin
B. Carboxypeptidase converts trypsin into chymotrypsin
C. * Its optimum pH is around 7
D. It hydrolyses peptide bonds involving basic amino acids
E. None of the above
The percentage of food nitrogen that is retained in the body represents
A. Digestibility coefficient
B. Biological value of proteins
C. Protein efficiency ratio
D. * Net protein utilisation
E. All of these
The percentage of ingested protein/nitrogen absorbed into blood stream is known
A.
B.
C.
D.
E.
951.
A.
B.
C.
D.
E.
952.
A.
B.
C.
D.
E.
953.
A.
B.
C.
D.
E.
954.
A.
B.
C.
D.
E.
955.
A.
Net protein utilisation
Protein efficiency ratio
* Digestibility coefficient
Biological value of protein
All of these
The percentage of nitrogen retained in the body after absorption of diet represents
Digestibility coefficient of proteins
* Biological value of proteins
Protein efficiency ratio
Net protein utilisation
All of these
The pH of gastric juice become low in
Hemolytic anemia
* Pernicious anemia
Both (A) and (B)
Jaundice
None of these
The pH of gastric juice become low in
Hemolytic anemia
* Pernicious anemia
Both (A) and (B)
Gastritis
None of these
The recommended daily allowance (RDA) of proteins for an adult man is
* 70 gms
50 gms
40 gms
30 gms
200 gms
The symptom of ammonia intoxication includes
* Blurring of vision
B.
C.
D.
E.
Constipation
Mental confusion
Diarrhoea
All of these
956.
The symptom of ammonia intoxication includes
A. * Blurring of vision
B. Constipation
C. Mental confusion
D. Diarrhea
E. All of these
957.
The third active process for amino acids transport involves
A. Acidic amino acids
B. Basic amino acids
C. * Neutral amino acids
D. Sulphur containing amino acids
E. All of these
958.
The third active process for amino acids transport involves
A. Acidic amino acids
B. Basic amino acids
C. * Neutral amino acids
D. Sulphur containing amino acids
E. C and D
959.
The transaminase activity needs the coenzyme:
A. ATP
B. * B6 – PO4
C. FAD+
D. NAD+
E. TPP
960.
The transport of amino acids regulated by active processes of different numbers:
A. 1
B. 2
C. * 3
D. 4
E. 10
961.
The transport of amino acids regulated by active processes of different numbers:
A. 1
B. 2
C. * 3
D. 4
E. None of the above
962.
The unwanted amino acids abstracted from the tissues are either used up by the
tissue or in the liver converted into
A. Ammonia
B. * Urea
C. Ammonium salts
D. Uric acid
E. All of these
963.
The useful reagent for detection of amino acids is
A. Molisch reagent
B. Dichlorophenol Indophenol
C. * Ninhydrin
D. Biuret
964.
by
E. All of these
The zymogen from trypsinogen of pancreatic juice is converted to active trypsin
A.
B.
C.
D.
E.
965.
by
A.
B.
C.
D.
E.
Peisin
Enterocrinin
* Enterokinase
Rennin
HCl
The zymogen from trypsinogen of pancreatic juice is converted to active trypsin
Peisin
Enterocrinin
* Enterokinase
Rennin
B and C
966.
The ?-ketoacid is decarboxylated by H2O2 forming a carboxylic acid with one
carbon atom less in the absence of the enzyme:
A. * Catalase
B. Decarboxylase
C. Deaminase
D. Phosphatase
E. Both A and B
967.
To endopeptidases belong all below-mentioned enzymes, except:
A. Pepsin
B. Elastase
C. * Carboxypeptidase
D. Chemotrypsin
E. Trypsin
968.
Transamination is a
A. Irreversible process
B. * Reversible process
C. Both (A) and (B)
D. Removal of CO2
E. None of these
969.
Transcortins are
A. * Mucoproteins
B. Glycoproteins
C. Metalloproteins
D. Lipoproteins
E. None of the above
970.
Transmethylation of guanido acetic acid gives
A. * Creatine phosphate
B. Creatinine
C. Choline
D. n-methyl nicotinamide
E. All of these
971.
Transmethylation of guanido acetic acid gives
A. * Creatine phosphate
B. Creatinine
C. Choline
D. n-methyl nicotinamide
E. None of these
972.
A.
B.
C.
D.
E.
973.
is
A.
B.
C.
D.
E.
Tryptophan could be considered as precursor of
* Melanotonin
Thyroid hormones
Melanin
Epinephrine
All of these
Vitamin required for the conversion of hydroxyphenylpyruvate to homogentisate
Folacin
Cobalamin
Ascorbic acid
* Niacin
Cholecalciferol
974.
What biologically active substance is formed in the process of decarboxylation of
5-hydroxytryptophane?
A. Corticosterone
B. Thyroxine
C. * Serotonine
D. Histamine
E. Anserine
975.
What chemical compound is formed in reaction of amino acids transamination?
A. Ammonia.
B. H2O.
C. * ?-Keto-acid.
D. Pyruvate.
E. None of these
976.
What contents of free hydrochloric acid is in the gastric juice?
A. 10-20 mmol/l
B. * 20-40 mmol/l
C. 40-60 mmol/l
D. 60-80 mmol/l
E. 80-100 mmol/l
977.
What disease is proved by increase of AsAT in blood?
A. Parotiditis
B. Gastritis
C. Pancreatitis
D. * Cardiac infarction
E. Pneumonia
978.
What is the name of the cofactor for AST, ALT, or any other transamination
reaction?
A. NAD+
B. Acetyl CoA
C. * Pyridoxal phosphate (PLP)
D. FADH2
E. NADPH
979.
When egg albumin is heated till it is coagulated, the secondary and tertiary
structures of the proteins are completely lost resulting in a mixture of randomly arranged
A. Dipeptide chains
B. Tripeptide chains
C. * Polypeptide chains
D. All of these
E. None of these
980.
A.
B.
C.
D.
E.
981.
A.
B.
C.
D.
E.
982.
A.
B.
C.
D.
E.
983.
A.
B.
C.
D.
E.
984.
A.
B.
C.
D.
E.
985.
A.
B.
C.
D.
E.
986.
A.
B.
C.
D.
E.
987.
A.
B.
C.
D.
E.
988.
A.
B.
C.
Which amino acid is a lipotropic factor?
Lysine
Leucine
Tryptophan
* Methionine
All of these
Which amino acid is a lipotropic factor?
Lysine
Lecuine
Tryptophan
* Methionine
A and D
Which among the following is a nutritionally essential amino acid for man ?
Alanine
Glycine
Tyrosine
* Tryptophan
Both A and D
Which among the following is an essential amino acid?
Cysteine
* Leucine
Tyrosine
Aspartic acid
Both B and D
Which compounds are formed during the oxidative deamination of amino acids?
* Keto acid and ammonia.
Saturated fatty acid and ammonia.
Unsaturated fatty acid and ammonia.
Keto acid and H2O.
A and D.
Which enzyme activates the trypsinogen?
Chymotrypsin
Carboxypeptidase
* Enterokinase
Trypsin
Aminopeptidase
Which enzyme cleaves proteins in the stomach?
Trypsin
Chymotrypsin
Carboxypeptidase
Enterokinase
* Pepsin
Which enzyme takes part in oxidative deamination of amino acids?
Pyridine-linked oxidase.
* Pyridine-linked dehydrogenase.
Pyridine-linked carboxypeptidase
Pyridine-linked transferase.
B and D
Which of the amino acid produces a vasodilator on decarboxylation?
Glutamin acid
* Histidine
Ornithine
D. Cysteine
E. C and D
989.
Which of the amino acid produces a vasodilator on decarboxylation?
A. Glutamic acid
B. * Histidine
C. Ornithine
D. Cysteine
E. Alanine
990.
Which of the following is an amino acid that is found in proteins?
A. Adenosine
B. Adenine
C. * Alanine
D. Linoleic acid
E. Creatine
991.
Which of the following is an essential amino acid?
A. Glutamine
B. Proline
C. * Methionine
D. Cysteine
E. Asparagine
992.
Which of the following is an essential amino acid?
A. Glutamine
B. Proline
C. Methionine
D. Cysteine
E. Asparagine
993.
Which one of the following is an essential amino acid?
A. Arginine
B. Tyrosine
C. * Phenylalanine
D. Proline
E. B and C
994.
Which pathological component appears in the gastric juice in cancer of the
stomach?
A. * Lactic acid
B. Hydrochloric acid
C. Acetic acid
D. Sulfuric acid
E. Citric acid
995.
Which pathological component appears in the gastric juice in cancer of the
stomach?
A. * Lactic acid
B. Hydrochloric acid
C. Acetic acid
D. Sulfuric acid
E. Citric acid
996.
Which vitamins take part in deamination of amino acids?
A. B6
B. B3
C. * B5
D. B1
997.
With decarboxylation of histidine is formed:
A.
B.
C.
D.
E.
Tyramine
* Histamine
Putrescine
Cadaverine
Serotonine
998.
Zymogen is
A. An intracellular enzyme
B. Serum enzyme
C. A complete extracellular enzyme
D. * An inactivated enzyme
999.
All of t_________ provides one of the N atoms in urea synthesized by the urea
cycle, while ___________ is a product of the urea cycle.
A. NH3 ; aspartate
B. NH3 ; carbamate
C. * Aspartate ; fumarate
D. Fumarate ; aspartate
E. Carbamate ; fumarate
1000.
________ contain(s) nitrogen
A. * Amino acids
B. Nucleotides
C. Creatine phosphate
D. All of the above
E. None of the above
1001.
A coenzyme required for the synthesis of glycine from serine is
A. ATP
B. Pyridoxal phosphate
C. * Tetrahydrofolate
D. NAD
E. FAD
1002.
A compound serving a link between citric acid cycle and urea cycle is
A. Malate
B. Citrate
C. Succinate
D. * Fumarate
E. Ornithine
1003.
A dietary deficiency of tryptophan and nicotinate leads to
A. Beri Beri
B. Xerophthalmia
C. Anemia
D. * Pellegra
E. Rickets
1004.
A phosphoprotein present in egg is
A. Casein
B. Albumin
C. Ovoglobulin
D. Ovovitellin
E. Ihtulin
1005.
A positive nitrogen balance occurs
A. * In growing infant
B. Following surgery
C. In advanced cancer
D. In kwashiorkar
1006.
1007.
1008.
1009.
1010.
1011.
1012.
1013.
1014.
E. In old year
Activity of ceruloplasmin shown in vitro:
A. Reductase
B. * Hydrolase
C. Ligase
D. Oxidase
E. Transferases
Alanine can be synthesized from
A. Glutamate and alfa-ketoglutarate
B. * Pyruvate and glutamate
C. Pyruvate and alfa-ketoglutarate
D. Asparate and alfa-ketoglutarate
E. None of the above
All amino acids have one asymmetric carbon atom, except
A. Arginine
B. Aspargine
C. * Histidine
D. Glycine
E. Valine
All followings are ketogenic aminoacids, except
A. Leucine
B. Isoleucine
C. * Phenyl alanine
D. Glycine
E. Lysine
All of the following are required for synthesis of alanine except
A. Pyruvate
B. Alfa-ketoglutarate
C. Glutamate
D. Pyridoxal phosphate
E. * None of the above
All of the following are required for synthesis of glutamine except
A. Glutamate
B. Ammonia
C. * Pyridoxal phosphate
D. ATP
E. None of the above
All of the following statements about aspartate are true except
A. It is non-essential amino acid
B. It is a dicarboxylic amino acid
C. It can be synthesized from pyruvate and glutamate
D. * It can be converted into asparagine
E. None of the above
All the following statements about albinism are correct, except
A. Tyrosine hydroxylase (tyrosinase) is absent or deficient in melanocytes
B. Skin is hypopigmented
C. * It results in mental retardation
D. Eyes are hypopigmented
E. A, B, D are correct
All the following statements about pepsin are correct, except
A. It is smaller than pepsinogen
B. It is formed by the action of HCl on its precursor
1015.
1016.
1017.
1018.
1019.
1020.
1021.
1022.
C. Its optimum pH is 1.0–2.0
D. * It hydrolyses the C-terminal and N-terminal peptide bonds of proteins
E. Its optimum pH is 10–12
All the following statements about phenylketonuria are correct, except
A. * Phenylalanine cannot be converted into tyrosine
B. Urinary excretion of phenylpyruvate and phenyllactate is increased
C. It can be controlled by giving a lowphenylalanine diet
D. It leads to decreased synthesis of thyroid hormones, catecholamines and melanin
E. It leads to increased synthesis of insulin, catecholamines and homogentosonic
acid
Allosteric inhibitor of glutamate dehydrogenase is
A. ATP
B. ADP
C. AMP
D. * GDP
E. AMP
Allsoteric activator of glutamate dehydrogenase is
A. * ADP
B. GTP
C. ADP and GDP
D. AMP and GMP
E. ATP
Along with CO2, NH3 and ATP the aminoacid that is needed in urea cycle is
A. Alanine
B. Isoleucine
C. * Aspartate
D. Glycine
E. Valine
Amino acids excepting histidine are not good buffering agents in cell because
A. They exist as zwitter ions
B. * Their pk and not in the physiological pH of a cell
C. Only Histidine has pk of its R group at 6.0 unlike the others which have at a
different pH
D. None of these
E. All of the above
Amino acids, monosaccharrides, glycerol and fatty acids are:
A. Transported in the lymph to all the lymph nodes in the body
B. All hormones
C. Products of carbohydrate metabolism
D. Enzymes
E. * End products of digestion
Ammonia is transported from muscles to liver mainly in the form of
A. Free ammonia
B. Glutamine
C. Asparagine
D. * Alanine
E. ?-ketoglutarate
An amino acid not containing the usual— COOH group is
A. Alanine
B. Tryptophan
C. Methionine
D. * Taurine
E. Serine
1023.
An amino acid not involved in urea cycle is
A. Arginine
B. * Histidine
C. Ornithine
D. Citrulline
E. Only C and D
1024.
An amino acid which contains a disulphide bond is
A. Lysine
B. Methionine
C. Homocysteine
D. * Cystine
E. None of the above
1025.
An exopeptidase is
A. Trypsin
B. Chymotrypsin
C. Elastase
D. * Peptidase
E. Phospholipase
1026.
An important reaction for the synthesis of amino acid from carbohydrate
intermediates is transamination which requires the cofactor:
A. Thiamin
B. Riboflavin
C. Niacin
D. * Pyridoxal phosphate
E. Pantotenic acid
1027.
An organ which is extremely sensitive to ammonia toxicity is
A. Liver
B. * Brain
C. Kidney
D. Heart
E. Lung
1028.
As a result of amino acids decarboxylation in the organism are formed:
A. Ammonia, urea, creatine
B. * Amines, diamines
C. Polypeptides, uric acid
D. Dipeptides, xanthine
E. Allantion, indican
1029.
Branched chain amino acids are
A. Cysteine and cystine
B. Tyrosine and Tryptophan
C. Glycine and Serine
D. * Valine, Leucine and Isoleucine
E. Serin ans glycine
1030.
By decarboxylation of histidine are formed:
A. Tyramine
B. * Histamine
C. Putrescine
D. Cadaverine
E. Serotonine
1031.
C Splitting polymers into absorbable monomers in the GI tract
A. * Transport of nutrient molecules from the intestine to cells
B. Intracellular change of nutrient molecules into essential intracellular nutrients
C. Secretion of insulin and glucagon
D. Absorption of nutrient molecules into the body
1032.
Carbamoyl phosphate required for urea synthesis is formed in
A. Cytosol
B. * Mitochondria
C. Both (A) and (B)
D. None of these
E. Only A
1033.
Chymotrypsin is specific for peptide bonds containing
A. * Uncharged amino acid residues
B. Acidic amino acids
C. Basic amino acid
D. Small amino acid residues
E. None of the above
1034.
Clinical features of Kwashiorkor include all of the following except
A. Mental retardation
B. Muscle wasting
C. Oedema
D. * Anaemia
E. A and B
1035.
Coenzyme of amino acids decarboxylases is:
A. Thymidine diphosphate
B. Favin adenine dinucleotide
C. Favin mononucleotide
D. HS-CoA
E. * PALP
1036.
Combine the correct organism with the type of excretion of ammonia it uses.
Draw lines between.
A. Birds; Urea
B. * Humans; Uric acid
C. Fishes; Ammonia (ammonium ion)
D. Birds; Uric acid
E. Fishes; Uric acid
1037.
Consumption of raw eggs can cause deficiency of
A. * Calcium
B. Lipoic acid
C. Biotin
D. Vitamin A
E. Vitamin B2
1038.
Cysteine can be synthesized from methionine and
A. * Serine
B. Homoserine
C. Homocysteine
D. Threonine
E. Cystein
1039.
Cysteine has the formula: a. CH3SH; b. H2N—CH2—COOH c. HS—CH2—
CH(NH2)—COOH d. S—CH2—CH(NH2)—COOH S—CH2—CH(NH2)—COOH
A. d
B. c
C. * b
D. a
1040.
1041.
1042.
1043.
1044.
1045.
1046.
1047.
1048.
E. None of the above
Cystine is synthesized from
A. * Cysteine
B. Methionine
C. Arginine
D. Leucine
E. Valine
Cystinuria results from inability to
A. Metabolise cysteine
B. Convert cystine into cysteine
C. Incorporate cysteine into proteins
D. * Reabsorb cystine in renal tubules
E. Only A
Daily excretion of nitrogen by an adult man is about
A. 15–20 mg
B. 1.5–2 gm
C. 5–10 gm
D. 15–20 gm
E. * 3,5-4 gm
Denitrification involves reduction of nitrate, and the production of _______.
A. N2
B. NO
C. N2O
D. * All of the above
E. More than one of the above, but not all
Digestion of proteins:
A. Consumes water (hydrolysis) and amino acids
B. Consumes water (hydrolysis) and releases amino acids
C. * Releases water (dehydration) and amino acids
D. Releases water (dehydration) and consumes amino acids
E. None of the above
E None of the above
A. Metabolism of consumed foods and vitamins
B. Absorption of many of the digested nutrients
C. * Mixing of the gastric contents to form chyme
D. Secretion of alkaline enzymes
Egg is poor in
A. * Essential amino acids
B. Carbohydrates
C. Avidin
D. Biotin
E. None of the above
Egg is rich in all of the following except
A. Cholesterol
B. Saturated fatty acids
C. * Ascorbic acid
D. Calcium
E. Unsaturated fatty acids
Excess amino acids in the body are broken down to form urea in
A. Kidney
B. * Liver
C. Spleen
1049.
1050.
1051.
1052.
1053.
1054.
1055.
1056.
D. Pancreas
E. Stomach
For biosynthesis of proteins
A. * Amino acids only are required
B. Amino acids and nucleic acids only are required
C. Amino acid, nucleic acids and ATP only are required
D. Amino acids, nucleic acids, ATP, GTP, enzymes and activators are required
E. All are correct
For vegetarians, pulses are an important source of
A. Carbohydrates
B. * Proteins
C. Fat
D. Iron
E. All of the above
Free ammonia is released during
A. * Oxidative deamination of glutamate
B. Catabolism of purines
C. Catabolism of pyrimidines
D. All of these
E. Only C
Functions of the liver
A. Acts as a storehouse for certain vitamins and iron
B. Synthesizes fibrinogen and prothrombin for coagulation
C. * Synthesis of the serum albumin essential for blood colloid osmotic pressure and
fluid balance
D. Regulates blood levels of glucose, ketone bodies and fatty acids
E. Destroys bacteria that enter the blood from the intestine before reaching the
general circulation
Gastrin, which is secreted by stomach cells, functions to:
A. Inhibit the production of hydrochloric acid
B. Increase the secretions of the gastric glands
C. * Change pepsin into pepsinogen
D. Initiate the cephalic phase of digestion
E. Inhibit the secretory activities of the pancreas, liver and small intestine
Glycine can be synthesized from
A. * Serine
B. Choline
C. Betaine
D. All of these
E. Methionine
Glycine is not required for the formation of
A. Taurocholic acid
B. Creatine
C. Purines
D. * Pyrimidines
E. Glutation
Histamine is formed from histidine by
A. Deamination
B. Dehydrogenation
C. * Decarboxylation
D. Carboxylation
E. Oxidation
1057.
Histidine is converted to histamine through the process of
A. Transamination
B. * Decarboxylation
C. Oxidative deamination
D. Urea cycle
E. Krebs cycle
1058.
How many different amino acids are necessary for our protein production?
A. 15
B. 18
C. * 20
D. 25
E. 10
1059.
In carcinoid syndrome the argentaffin tissue of the abdominal cavity overproduce
A. Serotonin
B. Histamine
C. * Tryptamine
D. Tyrosine
E. None of the above
1060.
In humans, NH3 is detoxified in liver as
A. Creatinine
B. Uric acid
C. * Urea
D. Uronic acid
E. Amino acid
1061.
In the following reaction, Alanine acts as a H- H| | H- N– –COO—— H- N– –
COOH- C- C| |CH- CH
A. Acid
B. * Base
C. Zwitter ion
D. None of these
E. Only C are correct
1062.
In the mouth:
A. Food is propelled primarily by peristalsis
B. * The saliva is used to lubricate and dissolve food
C. The front teeth are called incisors
D. All of the above
E. B and C only
1063.
In the synthesis of 1 molecule of urea in the Kreb’s Hanseleit cycle, the number of
ATPs required is
A. 1
B. * 2
C. 3
D. 4
E. 5
1064.
In urea synthesis, the amino acid functioning solely as an enzyme activator:
A. * N-acetyl glutamate
B. Ornithine
C. Citrulline
D. Arginine
E. Serine
1065.
Increased serum alanine during fasting is due to
A. Breakdown of muscle proteins
B.
C.
D.
E.
* Decreased utilization of non essential amino acids
Leakage of aminoacids to plasma
Impaired renal function
Only C
1066.
Isoelectric pH of an amino acid is that pH at which it has a
A. Positive charge
B. Negative charge
C. * No charge
D. None of these
E. All of the above
1067.
Kwashiorkor occurs when the diet is severely deficient in
A. Iron
B. Calories
C. * Proteins
D. Essential fatty acids
E. Ca++
1068.
Maple syrup urine diseases is an inborn error of metabolism of
A. Sulphur-containing amino acids
B. Aromatic amino acids
C. * Branched chain amino acids
D. Dicarboxylic amino acids
E. None of the above
1069.
Mechanism by which NH3 is removed from the kidneys is
A. Urea formation
B. Uric acid formation
C. * Creatinine formation
D. None of these
E. All of the above
1070.
Methionine is synthesized in human body from
A. Cysteine and homoserine
B. Homocysteine and serine
C. * Cysteine and serine
D. None of these
E. Only A
1071.
Milk is a good source of
A. Proteins, calcium and iron
B. Proteins, calcium and ascorbic acid
C. Proteins, lactose and retinol
D. * Proteins, lactose and essential fatty acids
E. All of the above
1072.
Most of the ammonia released from L- amino acids reflects the coupled action of
transaminase and
A. * L-glutamate dehydrogenase
B. L-amino acid oxidase
C. Histidase
D. Serine dehydratase
E. Only D
1073.
Name the matter in the urine, which is a test of intensivity of protein putrefaction
processes in the intestine:
A. Urates
B. Urea
C. Creatinine
D. * Indican
E. Lactic acid
1074.
Natural L-isomers of amino acids are absorbed from intestine by
A. Passive diffusion
B. Simple diffusion
C. Faciliated diffusion
D. * Active process
E. None of the above
1075.
Naturally occurring amino acids have
A. L-Configuration
B. D-Configuration
C. * DL-Configuration
D. None of these
E. B-Configuration
1076.
NH3 is detoxified in brain chiefly as
A. * Urea
B. Uric acid
C. Creatinine
D. Glutamine
E. Cesteine
1077.
NH3 is removed from brain mainly by
A. Creatinine formation
B. Uric acid production
C. * Urea formation
D. Glutamine formation
E. None of the above
1078.
Non-protein amino acids are
A. * Ornithine
B. ?-alanine
C. ?-amino butyric acid
D. All of these
E. Serine
1079.
Number of amino acids present in the plant, animal and microbial proteins:
A. * 20
B. 80
C. 150
D. 200
E. 101
1080.
One of the following amino acid is solely ketogenic:
A. * Lysine
B. Alanine
C. Valine
D. Glutamate
E. Arginine
1081.
Only one type of digestive juice contains carbohydrate, protein and fat-digesting
enzymes. Which one is it?
A. * Pancreatic juice
B. Saliva
C. Bile
D. Intestinal juice
E. Gastric juice
1082.
Pancreatic juice contains the precursors of all of the following except
A.
B.
C.
D.
E.
* Trypsin
Chymotrypsin
Carboxypeptidase
Aminopeptidase
Elastase
1083.
Patient has positive nitric balance. The reason of this can be:
A. Starvation
B. Tuberculosis
C. * Pregnancy
D. Malignant neoplasms
E. AIDS
1084.
Patient on a vegetarian diet has negative nitric balance, hyperproteinemia,
affection of colloid-osmotic pressure and water-salt metabolism by normal function of
alimentary canal. This was caused by:
A. * Monotonous protein nutrition
B. Monotonous carbohydrate nutrition
C. Lack of unsaturated fatty acids
D. Lack of phospholipids in food
E. Lack of vitamins in food
1085.
Patient with enterocolitis has affected protein adoption. What mechanism of
amino acids absorption is affected?
A. Pinocytosis
B. Primary active transport
C. Simple diffusion
D. * Secondary sodium-dependent transport
E. Phagocytosis
1086.
Pepsin acts on denatured proteins to produce
A. Proteases and peptones
B. * Polypeptides
C. Peptides
D. Dipeptides
E. Glicerol
1087.
Pepsinogen is converted to active pepsin by
A. * HCl
B. Bile salts
C. Ca++
D. Enterokinase
E. Lipase
1088.
Physiologically active configuration of amino acids:
A. * L
B. D
C. For some amino acids it is either of two
D. Neither L nor D
E. C
1089.
Positive nitrogen balance is seen in
A. Starvation
B. Wasting diseases
C. * Growing age
D. Intestinal malabsorption
E. All of the above
1090.
Proteins that carries Iron into different tissues is
A. Ceruloplasmin
B.
C.
D.
E.
* Trans cortin
Mucoproteins
Glycoproteins
Lipoprotein
1091.
Pyridoxal phosphate, what is it?
A. It’s essential since our body can produce it itself
B. It’s essential since our body cannot produce it itself and it needs to be taken up
from the diet
C. It is a receptor molecule
D. * It is a very important co-enzyme in for instance transamination reactions
E. All of the above
1092.
Renin converts casein to paracasein in presence of
A. * Ca++
B. Mg++
C. Na+
D. K+
E. Fe++
1093.
Secretin produced by the mucosa of the duodenum is stimulated by:
A. Distention of the stomach
B. Large quantities of bile
C. A fatty chyme
D. * Acidity of the chyme
E. None of the above
1094.
Sulphur-containing amino acid is
A. Glutathione
B. Chondroitin sulphate
C. * Homocysteine
D. Tryptophan
E. Histidin
1095.
Sulphur-containing amino acid is
A. Glutathione
B. Chondroitin sulphate
C. * Homocysteine
D. Tryptophan
E. Histidin
1096.
The 2 nitrogen atoms in urea are contributed by
A. Ammonia and glutamate
B. Glutamine and glutamate
C. * Ammonia and aspartate
D. Ammonia and alanine
E. Ammonia and Glutamine
1097.
The absorption of intact protein from the gut in the foetal and newborn animals
takes place by
A. Pinocytosis
B. Passive diffusion
C. Simple diffusion
D. * Active transport
E. A and B
1098.
The acceptor of amino group in the processes of transamination is:
A. Pyruvate
B. Oxaloacetate
C. *
D. Fumarate
E. Dioxyacetonephosphate
1099.
The activators of trypsinogen are:
A. Hydrochloric acid
B. * Chymotrypsin
C. Enterokinase
D. Aminopeptidases
E. Carboxypeptidase
1100.
The amino acid containing an indole ring:
A. Tryptophan
B. Arginine
C. Threonine
D. * Phenylalanine
E. Valine
1101.
The amino acid that undergoes oxidative deamination at significant rate is
A. Alanine
B. Aspartate
C. * Glutamate
D. Glutamine
E. Serin
1102.
The amino acid which contains an indole group is
A. Histidine
B. Arginine
C. lycine
D. * Tryptophan
E. Cysteine
1103.
The amino acid which contains an indole group is
A. Histidine
B. Arginine
C. lycine
D. * Tryptophan
E. Cysteine
1104.
The amino acid which has a pK near 4 and thus is negatively charged at pH 7 is
A. * Alanine
B. Glutamic acid
C. Glutamine
D. Aspargine
E. None of the above
1105.
The amino acids in which the R groups have a net positive charge at pH 7.0 are
A. Lysine, Arginine, Histidine
B. Lysine, Aspargine
C. * Histidine, Aspargine
D. Glutamine, Arginine
E. Only D
1106.
The amount of urea in patient's urine is normal. Which of the listed figures are
true in this case?
A. 10-15 g
B. 20-40 g
C. 10-35 g
D. * 25-30 g
E. 50-60 g
1107.
The body protein after eighteen years
A.
B.
C.
D.
E.
Remains unchanged
Is decomposed only slightly at intervals of one month
Is in a constant state of flux
* Is used only for energy requirement
None of the above
1108.
The bond in proteins that is not hydrolysed under usual conditions of
denaturation:
A. Hydrophobic bond
B. Hydrogen bond
C. Disulphide bond
D. * Peptide bonds
E. Hydrophobic interaction
1109.
The catalytic triad of most serine proteases contains an Asp, His and Ser residue.
Which of the following is true?
A. The Asp residue is used in an acid catalyzed attack on the peptide bond.
B. The His residue is responsible for the substrate specificity of Trypsin.
C. * The Ser residue is used in a nucleophilic attack on the peptide bond.
D. The Asp residue is responsible for the substrate specificity of Trypsin.
E. None of the above.
1110.
The cleavage of trypsinogen to form active trypsin
A. Ocurs in the small intestine.
B. Can be catalyzed by trypsin.
C. Ocurs in the pancreas.
D. Can be catalyzed by chymotrypsin.
E. * The first two choices are both correct.
1111.
The cleavage specificity of trypsin, chymotrypsin, and elastase depend in part on:
A. The proximity of Ser 195 to the specificity pocket.
B. The size, shape, and charge of the specificity pocket.
C. The distance between the oxyanion hole and the specificity pocket.
D. * The presence of a low-barrier hydrogen bond in the specificity pocket.
E. The absence of water in the specificity pocket.
1112.
The compound having the formula H2N—CO—NH—CH2—CH2—CH2—CH—
COOH is NH2
A. * Lysine
B. Glutamine
C. Serine
D. Citrulline
E. Only C
1113.
The compound used by fish for the purpose of disposing of excess nitrogen is
A. * Uric acid
B. Urea
C. Ammonia
D. Aspartate
E. Fumarate
1114.
The concentration of urea in the blood of healthy man is:
A. 10-12 mM/l
B. 12-20 mM/l
C. 1-2 mM/l
D. * 3-8 mM/l
E. 15-18 mM/l
1115.
The concentration of urea is highest in
A. Renal vein
B.
C.
D.
E.
1116.
A.
B.
C.
D.
E.
1117.
A.
B.
C.
D.
E.
1118.
A.
B.
C.
D.
E.
1119.
A.
B.
C.
D.
E.
1120.
A.
B.
C.
D.
E.
1121.
A.
B.
C.
D.
E.
1122.
A.
B.
C.
D.
E.
1123.
A.
B.
C.
D.
E.
* Hepatic portal vein
Dorsal aorta
Hepatic vein
Only C
The concentration of urea is least in
Renal artery
* Renal vein
Post canal
Dorsal aorta
Hepatic portal vein
The defective enzyme in histidinemia is
Histidine carboxylase
Histidine decarboxylase
* Histidase
Histidine oxidase
None of the above
The end product of protein digestion in G.I.T. is
Dipeptide
Tripeptide
Polypeptide
* Amino acid
Peptide
The enzyme carbamoyl phosphate synthetase requires
* Mg++
Ca++
Na+
K+
F+
The enzyme trypsin is specific for peptide bonds of
Basic amino acids
Acidic amino acids
* Aromatic amino acids
Next to small amino acid residues
Fatty acids
The enzymes of urea synthesis are found in
Mitochondria only
Cytosol only
* Both mitochondria and cytosol
Nucleus
Lysosoms
The following enzyme of urea cycle is present in cytosol:
Argininosuccinic acid synthetase
Argininosuccinase
Arginase
* All of these
Ornithin-carbamoilphosphatetransferase
The following has the highest protein efficiency ratio:
Milk proteins
Egg proteins
* Meat proteins
Fish proteins
All of the above
1124.
A.
B.
C.
D.
E.
1125.
A.
B.
C.
D.
E.
1126.
A.
B.
C.
D.
E.
1127.
A.
B.
C.
D.
E.
1128.
A.
B.
C.
D.
E.
1129.
A.
B.
C.
D.
E.
1130.
A.
B.
C.
D.
E.
1131.
A.
B.
C.
D.
E.
1132.
A.
B.
C.
The main site of urea synthesis in mammals is
* Liver
Skin
Intestine
Kidney
Lung
The main sites for oxidative deamination are
* Liver and kidney
Skin and pancreas
Intestine and mammary gland
Lung and spleen
Duodenum and spleen
The major constituent of the proteins of hair and keratin of skin:
Arginine
* Cysteine
Glycine
Asparagine
Glutamic acid
The major end product of protein nitrogen metabolism in man is
Glycine
Uric acid
* Urea
NH3
Serine
The major site of urea synthesis is
Brain
Kidneys
* Liver
Muscles
Lung
The most important function of the large intestine is:
Absorption of water and electrolytes
Secretion of enzymes
* Digestion of incompletely digested foods
Removal of toxic substances
All of the above are equally important
The number of ATP required for urea synthesis is
0
1
*2
3
12
The only correct statement about chymotrypsin is
It is formed from trypsin
* Carboxypeptidase converts trypsin into chymotrypsin
Its optimum pH is around 7
It hydrolyses peptide bonds involving basic amino acids
All of the above
The optimal pH for the enzyme chymotrypsin is
2.0
4.0
6.0
D. 8.0
E. * 7,0
1133.
The optimal pH for the enzyme pepsin is
A. 1.5–2.5
B. * 4.0–5.0
C. 5.2–?6.0
D. 5.8–6.2
E. 6,0 – 8,0
1134.
The optimal pH for the enzyme rennin is
A. 2.0
B. 4.0
C. 8.0
D. * 5.0
E. 7,0
1135.
The oxidative deamination of the amino acid alanine in muscle produces:
A. * One molecule of pyruvic acid and a molecule of ammonia
B. One molecule of pyruvic acid and a molecule of carbon dioxide
C. One molecule of pyruvic acid and another amino acid
D. One molecule of pyruvic acid and a molecule of water
E. One molecule of pyruvic acid and a molecule of urea
1136.
The pancreatic enzymes, trypsin, chymotrypsin and elastase all have:
A. The same catalytic triad at their active sites.
B. Similar sequences and tertiary structures.
C. * The same catalytic mechanism.
D. Smilar processing pathways from inactive zymogens.
E. All of the above choices are correct
1137.
The pH of an amino acid depends
A. * Optical rotation
B. Dissociation constant
C. Diffusion coefficient
D. Chain length
E. None of the above
1138.
The pH of an amino acid depends
A. * Optical rotation
B. Dissociation constant
C. Diffusion coefficient
D. Chain length
E. None of the above
1139.
The reservoir for nitrogen is
A. The atmosphere.
B. Rocks.
C. Ammonia.
D. Nitrates.
E. * Amino acids.
1140.
The small intestine is the main site of absorption of:
A. Carbohydrates absorbed as monosacchorides
B. Carbohydrates absorbed as amino acids by active transport
C. Fats absorbed into lacteals of the lymphatic system
D. * Proteins absorbed as small molecules or proteoses
E. A and C only
1141.
The sodium bicarbonate in pancreatic juice helps to raise the pH of chyme. What
substance originally lowered the pH of chyme?
A.
B.
C.
D.
E.
Chyle
H2CO3
* HCL
Alkali
Base
1142.
The synthesis of urea finishes with:
A. Condencation of citrulline from NH3
B. * Fermentative hydrolysis of arginine to ornithine and urea
C. Synthesis of arginine from citrulline
D. Oxidation to ornithine and urea with participation of O2 and arginine
E. Oxidation of arginine with participation of O2 to ornithine and urea
1143.
The useful reagent for detection of amino acids is
A. Molisch reagent
B. Dichlorophenol Indophenol
C. * Ninhydrin
D. Biuret
E. None of the above
1144.
The useful reagent for detection of amino acids is
A. Molisch reagent
B. Dichlorophenol Indophenol
C. * Ninhydrin
D. Biuret
E. None of the above
1145.
Transfer of the carbamoyl moiety of carbamoyl phosphate to ornithine is catalysed
by a liver mitochondrial enzyme:
A. Carbamoyl phosphate synthetase
B. * Ornithine transcarbamoylase
C. N-acetyl glutamate synthetase
D. N-acetyl glutamate hydrolase
E. Arginase
1146.
Trypsinogen is converted to active trypsin by
A. * Enterokinase
B. Bile salts
C. HCl
D. Mg2+
E. Colipase
1147.
Tyrosine could be considered as precursor of
A. Melanotonin
B. * Thyroid hormones
C. Melanin
D. Epinephrine
E. Insulin
1148.
Urea is transported by
A. * Plasma
B. Blood
C. RBC
D. WBC
E. None af the above
1149.
What amino acid in the process of urea biosynthesis splits to urea and ornithine?
A. Leucine
B. Citrulline
C. * Arginine
D. Valine
E. Proline
1150.
What biologically active substance is formed in the process of decarboxylation of
5-hydroxytryptophane?
A. Corticosterone
B. Thyroxine
C. * Serotonine
D. Histamine
E. Anserine
1151.
What type of covalent bonds link the amino acids in a protein?
A. * Peptide bonds
B. Hydrogen bonds
C. Ionic bonds
D. Glycosidic bonds
E. Ester bonds
1152.
When branched chain amino acids are deaminated in muscle, the ammonia
produced is mostly:
A. Converted into arginine and released from the muscle
B. * Converted into alanine and glutamine and released from the muscle
C. Converted into urea and released from the muscle
D. Ued to synthesise purines and pyrimidines in the muscle
E. Oxidised to form nitric oxide
1153.
Which of the following amino acids commonly acts as a nucleophilic group in
enzyme catalysed reaction mechanisms?
A. Serine
B. * Phenylalanine
C. Histidine
D. Valine
E. Tyrosine
1154.
Which of the following contributes nitrogen atoms to both purine and pyrimidine
rings?
A. * Aspartate
B. Carbamoyl phosphate
C. CO2
D. Glutamine
E. Fumarate
1155.
Which of the following is an amino acid that is found in proteins?
A. Adenosine
B. Adenine
C. * Alanine
D. Linoleic acid
E. Creatine
1156.
Which of the following is not a function of the pancreas?
A. Secretes insulin as well as glucagon into pancreatic duct
B. * Secretes alkaline juice which neutralizes chyme while chyme is stored in the
stomach
C. Secretes both endocrine and exocrine substances
D. Secretes lipase which acts on bile-emulsified fats to convert them to fatty acids
and glycerol
E. Secretes amylase which acts on starches to convert them to maltose
1157.
Which of the following is not an amino acid?
A. Glutamic acid
B.
C.
D.
E.
Aspartic acid
Glutamine
* Palmitic acid
Leucine
1158.
Which of the following reactions is carried out by species of Nitrobacter?
A. Nitrate _ N2
B. Nitrite _ nitrate
C. Nitrite-->ammonia
D. Nitrate _ nitrite
E. Ammonia _ nitrite
1159.
Which of the following takes place in the mouth?
A. Food is fashioned into a bolus
B. * Carbohydrate digestion begins
C. Food becomes partially liquified
D. All of the above
E. A and B only
1160.
Which of the followings gives a positive test for Ninhydrin?
A. Reducing sugars
B. Triglycerides
C. * Alpha aminoacids
D. Esterified Fats
E. Vitamins
1161.
Which of the molecule(s) is/are the most common donators of amino groups
during transaminations?
A. Tyrosine
B. * Glutamate
C. Alanine
D. Glutamin
E. A and C
1162.
Which of these is false about “gastric juice?”
A. * Is highly acidic because of its HCL content
B. Contains the intrinsic factor for absorption of vitamin B12
C. Is controlled in part by a hormone called gastrin
D. Becomes neutralized in the duodenum by the action of sodium bicarbonate from
the pancreas
E. None of the above
1163.
Which one of the following is an essential amino acid?
A. Arginine
B. Tyrosine
C. * Phenylalanine
D. Proline
E. Ornitine
1164.
With which of the listed cycles is connected the cycle of urea formation?
A. PFC
B. * Tricarboxylic acid cycle
C. Gluconeogenesis
D. Glycolysis
E. Lipogenesis hese
Real life situation questions
1. A 2-years-old child has symptoms of galactosemia. Among them the following is
marked:
A. Dermatitis
B. Dementia
C. Keratomalacia
D. * Cataract
E. Xerophtalmia
2. A 46 years old woman appealed to the doctor with complaints about pain in the
epigastrium after the fatty meal. Doctor suspected acute pancreatitis. Content of which
from the transferred substances in blood increased at this pathology?
A. Glucose
B. Aspartate amino transferase
C. Alanine amino transferase
D. * Amylase
E. Calcium
3. A child after the mixed meal which contains sucrose suffered from vomiting and cramps.
After the exception of sugar from a diet, these symptoms disappeared. Probably, that
patient suffers from:
A. Galactosemia
B. Undigestion of lactose
C. Essential pentoseuria
D. Glycogenosis
E. * Undigestion of fructose
4. A child was delivered to a reanimation with such symptoms: vomit, diarrhea, abnormal
physical and mental development, cataract. Laboratory test showed galactosemia. The
deficiency of what enzyme does take place?
A. Galactokinase
B. UDP glucose-4-epimerase
C. * Hexose-1-phosphate-uridiltransferase
D. UDP- glucose –pyrophosphorilase
E. Glucose-6-phosphate dehydrogenase.
5. A deficiency in thiamin causes the disease beriberi. Which might you expect to have a
higher than normal blood concentration in an individual with this condition?
A. isocitrate
B. * pyruvate
C. oxaloacetate
D. acetyl CoA
E. malate
6. A new-born child has diarrhea, vomiting and cataract. Disorder of synthesis of what
enzyme causes this disease?
A. * Galactose – 1 – phosphate uridyl transpherase
B. Glucose - 6 phosphatase
C. Hexokinase
D. Glycogen synthase
E. Glucose phosphate isomerase
7. A new-born child has diarrhea, vomiting and cataract. Disorder of synthesis of what
enzyme caused this disease?
A. * Galactose – 1 – phosphate uridyl transpherase
B. Glucose - 6 phosphatase
C. Hexokinase
D. Glycogen synthase
E. Glucose phosphate isomerase
8. A newborn suffers from the fatty degeneration of liver, takes place a galactosuria and
amino aciduria, increased level of total bilirubin in blood. What matter is it necessary to
exclude from the diet of child?
A. * Lactose
B. Fatty acids
C. Phenylalanine
D. Cholesterol
E. Saccharose
9. A patient complains about acute stomach-ache, nausea, vomit. Diastase activity is 800
grammes/l• hour What diagnosis is the most possible?
A. Acute cholecystitis
B. * Acute pancreatitis
C. Ulcer of stomach
D. Acute appendicitis
E. Enterocolitis
10. A patient has metabolic acidosis. How will it change activity of intracellular enzymes?
A. Activity of intracellular enzymes do not change substantially.
B. Activity of mitochondrial enzymes increased, activity of lysosomal enzymes
inhibited, which is accompanied the decline of catalytic processes.
C. * Activity of mitochondrial enzymes is low, activity of lysosomal enzymes, which
will result in strengthening of catalytic processes, is increased.
D. Total inhibition of all of tissue enzymes.
E. Total activating of all of tissue enzymes.
11. A patient has metabolic acidosis. How will it change activity of intracellular enzymes?
A. Activity of intracellular enzymes does not change substantially.
B. Activity of mitochondrial enzymes increases, activity of lysosomal enzymes
inhibits, which is accompanied the decline of catalytic processes.
C. * Activity of mitochondrial enzymes is low, activity of lysosomal enzymes, which
will result in strengthening of catalytic processes, is increased.
D. Total inhibition of all of tissue enzymes.
E. Total activating of all of tissue enzymes.
12. A patient is found to be deficient in the enzyme galactose 1-phosphate
uridylyltransferase. Specifically due to this deficiency what might a doctor recommend?
A. avoid all strenuous exercise
B. eat a fat-free diet
C. increase intake of vitamin C
D. * avoid ingestion of milk and milk products
E. increase intake of milk
13. A patient is found to be deficient in the enzyme galactose 1-phosphate
uridylyltransferase. Specifically due to this deficiency what might a doctor recommend?
A. avoid all strenuous exercise
B. eat a fat-free diet
C. increase intake of vitamin C
D. * avoid ingestion of milk and milk products
E. all of the above.
14. A patient was diagnosed with beri-beri disease. Activity of which enzyme is inhibited in
an organism of the patient?
A. Fumarase
B. Citrate synthase
C. Malate dehydrogenase
D. Succinate dehydrogenase
E. * Pyruvate dehydrogenase
15. A patient with pellagra (a result of vitamin of PP luck) has low activity of the followings
enzymes of glycolysis:
A. Glucokinase and enolase
B. * Glyceraldehyde 3-Phosphate dehydrogenase and lactate dehydrogenase
C. Aldolase and pyruvatekinase
D. Lactate dehydrogenase and hexokinase
E. Pyruvate dehydrogenase and aldolase
16. After hard physical work during rest there are stimulation some metabolic ways of
carbohydrates, except:
A. Glycogenes
B. Gluconeogenesis from lactat
C. Gluconeogenesis from glycerin
D. * Glycolysis, Krebs cycle
E. Gluconeogenesis from amino acids
17. As a result of worker's exhausting muscle activity the strongly decreased buffer capacity
of blood occurs. Accumulation of which substance in the blood explains this event?
A. 3-phosphoglycerate
B. pyruvate
C. 1,3-biphosphoglycerate
D. * lactate
E. E-ketoglutarate
18. At entering in organism big amount of toxic substances glucose-6-phosphate in a liver
including in:
A. Glycolysis
B. Glycogenogenesis
C. Glycolysis, Krebs cycle
D. * Pentosophosphate cycle
E. Gluconeogenesis
19. At the short starvation after the consumption of carbohydrate meal, hexoses in a liver
transform into glucose-6-phosphate, which can be used in all pathways, except:
A. * Gluconeogenesis
B. Pentosophosphate cycle
C. Synthesis of glycogen
D. Glycolysis
E. Synthesis of fatty acids
20. By the ambulane car a patient was delivered to the hospital, who became unconscious in
the street. Biochemical blood analysis showed: ketone bodies concentration 6,8 mmol/l,
urea concentration 7,5 mmol/l, increased concentration of C-protein, amount of calcium
2,2 mmol/l, blood glucose 2,5 mmol/l. What is the possible reason for getting
unconscious?
A. * hypoglycemia
B. ammonia formation malfunction
C. hypocalcemia
D. acetonemia
E. hyperthyroidism
21. Concentration of pyruvic acid and lactic acid in blood of a patient with sympthoms of
beri beriis is increased due to deficiency of the vitamin:
A. * Thiamin
B. Riboflavin
C. Niacin
D. Pantothenic acid
E. Ascorbic acid
22. During wet beriberi, peripheral vessels dilate and heart muscles loose their contractility.
Some people think this is a result of a failure of the TCA cycle. Evidence for this is an
increase in the
A. Rate of production of succinate
B. Rate of production of NADH
C. * Concentration of alpha-ketoglutarate and pyruvate
D. Production of ATP
E. Production of FADH2
23. For diagnosis of patients with an innate and early cataract, it is necessary to inspect them
on a presence of:
A. Fructosemia
B. Hyperuricemia
C. Pentosemia
D. Hyperlactemia
E. * Galactosemia
24. If a patient with the deficiency of lactase uses milk and milk products, such symptom can
be observed as:
A. An increase of concentration of glucose is in blood
B. Appearance of galactose in urine
C. An increase of concentration of galactose in blood
D. * Diarrhea
E. Dermatitis
25. In blood and urine of a patient it was takes place increase value of pyruvic and ?ketoglutaric acids. The deficiency of which coenzyme causes these changes?
A. Pyridoxal phosphate
B. * Thiamin pyrophosphate
C. Flavin mononucleotide
D. Biotin
E. Ubiqinon
26. In blood and urine of a patient it was takes place increase value of pyruvic and
ketoglutaric acids. The deficiency of which coenzyme causes these changes?
A. Pyridoxal phosphate
B. * Thiamin pyrophosphate
C. Flavin mononucleotide
D. Biotin
E. Ubiqinon
27. In some Diabetic patients, glucose increases disproportionately and is unresponsive to an
insulin challenge; under these conditions, how would one's liver normally respond?
A. Phosphorylating glucose for entry into the glycolytic pathway
B. Saturating glucokinase with glucose
C. Phosphorylating glucose for entry into the glycogen synthesis pathways
D. a and b only
E. * a and c only
28. In the organism of patient with inherited galactosemia accumulate galactose-1-phosphate,
a free galactose and a such alcohol - toxic product of its reduction as:
A. Glycerol
B. * Dulcitol
C. Inositol
D. Retinol
E. Ethanol
29. Patient has sympthoms of beri beri (lack of B1). B1 formes coenzyme TPP which is
involved in:
A. * Oxidative decarboxylation
B. Hydroxylation
C. Transamination
D. Carboxylation
E. Oxidation
30. Patient has sympthoms of beri beri. Both Wernicke’s disease and beri beri can be
reversed by administrating:
A. Retinol
B. * Thiamin
C. Pyridoxine
D. Vitamin B12
E. Cholecalciferol
31. Patient with atherosclerosis suffers from severe ischemic heart disease. Which process
will prevail in the cardiac muscle?
A. * anaerobic glucose decomposition
B. ketone bodies synthesis
C. glycerin oxidation
D. aerobic glucose decomposition
E. ketone bodies oxidation
32. Patients which suffer from inherited undigestion of fructose must not use such
disaccharide as:
A. Cellobiose
B. Lactose
C. Maltose
D. * Sucrose
E. All of the above
33. People, who for a long time were in the condition of hypodynamia after intensive
physical activity have strong muscle pain. What is the most probable reason of this
condition?
A. * increased concentration of lactic acid in the muscles
B. increased decomposition of muscle proteins
C. concentration of creatine in the muscles
D. increased muscle stimulation
E. increase of ADP in the muscles
34. Teturam which is the inhibitor of aldehyde dehydrogenase used in medical practice for
the prophylaxis of alcoholism. An increase in blood of what metabolite causes disgust of
an alcohol?
A. malonic aldehyde
B. Ethanol
C. * Acetaldehyde
D. Proponic aldehyde
E. Methanol
35. The accumulation of pyruvate in blood, decrease of activity of red blood cells
transketolase takes place of a 36 years patient, which suffers with chronic alcoholism.
Insufficiency of coenzyme form of which vitamin causes such changes?
A. * Thiamine diphosphate
B. Carboxybiotin
C. Metylcobalamin
D. Phosphopyridoxal
E. Tetrahydrofolate
36. The inherited genetic defects result in violation of synthesis of some enzymes in the
human organism. The defect of which enzyme results in disorders of splitting of lactose:
A. * Lactase
B. Maltase
C. Sucrase
D. Amylase
E. Peptidase
37. The patient complains for undue fatiguability after working day, constant thirst. A doctor
suspected diabetes mellitus. Choose the blood plasma glucose concentration, which
proves the diagnosis:
A. * 8,5 mmol/l
B. 2 mmol/l
C. 4,5 mmol/l
D. 5 mmol/l
E. 3,3 mmol/l
38. The patient has inhibited oxidative pyruvate decarboxylation. How many ATP molecules
less the organism will receive at the oxidation of a single glucose molecule?
A. * 30 ATP molecules per each glucose molecule
B. 36 ATP molecules per each glucose molecule
C. 2 ATP molecules per each glucose molecule
D. there will be no ATP deficiency because less synthesis is compensated by
intensive fat decomposition
E. 12 ATP molecules per each glucose molecule
39. The rate at which pyruvate from glycolysis is used by the TCA cycle to produce energy is
regulated by pyruvate dehydrogenase. During muscle contraction, this enzyme is
A. Inhibited by increases in the calcium concentration.
B. Activated by increased in acetyl CoA
C. Activated by increases in NADH
D. * Activated by increases in ADP
E. Inhibited by increases in AMP
40. Vomiting, diarrhea after consuming milk, enlargement of the liver, jaundice,
cirrhosis,cataracts, retardation of the mental development, elevated blood-galactose level,
absence of the galactose 1-phosphate uridyl transferase in red blood cells are the
symptoms of:
A. lactose intolerance
B. hypolactasia
C. * galactosemia
D. all of the above
E. diabetes mellitus
41. Vomiting, diarrhea after consuming milk, enlargement of the liver, jaundice,
cirrhosis,cataracts, retardation of the mental development, elevated blood-galactose level,
absence of the galactose 1-phosphate uridyl transferase in red blood cells are the
symptoms of:
A. lactose intolerance;
B. hypolactasia;
C. * galactosemia;
D. all of the above.
E. diabetes mellitus
42. Which metabolic ways transformation of glucose-6-phosphate will prevail in a liver at
the condition activation of biosynthetic processes?
A. Gluconeogenesis
B. Glycolisis
C. Transformation of glucose-6-phosphate on free glucose and phosphate
D. * Pentosophosphate cycle
E. Glycogenesis
43. Yeast will normally convert pyruvate to ethanol. Why is this better for the yeast than a
conversion to lactate?
A. Conversion to ethanol releases more NAD+ per mole than the conversion to
lactate.
B. The carbon atoms are more oxidized in ethanol than in lactate.
C. * Ethanol is neutral, but lactate production is accompanied by a sharp decrease in
pH.
D. Ethanol production is not better. Yeast normally produces ethanol and lactate in
equilmolar amounts.
E. All of the above
44. 42 years old man for dinner ate a big portion of spaghetti, a piece of cake, had a glass of
sweet tea. What hepatocyte enzyme activity is activated mostly ?
A. * Hexokinase
B. Glucose-6-phosphatase
C. Glucose-6-phosphatdehydrogenase
D. Glycogen phosphorylase
E. E-galactosidase
45. 8 month old child has vomiting and diarrhea after having fruit juice. There is an increased
level of fructose in the blood. This is conneceted with the lack of the enzyme:
A. * Fructokinase
B. Ketose-1-phosphate aldolase
C. Isomerase
D. Triosephosphate isomerase
E. Phosphorylase
46. A 62 yaears old woman with diabetes mellitus has cataract (lenticular opacity) by . What
is the reason?
A. Ketogenesis
B. Lipolysis
C. Protein proteolysis
D. * Protein glycosylation
E. . Gluconeogenesis
47. A doctor was visited by a patient with complains on constant thirst. Hyperglycemia,
polyuria and increased amount of 17-ketosteroid in the blood were found. What disease is
the most possible?
A. Insulin-dependent diabetes
B. * Steroid diabetes
C. Myxedema
D. I type glycogenolysis
E. Addison's disease
48. A key element in the regulation of glycogen metabolism is the phosphorylation by
protein kinase A of phosphorylase kinase which ______ (activates, inactivates) and the
phosphorylation of glycogen synthase which _______ (activates, inactivates).
A. Activates; activates
B. * Activates; inactivates
C. Inactivates; inactivates
D. Inactivates; activates
49. A patient has developed general adiposity and atherosclerosis as a result of everyday
usage of:
A. * 600 g carbohydrates
B. 150 g protein
C. 50 g vegetable oil
D. 100 g fats
E. 10 mg vitamin A
50. A patient is found to be deficient in the enzyme galactose 1-phosphate
uridylyltransferase. Specifically due to this deficiency what might a doctor recommend?
A. Avoid all strenuous exercise
B. Eat a fat-free diet
C. Increase intake of vitamin C
D. * Avoid ingestion of milk and milk products
E. Increase intake of vitamin B1
51. A woman of 58 years is hard condition. Consciousness is blear, skin is dry, eyes are
sunken, cyanosis, the smell of rotting apples from the mouth. Blood glucose - 15,1
mmol/l, urine glucose 3,5%. What is the most real reason for such condition?
A. Uremic coma
B. Hypovalemic coma
C. Anaphylactic shock
D. * Hyperglycemic coma
E. Hypoglycemic coma
52. According to the classic glucose-fatty acid cycle, which two metabolites are predicted to
increase in skeletal muscle when fat becomes the major substrate oxidized?
A. ADP, AMP
B. * Acetyl CoA, citrate
C. Pyruvate dehydrogenase, phosphofructokinase
D. Glycogen, triglyceride
E. All of the above
53. All are true for ?-amylase EXCEPT:
A. Catalyzes hydrolysis of ? (1 4) linkages in starch.
B. Is an important component of saliva and pancreatic juice.
C. * Catalyzes hydrolysis of ? (1 6) linkages in amylopectin.
D. Catalyzes hydrolysis of ? (1 4) linkages in glycogen.
E. Activity is reduced in highly branched regions of polysaccharides and stops four
residues from any branch point.
54. All enzymes of gluconeogenesis are located in:
A. * Cytoplasm
B. Matrix of mitochondria
C. Inner membrane of mitochondria
D. None of the above.
E. Nucleus
55. An enzyme used in both glycolysis and gluconeogenesis is:
A. * 3-phosphoglycerate kinase.
B. Glucose 6-phosphatase.
C. Hexokinase.
D. Phosphofructokinase-1.
E. Pyruvate kinase.
56. At the subcellular level, where do the reactions of gluconeogenesis occur?
A. Al in the cytosol
B. Al in the mitochondria
C. In the cytosol and the endoplasmic reticulum
D. * In the cytosol and the mitochondra
E. In the plasma membrane
57. Biochemical blood analysis of a 1-year old child showed hypoglycemia. After injection
of epinephrine hyperglycemia did not appear. During clinical inspection were observed
growth inhibition, spasms, increase of liver and kidney. Lack or excess of which enzyme
causes such pathology?
A. Lack of muscle phosphorylase
B. * Excess of glucose-6-phosphatase
C. Lack of liver phosphorylase
D. Excess of muscle phosphorylase
E. Lack of glucokinase
58. Biochemical blood analysis of a 32 years old patient, who starved for 3 days, showed that
the amount of glucose is normal. What process provides the normal glucose level in the
blood of a healthy man after 3-4 days of starvation?
A. * Gluconeogenesis
B. Glycolysis
C. Pentose-phosphate pathway
D. Krebs cycle
E. Glycogen synthesis
59. Bypassed reactions in gluconeogenesis are:
A. * Conversion of pyruvate to phosphoenolpyruvate; conversion of fructose 1,6bisphosphate to fructose 6-phosphate; conversion of glucose 6-phosphate to
glucose;
B. Conversion of pyruvate to phosphoenolpyruvate; conversion 2-phosphoglicerate
to 3-phosphoglicerate; conversion of fructose 6-phosphate to glucose 6phosphate;
C. Conversion of 2-phosphoglicerate to 3-phosphoglicerate; conversion of fructose
6-phosphate to glucose 6-phosphate; conversion of glucose 6-phosphate to
glucose;
D. Conversion of 3-phosphoglicerate to 1,3-biphosphoglycerate; conversion of
fructose 1,6-bisphosphate to fructose 6-phosphate; conversion of glucose 6phosphate to glucose.
E. None of the above
60. Concerning some general principles of glucose metabolism:
A. * Gluconeogenesis is stimulated when plasma glucose is low.
B. Gluconeogenesis is the formation of glucose from glycogen.
C. Glycogenesis is stimulated when cellular ATP reserves are low.
D. Glucose may be synthesized from fatty acids.
E. Glucose may be synthesized from vitamins.
61. During fasting, what role does gluconeogenesis play?
A. Aids in the release of glucose from stored glycogen in the liver.
B. * Uses protein (amino acids) to restore blood glucose levels.
C. Produces products that slow the oxidation of pyruvate.
D. Helps transport glucose across the blood-brain barrier to maintain brain glucose
levels.
E. None of the above
62. During strenuous exercise, the NADH formed in the glyceraldehyde 3-phosphate
dehydrogenase reaction in skeletal muscle must be reoxidized to NAD+ if glycolysis is to
continue. The most important reaction involved in the reoxidation of NADH is:
A. Dihydroxyacetone phosphate > glycerol 3-phosphate
B. Glucose 6-phosphate > fructose 6-phosphate
C. Isocitrate > ?-ketoglutarate
D. Oxaloacetate > malate
E. * Pyruvate > lactate
63. Each following line is a statement about Glycolysis.
A. n enzyme which catalyses a reaction consuming ATP
B. * n enzyme which catalyses a reaction generating ATP.
C. n enzyme which has an allosteric ATP site
D. An enzyme which catalyses a reaction fixing Pi.
E. An enzyme which catalyses a reaction generating NADH.
64. Each of the following is a statement about energy metabolism. If it can be applied to
carbohydrates:
A. Can provide energy to flowering plants.
B. Can provide energy to some germinating seeds.
C. * Is the usual source of energy for brain.
D. Is the usual source of energy for skeletal muscle.
E. Can provide energy to skeletal muscle.
65. Each of the following is the name of an enzyme found in the cytosol of hepatocytes,
which are capable of both glycolysis and gluconeogenesis.
A. Ppyruvate kinase
B. * Fructose 1,6-bisphosphatase
C. Phosphoenolpyruvate carboxykinase
D. Hexokinase
E. Phosphofructokinase 1
66. For each molecule of glucose converted to pyruvate in the glycolytic pathway ___
molecules of ATP are used initially (Stage I) and ____ molecules of ATP are produced
(Stage II) for an overall yield of ___ molecules of ATP/glucose. The "ATP math" is:
A. * -2 + 4 = 2
B. -1 + 4 = 3
C. -2 + 5 = 3
D. -1 + 2 = 1
E. 2 + 2 = 4
67. ?For long-term storage, glucose is converted to ______ , while for short-term storage,
glucose is converted to
A. Fat, glycogen
B. * Glycogen, ketone bodies
C. Fat, pyruvic acid
D. Glycogen, protein
E. Pyruvic acid, fat
68. For oxaloacetate to be used as a precursor in gluconeogenesis, it must first be transported
across the inner mitochondrial membrane in the form of:
A. * Malate.
B. Oxaloacetate.
C. Aspartate
D. Citrate
E. Both A and C are correct.
69. Fructose 2,6-bisphosphate (F-2,6-BP) is a:
A. Activator of phosphofructokinase 1;
B. Inhibitor of phosphofructokinase 1;
C. Activator of hexokinase;
D. * Activator of pyruvate-kinase.
E. All of the above.
70. Galactosemia is a genetic error of metabolism associated with:
A. Deficiency of galactokinase.
B. Deficiency of UDP-glucose.
C. * Deficiency of UDP-glucose: galactose 1-phosphate uridylyltransferase.
D. Excessive ingestion of galactose.
E. Inability to digest lactose.
71. Gluconeogenesis is
A. The result of ?-amylase activity
B. The formation of glycogen
C. The formation of starches
D. * The formation of glucose from noncarbohydrates
E. The formation of glucose from other carbohydrates
72. Gluconeogenesis shares some, but not all, enzymes with the glycolytic pathway. It would
appear to be more efficient if both pathways used all of the same enzymes since the
pathways are essentially the reverses of each other. Why don't both pathways use all of
the same enzymes?
A. The reactions where enzymes differ occur in different parts of the cell for
glycolysis versus gluconeogenesis.
B. Enzymes can catalyze a reaction only in one direction, so naturally the two
pathways have some enzymes that differ.
C. In tissues where gluconeogenesis occurs, the glycolytic enzymes are present at
extremely low concentrations.
D. * Three of the reaction steps in gluconeogenesis would have prohibitively large,
positive free energies if they used glycolytic enzymes for their catalysis.
E. Only A
73. Gluconeogenesis uses the same enzymatic reaction of glycolysis except for
A. Pyruvate kinase
B. * 4 irreversible reactions in glycolysis
C. 3 irreversible reactions in glycolysis
D. 2 irreversible reactions in glycolysis
E. 1 irreversible reactions in glycolysis
74. Glucose 1-phosphate formed by glycogenolysis is converted to glucose 6 phosphate by
phosphoglucomutase because
A. Glucose 6-phosphate is more stable
B. Glucose 6-phosphate is converted to free glucose
C. * Glucose 6-phosphate is an intermediate in several pathways, including
glycolysis
D. Glucose 6-phosphate can be transported to the liver
E. All of the above
75. Glucose can be produced by gluconeogenesis by all the sources below except:
A. Glycerol
B. Amino acids
C. * Fatty acids
D. Lactate
E. Pyruvat
76. Glucose labeled with 14C in C-1 and C-6 gives rise in glycolysis to pyruvate labeled in:
A. A and C.
B. All three carbons.
C. Its carbonyl carbon.
D. Its carboxyl carbon.
E. Its methyl carbon.
77. Glucose-6-phosphate dehydrogenase deficiency is the ______________ which is
characterized by ___________________________________.
A. Genetically determined disease; the absence of receptors to LDL on cells and
prominent atherosclerosis;
B. Diet-induced disease; the deficit of thiamin in the organism and neurological and
cardiac symptoms;
C. * Genetically determined disease; the low level of reduced glutathione in
erythrocytes and increased susceptibility of erythrocytes to hemolysis;
D. Disease induced by environmental factors; the high level of oxidized glutathione
in the red blood cell and increased resistance of erythrocytes to hemolysis.
E. None of the above.
78. Glycogen phosphorylase is ________ (more, less) active when phosphorylated and it is
__________ (activated, inhibited) by glucose 6-phosphate.
A. More; activated
B. Less; activated
C. * More; inhibited
D. Less; inhibited
E. None of the above
79. Glycogen synthesis takes place in the following oragans:
A. Brain and muscles
B. * Liver and muscles
C. Liver and kidney
D. Liver and pancreas
E. All of the above
80. Glycolysis is defined as:
A. Aerobic process in which glucose is transformed to CO2 and H2O;
B. Anaerobic process in which glucose is transformed to pyruvate;
C. Anaerobic process in which glucose is converted to pentoses and NADPH is
produced;
D. Aerobic process in which glucose is converted to glycogen.
E. * Anaerobic process in which glucose is transformed to lactate;
81. Glycolysis is the name given to a metabolic pathway occurring in many different cell
types. It consists of 11 enzymatic steps that convert glucose to lactic acid. Glycolysis is
an example of:
A. Aerobic metabolism.
B. Anabolic metabolism.
C. A net reductive process.
D. * Fermentation.
E. Oxidative phosphorylation.
82. I type glycogenosis (hepatorenal glycogenosis) is connected with the deficit of glucose-6phosphatase in the liver, kidney and intestines mucous tunic. Which of the mentioned
symptoms is not characteristic for this disease?
A. Hyperglycemia
B. Hypoglycemia
C. * Hyperazotemia
D. Lipemia
E. Ketonuria
83. If glucose labeled with 14C at C-1 (the aldehyde carbon) were metabolized in the liver,
the first radioactive pyruvate formed would be labeled in:
A. All three carbons.
B. Both A and C.
C. Its carbonyl carbon.
D. Its carboxyl carbon.
E. * Its methyl carbon.
84. If glucose labeled with 14C in C-1 were fed to yeast carrying out the ethanol
fermentation, where would the 14C label be in the products?
A. In C-1 of ethanol and CO2
B. In C-1 of ethanol only
C. * In C-2 (methyl group) of ethanol only
D. In C-2 of ethanol and CO2
E. In CO2 only
85. In an anaerobic muscle preparation, lactate formed from glucose labeled in C-3 and C-4
would be labeled in:
A. All three carbon atoms
B. Only the carbon atom carrying the OH
C. * Only the carboxyl carbon atom
D. Only the methyl carbon atom
E. The methyl and carboxyl carbon atoms
86. In glycolysis, fructose 1,6-bisphosphate is converted to two products with a standard
free-energy change (?G'°) of 23.8 kJ/mol. Under what conditions (encountered in a
normal cell) will the free-energy change (?G) be negative, enabling the reaction to
proceed to the right?
A. If the concentrations of the two products are high relative to that of fructose 1,6bisphosphate.
B. The reaction will not go to the right spontaneously under any conditions because
the ?G'° is positive.
C. Under standard conditions, enough energy is released to drive the reaction to the
right.
D. * When there is a high concentration of fructose 1,6-bisphosphate relative to the
concentration of products.
E. When there is a high concentration of products relative to the concentration of
fructose 1,6-bisphosphate.
87. In humans, gluconeogenesis:
A. * Can result in the conversion of protein into blood glucose.
B. Helps to reduce blood glucose after a carbohydrate-rich meal.
C. Is activated by the hormone insulin
D. Is essential in the conversion of fatty acids to glucose.
E. Requires the enzyme hexokinase.
88. In some Diabetic patients, glucose increases disproportionately and is unresponsive to an
insulin challenge; under these conditions, how would one's liver normally respond?
A. Phosphorylating glucose for entry into the glycolytic pathway
B. Saturating glucokinase with glucose
C. Phosphorylating glucose for entry into the glycogen synthesis pathways
D. A and b only
E. * A and c only
89. In the alcoholic fermentation of glucose by yeast, thiamine pyrophosphate is a coenzyme
required by:
A. Aldolase.
B. Hexokinase.
C. Lactate dehydrogenase.
D. * Pyruvate decarboxylase.
E. Transaldolase.
90. In the blood of the patient the amount of glucose on the empty stomach is 5,6 mmol/l,
after an hour after sugary loading – 13,8 mmol/l, after two hours – 9,2 mmol/l. Such
figures are typical for:
A. Healthy man
B. Patient with thyrotoxicosis
C. * Patient with hidden form of diabetes mellitus
D. Patient with Cushing's syndrome
E. Patient with acromegaly
91. In the Cori cycle, gluconeogenesis occurs in _____ and glycolysis in ______.
A. * Liver; muscle
B. Liver; liver
C. Muscle; muscle
D. Muscle; liver
E. Heart; brain
92. Inorganic fluoride inhibits enolase. In an anaerobic system that is metabolizing glucose as
a substrate, which of the following compounds would you expect to increase in
concentration following the addition of fluoride?
A. * 2-phosphoglycerate
B. Glucose
C. Glyoxylate
D. Phosphoenolpyruvate
E. Pyruvate
93. Lactate is formed from pyruvate in an _____ organism in ______ conditions.
A. Yeast; aerobic;
B. Animal; aerobic;
C. Fungi; aerobic;
D. * Animal; anaerobic.
E. Yeast; aerobic.
94. More ATP is formed from glucose in glycogen than from free glucose, even free glucose
released from glycogen because
A. Limit dextrin contains additional molecules
B. The debranching enzyme releases free glucose
C. * The glucose is already phosphorylated after glycogen phosphorylase action
D. The glucose bypasses glycolysis
E. All of the above
95. Nervous tissue depends upon glucose for its primary source of energy. However, these
molecules can be used as an alternative source of energy by nervous tissue:
A. * Ketones
B. Steroids
C. Fatty acids
D. Amino acids
E. All of the above
96. Pentoses, which are formed in pentose-phosphate cycle are used for the following
processes, except:
A. * Glycogen synthesis
B. Coenzyme synthesis
C. Nucleic acid synthesis
D. ATP synthesis
E. Cyclic adenosine monophosphate synthesis
97. PFK-2 and fructose 2,6-bisphosphatase are two names for the same enzyme. The name
PFK-2 is used for the enzyme's catalysis of the phosphorylation of fructose 6-phosphate
to fructose 2,6-bisphosphate. The name fructose 2,6-bisphosphatase is used for its
catalysis of the reverse reaction. What is unique about this enzyme that makes it logical
to use both names?
A. It is one of very few enzymes that can catalyze both the forward and reverse
reactions.
B. The enzyme is a monomer when catalyzing the phosphorylation reaction and a
dimer when catalyzing the reverse reaction.
C. The forward and reverse reactions occur in different compartments within the
cell, so a different name is used for each activity.
D. * The enzyme is bifunctional. The forward and reverse reactions are catalyzed by
different sites on the same enzyme.
E. All of the above.
98. Phosphofructokinase, the major flux-controlling enzyme of glycolysis is allosterically
inhibited by ___ and activated by ___.
A. AMP Pi
B. ADP AMP
C. Сitrate ATP
D. ATP PEP
E. * ATP ADP
99. Protein kinase A, which stimulates glycogen degradation, is activated directly by
A. Glucagon
B. Insulin
C. Epinephrine
D. * Cyclic AMP
E. Adrenergic receptors
100.
Regarding diabetes mellitus:
A. * Is always the result of a failure of insulin synthesis.
B. Is characterized by a fall in urine output.
C. Is associated with an increase in lipolysis
D. eads to hypoglycemia.
E. None of the above
101.
The anaerobic conversion of 1 mol of glucose to 2 mol of lactate by fermentation
is accompanied by a net gain of:
A. 1 mol of ATP.
B. 1 mol of NADH.
C. * 2 mol of ATP.
D. 2 mol of NADH.
E. None of the above.
102.
The concentration of glucose in the blood of a patient is 4,5 mmol/l, glucose
appears in urine. The possible reason for this condition:
A. Thyroid gland hyperfunctioning
B. Acromegaly
C. Glycogenosis
D. * Kidney disease
E. Stress
103.
The conversion of 1 mol of fructose 1,6-bisphosphate to 2 mol of pyruvate by the
glycolytic pathway results in a net formation of:
A. 1 mol of NAD+ and 2 mol of ATP.
B. 1 mol of NADH and 1 mol of ATP.
C. 2 mol of NAD+ and 4 mol of ATP.
D. 2 mol of NADH and 2 mol of ATP.
E. * 2 mol of NADH and 4 mol of ATP.
104.
The conversion of glycogen to glucose-6-phosphate is the first step in
A. Glycolysis
B. Gluconeogenesis
C. * Glycogenolysis
D. Glycogenesis
E. A and C
105.
The enzyme found only in the liver that removes phosphate groups and can, thus,
release free glucose for distribution around the body by the blood is
A. Glycogen synthetase
B. Glucose-6-phosphatase
C. * Glycogen phosphorylase
D. Glucose isomerase
E. only B
106.
The enzyme which the key regulatory step in glycogen biosynthesis is
A. * Glycogen synthase
B. Glycogenin
C. Branching enzyme
D. Phosphoglucomutase
E. UDP-glucose pyrophosphorylase
107.
The family of GluT (glucose transporters) is:
A. * The proteins embedded into the cell membrane and facilitating the glucose
transport across the membrane;
B. The proteins embedded into the cell membrane and inhibiting the glucose
transport across the membrane;
C. The cytoplasmic proteins transporting glucose in the cytoplasm of cell;
D. The proteins of blood plasma transporting glucose via the blood.
E. None of the above.
108.
The initiation of glycogen synthesis (ie the very first glucose residue added )
occurs by transfer of glucose from UDP-glucose to the:
A. * 4-position of free glucose.
B. 2-position of fructose.
C. Non-reducing end of maltose.
D. Serine-OH group of glycogen synthase.
E. Tyrosine-OH group of glycogenin.
109.
The interconversion of which pair of substrates is used as a regulatory point in
gluconeogenesis?
A. Lactate and pyruvate
B. Dihydroxyacetone phosphate and glyceraldehyde-3-phosphate
C. * Fructose 1,6-bisphosphate and fructose 6-phosphate
D. Phosphoenolpyruvate and 2-phosphoglycerate
E. None of the above
110.
The main function of the pentose phosphate pathway is to:
A. Give the cell an alternative pathway should glycolysis fail.
B. Provide a mechanism for the utilization of the carbon skeletons of excess amino
acids.
C. Supply energy.
D. Supply NADH.
E. * Supply pentoses and NADPH.
111.
The main hormones that regulate the synthesis and decomposition of glycogen
are:
A. Insulin, glucocorticoids, thyroxin;
B. Glucagons, glucocorticoids, vasopressin;
C. * Insulin, glucagons, epinephrine;
D. Glucocorticoids, glucagons, epinephrine.
E. All of the above.
112.
The main precursors for the glucose synthesis in gluconeogenesis are:
A. Lactate, pyruvate, glycerol and cholesterol;
B. * Lactate, pyruvate, glycerol and amino acids;
C. Lactate, pyruvate, glycerol and glycogen;
D. Cholesterol, fatty acids, glycerol and amino acids.
E. The accelerating of glycolysis
113.
The main precursors for the glucose synthesis in gluconeogenesis are:
A. Lactate, pyruvate, glycerol and cholesterol;
B. * Lctate, pyruvate, glycerol and amino acids;
C. Lactate, pyruvate, glycerol and glycogen;
D. Cholesterol, fatty acids, glycerol and amino acids.
E. The accelerating of glycolysis
114.
The organ most responsible for extracting and converting lactic acid to pyruvic
acid, and which ultimately reforms and releases free glucose via the Cori cycle into the
bloodstream, is the
A. Liver.
B. Brain.
C. Cardiac muscle.
D. * Skeletal muscle.
E. Heart
115.
The Pasteur Effect is:
A. The accelerating of glycolysis in the presence of oxygen;
B. * The slowing of glycolysis in the presence of oxygen;
C. The slowing of glycolysis in the absence of oxygen;
D. The accelerating of glycolysis in the presence of 2,6-bisphosphate.
E. The accelerating of glycolysis.
116.
The pentose phosphate pathway has two primary products. They are _________.
A. ATP and NADPH
B. Oxaloacetate and acetyl CoA
C. Sorbitol and fructose
D. * Ribose-5-phosphate and NADPH
E. None of the above
117.
The possible metabolic pathways for pyruvate in anaerobic conditions are:
A. Conversion to lactate or acetyl CoA
B. Conversion to ethanol or acetyl CoA
C. * Conversion to lactate or ethanol
D. Conversion to lactate or ethanol or acetyl CoA.
E. All of the above.
118.
The process of polysaccharides digestion in the gastrointestinal tract is catalized
by:
A. Sucrase
B. Lactase
C. Maltase
D. * D-amilase.
E. None of the above.
119.
The sequence of glucose oxidation to lactate in peripheral tissues, delivery of
lactate to the liver, formation of glucose from lactate in the liver, and delivery of glucose
back to peripheral tissues is known as the ______.
A. Glyoxylate cycle
B. Kreb's cycle
C. * Cori cycle
D. Gluconeogenesis cycle
E. Glycogenolysis
120.
The three control sites in glycolysis are the reactions catalyzed by:
A. Hexokinase, phosphofructokinase 1 and aldolase;
B.
C.
D.
E.
Phosphofructokinase 1, aldolase and pyruvate kinase;
Hexokinase, aldolase and pyruvate kinase;
* Hexokinase, phosphofructokinase 1 and pyruvate kinase.
All of the above
Transfer of a high-energy phosphoryl group to ADP, resulting in ATP occurs
121.
when:
A. 1,3 Bisphosphoglycerate -> 3-phosphoglycerate
B. * Phosphoenolpyruvate (PEP) -> pyruvate
C. 3-Phosphoglycerate -> 2-phosphoglycerate
D. Both a and b
E. Only A
122.
Under starvation conditions, about how long does it take in humans for the body
to deplete the glycogen store in the liver?
A. 10 minutes
B. 4 hours
C. * 24 hours
D. 1 week
E. 2 weeks
123.
Unlike liver tissue, muscle and some other tissues differ in regulation of glycogen
metabolism because
A. The enzymes are different
B. There is no effect of glucagon
C. There is no effect of insulin
D. * Inhibitor-1 controls a phosphatase
E. There is no role for cyclic AMP
124.
Vomiting, diarrhea after consuming milk, enlargement of the liver, jaundice,
cirrhosis,cataracts, retardation of the mental development, elevated blood-galactose level,
absence of the galactose 1-phosphate uridyl transferase in red blood cells are the
symptoms of:
A. Lactose intolerance;
B. Hypolactasia;
C. * Galactosemia;
D. All of the above.
E. Diabetes mellitus
125.
Vomiting, diarrhea after consuming milk, enlargement of the liver, jaundice,
cirrhosis,cataracts, retardation of the mental development, elevated blood-galactose level,
absence of the galactose 1-phosphate uridyl transferase in red blood cells are the
symptoms of:
A. Lactose intolerance;
B. Hypolactasia;
C. * Galactosemia;
D. All of the above.
E. Diabetes mellitus
126.
What is a cause of cataracts in the eye lens of individuals with diabetes?
A. * Accumulation of sorbitol and protein precipitation in the lens.
B. Precipitation of glucose not oxidized by glycolysis in the lens.
C. The absence of membrane transport proteins for pyruvate in the lens cells.
D. Lack of regulation of gluconeogenesis in the lens and the accumulation of
fructose.
E. Lack of regulation of gluconeogenesis in the lens and the accumulation of
maltose.
127.
When a muscle is stimulated to contract aerobically, less lactic acid is formed than
when it contracts anaerobically because:
A. Glycolysis does not occur to significant extent under aerobic conditions.
B. Muscle is metabolically less active under aerobic than anaerobic conditions.
C. The lactic acid generated is rapidly incorporated into lipids under aerobic
conditions.
D. Under aerobic conditions in muscle, the major energy-yielding pathway is the
pentose phosphate pathway, which does not produce lactate.
E. * Under aerobic conditions most of the pyruvate generated as a result of
glycolysis is oxidized by the citric acid cycle rather than reduced to lactate.
128.
Which is not a function of the main products of the pentose phosphate pathway?
A. * To maintain the reduced form of iron in hemoglobin.
B. To provide reducing power for the synthesis of fatty acids.
C. To serve as precursors in the biosynthesis of RNA and DNA.
D. To raise the concentration of cAMP.
E. Produce pentoses
129.
Which of the following compounds cannot serve as the starting material for the
synthesis of glucose via gluconeogenesis?
A. * Acetate
B. Glycerol
C. Lactate
D. Oxaloacetate
E. ?-ketoglutarate
130.
Which of the following is a cofactor in the reaction catalyzed by the glycolytic
enzyme glyceraldehyde 3-phosophate dehydrogenase?
A. NADP
B. * NAD
C. ATP
D. Heme
E. Cu++
131.
Which of the following is not regulated in glycolysis?
A. Pyruvate kinase
B. * Phosphoglycerate kinase
C. Hexokinase
D. PFK-1
E. ATP
132.
Which of the following promotes glucose and amino acid uptake by muscle?
A. Adrenaline
B. * Insulin
C. Glucagon
D. Cortisol
E. Glycogen
133.
Which of the following statements about the pentose phosphate pathway is
correct?
A. It generates 36 mol of ATP per mole of glucose consumed.
B. It generates 6 moles of CO2 for each mole of glucose consumed
C. It is a reductive pathway; it consumes NADH.
D. It is present in plants, but not in animals.
E. * It provides precursors for the synthesis of nucleotides.
134.
Which of the following statements is incorrect?
A. Aerobically, oxidative decarboxylation of pyruvate forms acetate that enters the
citric acid cycle.
B.
C.
D.
E.
In anaerobic muscle, pyruvate is converted to lactate.
In yeast growing anaerobically, pyruvate is converted to ethanol.
Reduction of pyruvate to lactate regenerates a cofactor essential for glycolysis.
* Under anaerobic conditions pyruvate does not form because glycolysis does not
occur.
135.
Which of the following statements is not true concerning glycolysis in anaerobic
muscle?
A. Fructose 1,6-bisphosphatase is one of the enzymes of the pathway
B. * It is an endergonic process.
C. It results in net synthesis of ATP
D. It results in synthesis of NADH
E. Its rate is slowed by a high [ATP]/[ADP] ratio.
136.
Which of the following substrates cannot contribute to net gluconeogenesis in
mammalian liver?
A. Alanine
B. Glutamate
C. * Palmitate
D. Pyruvate
E. ?-ketoglutarate
137.
Which of the following tissues can survive longest in the absence of adequate
oxygen (anaerobic conditions) by deriving energy from the anaerobic respiration of
glucose molecules?
A. The brain
B. * Cardiac muscle
C. The kidneys
D. Skeletal muscle
E. None of the above
138.
Which one of the following statements about gluconeogenesis is false?
A. For starting materials, it can use carbon skeletons derived from certain amino
acids.
B. * It consists entirely of the reactions of glycolysis, operating in the reverse
direction.
C. It employs the enzyme glucose 6-phosphatase.
D. It is one of the ways that mammals maintain normal blood glucose levels between
meals.
E. It requires metabolic energy (ATP or GTP)
139.
. A deficiency in the synthesis of sphingomyelins or cerebrosides will most likely
result in the proper formation of
A. Cell surfaces
B. Cell to cell communication
C. * Nerve cells
D. Blood groups
E. All of the above
140.
A diet containing this fat is helpful in lowering the blood cholesterol level.
A. * Unsaturated
B. Saturated
C. Vitamin enriched
D. Refined
E. All of the above
141.
A fatty acid not synthesized in man is
A. Oleic
B. Palmitic
C. * Linoleic
D. Stearic
142.
A lipid containing alcoholic amine residue is
A. Phosphatidic acid
B. Ganglioside
C. Glucocerebroside
D. * Sphingomyelin
E. All of the above
143.
A soluble system for synthesis of fatty acids have been isolated from avian liver,
required for the formation of long chain fatty acids by this system is
A. ATP
B. Acetyl CoA
C. NADPH
D. * All of these
E. Both (A) and (B)
144.
Absorption of higher fatty acids in the intestine is performed with the help of
mechanism:
A. simple diffusion
B. facillated diffusion
C. primary active transport
D. pinocytosis as choleic complex
E. secondary active transport
145.
Activation of fatty acids requires all the following except
A. ATP
B. Coenzyme A
C. Thiokinase
D. * Carnitine
E. A and B
146.
Acyl Carrier Protein contains the vitamin:
A. Biotin
B. Lipoic acid
C. * Pantothenic acid
D. Folic acid
E. Thiamine
147.
Acyl carrier protein is involved in the synthesis of
A. protein
B. glycogen
C. * fatty acid outside the mitochondria
D. fatty acid in the mitochondria
E. Both C and D
148.
Acyl-CoA dehydrogenase converts Acyl CoA to ? ??-unsaturated acyl-CoA in
presence of the coenzyme:
A. NAD+
B. NADP+
C. ATP
D. * FAD
E. TPP
149.
Acylsphingosine is also known as
A. Sphingomyelin
B. * Ceramide
C. Cerebroside
D. Sulphatide
E. None of these
150.
Adipose tissue lacks
A. Hormone-sensitive lipase
B. * Glycerol kinase
C. cAMP-dependent protein kinase
D. Glycerol-3-phosphate dehydrogenase
E. B and C
151.
Adipose tissue which is a store house for triacyl glycerol synthesis the same using
A. The glycerol released by hydrolysis of triacyl glycerol
B. * The glycerol-3-phosphate obtained in the metabolism of glucose
C. 2-phosphoglycerate
D. 3-phosphoglycerate
E. All of the above
152.
After having fatty food the patient had nausea and steatorrhea. The cause of this
condition can be:
A. amilase insufficiency
B. increase of lipase excretion
C. trypsin synthesis malfunction
D. * lack of bile acids
E. increased acidity of gastric juice
153.
After having fatty food the patient had nausea and steatorrhea. The cause of this
condition can be:
A. amilase insufficiency
B. increase of lipase excretion
C. trypsin synthesis malfunction
D. * lack of bile acids
E. increased acidity of gastric juice
154.
After meal conncentration of chylomicrones in blood of the patient is increased.
Chylomicrones are formed in the ____________________ and their main function is the
__________________________________.
A. wall of intestine; transport of cholesterol;
B. liver; transport of triacylglycerols synthesized in liver;
C. * wall of intestine; transport of dietary triacylglycerols;
D. blood; transport of cholesterol.
E. all of the above.
155.
All long chain fatty acids with even number of carbon atoms are oxidized to a
pool of _________ by ?-oxidation.
A. CO2
B. Propionic acid
C. Acetic acid
D. * Acetyl CoA
E. CoA
156.
An unknown lipid is treated with a mixture of phospholipases A1, A2, C and D.
Since no glycerol is formed after this treatment, the lipid is most likely
A. Phosphatidylethanolamine
B. Phosphatidylcholine
C. Plasmologen
D. * Ceramide
E. A mixture of a and b
157.
can be generated from
A. * The pentose phosphate pathway
B. Glycolysis
C. The citric acid cycle
D. Mitochondrial malate dehydrogenase
E. Citrate lyase
158.
Carboxylation of acetyl—CoA to malonyl — CoA takes place in presence of
A. FAD+
B. Biotin
C. * NAD+
D. NADP+
E. TPP
159.
Carnitine acylcarnitine translocase is present
A. * In the inner mitochondrial membrane
B. In the mitochondrial matrix
C. On the outer surface of inner mitochondrial membrane
D. On the inner surface of inner mitochondrial membrane
E. In cytoplasm
160.
Carnitine is synthesized from
A. * Lysine and methionine
B. Glycine and arginine
C. Aspartate and glutamate
D. Proline and hydroxyproline
E. None of the above
161.
Cerebrosides contain all the following except
A. Galactose
B. * Sulphate
C. Sphingosine
D. Fatty acid
E. B and C
162.
Chylomicron remnants are catabolised in
A. Intestine
B. Adipose tissue
C. * Liver
D. Liver and intestine
E. Pancreas
163.
Co-lipase is a
A. Bile salt
B. Vitamin
C. * Protein
D. Phospholipid
E. Lipoprotein
164.
Concentration of phospholipids in blood of the patient is decreased. Lecithins are
composed of
A. * Glycerol + Fatty acids + Phosphoric acid + Choline
B. Glycerol + Fatty acids + Phosphoric acid + Ethanolamine
C. Glycerol + Fatty acids + Phosphoric acid + Serine
D. Glycerol + Fatty acids + Phosphoric acid + Beaine
E. All of these
165.
Conncentration of LDL in blood of the patient is increased. Low density
lipoproteins transport ________________ from __________ to _________________.
A. triacylglycerols; intestine; liver
B. * cholesterol; liver; peripheral tissues;
C. cholesterol; peripheral tissues; liver
D. triacylglycerols; liver; adipose tissue and muscles
E. triacylglycerols; liver; muscles
166.
De novo synthesis of fatty acids is catalysed by a multi-enzyme complex which
contains
A. One-SH group
B. Two-SH groups
C. Three-SH groups
D. * Four-SH groups
E. Five –SH groups
167.
De novo synthesis of fatty acids occurs in
A. * Cytosol
B. Mitochondria
C. Microsomes
D. EPR
E. All of these
168.
De novo synthesis of fatty acids requires all of the following except
A. Biotin
B. NADH
C. Panthothenic acid
D. * ATP
E. Multienzyme complex
169.
Dietary fats after absorption appear in the circulation as
A. HDL
B. VLDL
C. LDL
D. * Chylomicron
E. None of the above
170.
Each of the following is a principle hormonal regulator of fatty acid metabolism,
except:
A. Glucagon
B. Epinephrine
C. Insulin
D. All of the above
E. * None of the above
171.
Elevated levels of the hormone ________ stimulates the conversion of
triacylglycerols stored in adipose cells to free fatty acids and monoacylglycerols to
provide energy when carbohydrate stores are depleted.
A. insulin
B. testosteron
C. * epinephrine
D. ergosterol
E. STH
172.
Ethanolamine, serine and choline can be cleaved from glycerophospholipids by
treatment with
A. Phospholipase A1
B. Phospholipase A2
C. Phospholipase B
D. Phospholipase C
E. * Phospholipase D
173.
Extramitochondrial synthesis of fatty acids occurs in
A. Mammary glands
B. Lungs
C. Brain
D. * All of these
E. B and c
174.
Fat depots are located in
A. Intermuscular connective tissue
B. Mesentary
C. Omentum
D. * All of these
E. A and C
175.
Fatty acid synthesis takes place in the presence of the coenzyme:
A. NAD+
B. Reduced NAD
C. NADP+
D. * Reduced NADP
E. TPP
176.
Fatty acids can not be converted into carbohydrates in the body, as the following
reaction is not possible:
A. Conversion of glucose-6-phosphate into glucose
B. Fructose 1, 6 diphosphate to fructose-6-phosphate
C. * Transformation of acetyl CoA to pyruvate
D. Formation of acetyl CoA from fatty acids
E. Both C and D
177.
For TAG synthesis glycerol converts into active form. Glycerol is transformed
into active form with the enzyme:
A. phosphatidate phosphatase
B. * glycerol kinase
C. glycerol phosphate dehydrogenase
D. glycerol phosphate acyltransferase
E. diglyceride-acyl-transferase
178.
For the patient was prescribed bile preparation for improvement of fatty food
digestion. Which components of the preparation participate in fat emulsification?
A. * bile acid salt
B. fatty acids
C. cholesterol and it's esters
D. bilirubin monoglucuronides
E. diglycerides
179.
For the patient with diagnosis atherosclerosis “Linetol” is prescribed, which
contains essential fatty acids. Which of the below-mentioned acids surely is a part of it?
A. stearic acid
B. palmitic acid
C. * linoleic acid
D. oleic acid
E. crotonic acid
180.
For the patient with diagnosis atherosclerosis “Linetol” is prescribed, which
contains essential fatty acids. Which of the below-mentioned acids surely is a part of it?
A. stearic acid
B. palmitic acid
C. * linoleic acid
D. oleic acid
E. crotonic acid
181.
Free glycerol cannot be used for triglyceride synthesis in
A. Liver
B. Kidney
C. Intestine
D. * Adipose tissue
E. Pancreas
182.
Gangliosides are complex glycosphingolipids found in
A. Liver
B. * Brain
C. Kidney
D. Muscle
E. Pancreas
183.
Glycerol is converted into glycerol-3-phosphate by
A. Thiokinase
B. Triokinase
C. * Glycerol kinase
D. Glycerol phosphatase
E. All of these
184.
Glycerol is converted to ___________ when it is used for gluconeogenesis.
A. dihydroxyacetone phosphate
B. phosphoenolpyruvate
C. oxaloacetate
D. * 3-phosphoglycerate
E. none of the above.
185.
Glycerol is transformed into active form with the help of enzyme:
A. phosphatidate phosphatase
B. * glycerol kinase
C. glycerol phosphate dehydrogenase
D. glycerol phosphate acyltransferase
E. diglyceride-acyl-transferase
186.
In a liver of 55 years old patient fatty acids synthesis is active. When the liver is
actively synthesizing fatty acids, a concomitant decrease in ? oxidation of fatty acids is
due to
A. * Inhibition of a translocation between cellular compartments
B. Inhibition by an end product
C. Activation of an enzyme
D. Detergent effects
E. Decreases in adipocyte lipolysis
187.
in adipose tissue
A. 2 mol of triacylglycerides is released
B. 2 mol of free fatty acids is released
C. * 1 mol of glucose can be synthesized in gluconeogenesis
D. 1 mol of triacylglyceride is released
E. 3 mol of acyl CoA is produced
188.
In an organism of 37 years old patient fatty acids synthesis is active. Which of the
following is a true statement for fatty acid synthesis?
A. It occurs in the mitochondria.
B. The reducing power for synthesis is supplied by NAD and ubiquinone.
C. Both a and b.
D. None of the above.
E. * The reducing power for synthesis is supplied by NADPH+
189.
In an organism of 40 years old patient fatty acids synthesis is active. The main
sources of NADPH for fatty acid biosynthesis is:
A. TCA cycle
B. oxidative phosphorylation
C. * the pentose phosphate pathway
D. glycolysis
E. All of the above.
190.
In an organism of 40 years old patient mobilisations of lipids is active. Elevated
levels of the hormone ________ stimulates the conversion of triacylglycerols stored in
adipose cells to free fatty acids and monoacylglycerols to provide energy when
carbohydrate stores are depleted.
A. insulin
B. glucagon
C. * epinephrine
D. ergosterol
E. somatotropin
191.
In an organism of 43 years old patient mobilisations of lipids and fatty acids
oxidation are active. How many cycles of ?-oxidation are required to completely process
a saturated C18 fatty acid?
A. 6
B. * 8
C. 9
D. 18
E. 7
192.
In an organism of 47 years old patient mobilisations of lipids and fatty acids
oxidation are active. How many QH2 and NADH are produced by one round of the ?oxidation pathway?
A. * 1 each
B. 1 QH2 and 2 NADH
C. 2 each
D. 2 QH2 and 1 NADH
E. all of the above
193.
In an organism of 47 years old patient mobilisation of lipids is active.
Tryacylglycerinlipase of fatty tissue is activated by:
A. adenosine triphosphate
B. guanosine triphosphate
C. adenosine diphosphate
D. * cyclic adenosine monophosphate
E. guanosine diphosphate
194.
In an organism of 55 years old patient with hyperfunction of pancreas takes plase
active synthesis of fatty acids Which of the following is the regulated step of fatty acid
synthesis in eukaryotes?
A. * Carboxylation of acetyl CoA.
B. Transportation of mitochondrial acetyl CoA into the cytosol.
C. Assembly of the fatty acid chain.
D. All of the above.
E. None of the above.
195.
In an organism of 55 years old patient fatty acids oxidation is active. Splitting off
acetyl-CoA in the final reaction of S-oxidation of fatty acids is performed by the enzyme:
A. dehydrogenase
B. enolase
C. hydrase
D. lipase
E. * thiolase (acetyl-CoA-acyltransferase)
196.
In an organism of 55 years old patient mobilisations of lipids and fatty acids
oxidation are active. Which of the below-mentioned metabolites is formed by oxidation
of fatty acids with odd amount of carbon atoms:
A. * propionyl-CoA
B. butyryl-CoA
C. malonyl-CoA
D. enoil-CoA
E. acetacetyl-CoA
197.
?In the blood of a patient is decreased amount of phospholipinds, increased
concentration of cholesterol, observed the symptoms of prostaglandin lack. What can
cause such events?
A. limited usage of carbohydrates
B. * limited usage of polyunsaturated fatty acids
C. D hypovitaminosis
D. excess of lipids in food
E. excess of carbohydrates in food
198.
In the blood of a patient is decreased amount of phospholipinds, increased
concentration of cholesterol, observed the symptoms of prostaglandin lack. What could
cause such events?
A. limited usage of carbohydrate
B. * limited usage of polyunsaturated fatty acids
C. D hypovitaminosis
D. excess of lipids in food
E. excess of carbohydrates in food
199.
In the blood of a patient is decreased amount of phospholipinds, increased
concentration of cholesterol, observed the symptoms of prostaglandin lack. What can
cause such events?
A. limited usage of carbohydrates
B. * limited usage of polyunsaturated fatty acids
C. D hypovitaminosis
D. excess of lipids in food
E. excess of carbohydrates in food
200.
In the blood of a patient there are decreased amount of phospholipids, increased
concentration of cholesterol, observed prostaglandins lack. What can cause such events?
A. limited usage of carbohydrates
B. * limited usage of polyunsaturated fatty acids
C. D hypovitaminosis
D. excess of lipids in food
E. excess of carbohydrates in food
201.
In the blood of a patient there is decreased amount of phospholipids, increased
concentration of cholesterol, observed prostaglandins lack. What can cause such events?
A. limited usage of carbohydrate
B. * limited usage of polyunsaturated fatty acids
C. D hypovitaminosis
D. excess of lipids in food
E. excess of carbohydrates in food
202.
is conjugated, reacts directly, and is a major component of bile?
A. * Bilirubin diglucuronide
B. Stercobilin
C. Biliverdin
D. Urobilinogen
E. Heme
203.
is most often due to
A. Lactose intolerance
B. Glycogen storage diseases
C. Antibody abnormalities
D. * Deficiency of fatty acid desaturase greater than
E. Deficiency of chylomicron and VLDL production
204.
Lisophospholipids, formed after action of phospholipase A2, have the following
feature:
A. activate lipase
B. * hemolytic
C. form conjugates with bile acids
D. participate in phospholipids resynthesis
E. accompany absorption of fatty acids
205.
mevastatin therapy. Which of the following will be a result of this therapy?
A. Low blood glucose
B. * Low blood LDLs
C. High blood cholesterol
D. High blood glucose
E. Low oxidation of fatty acids
206.
Patient suffers from disorders of lipid digestion. What are the functions of bile
salts in lipid digestion?
A. * emulsification of lipids; activation of lipase;
B. emulsification of lipids; decompose the cholesteryl esters;
C. activation of phospholipase A2; activation of cholesterol esterase;
D. inhibition of lipase; inhibition of phospholipase A2.
E. all of the above.
207.
The amount of phospholipids in the blood plasma of 45 years old patient is
normal. Choose correct result:
A. * 1,5-3,6 mmol/l
B. 2,0-5,0 mmol/l
C. 1,2-2,5 mmol/l
D. 0,7-2,0 mmol/l
E. 7,0-8,0 mmol/l
208.
The patient was prescribed bile preparation for improvement of fatty food
digestion. Which components of the preparation participate in fat emulsification?
A. * bile acid salt
B. fatty acids
C. cholesterol and it's esters
D. bilirubin monoglucuronides
E. diglycerides
209.
Why snake venom causes the lysis of erythrocytes?
A. contains hemolytic toxins which directly destroy the erythrocytes membranes;
B. * contains phospholipase A2 catalyzing the hydrolysis of glycerophospholipids
and formation of lysophosphoglycerides which can act as detergents and disrupt
cellular membranes;
C. contains lipase catalyzing the hydrolysis of triacylglycerols in the cellular
membranes;
D. contains cholesteryl esterase catalyzing the hydrolysis of cholesterol esters in the
cellular membranes.
E. none of the above.
210.
When can the production of ketone bodies occur? Which is/are true?
A. If you eat too much sugar and less of saturated butter
B.
C.
D.
E.
In diabetes patients
If you eat too much bacon and less sugar
* If you are starving for a longer time period
In pellagra patients
211.
A high carbohydrate, low-fat diet will:
A. * Promote the synthesis of fatty acids by inducing the synthesis of acetyl CoA
carboxylase.
B. Favor the oxidation of stored fatty acids by inducing the synthesis of lipoprotein
lipase.
C. Promote the synthesis of fatty acids by repressing synthesis of carnitine-palmitoyl
transferase.
D. Favor the oxidation of fatty acids by repressing the synthesis of fatty acid
synthase.
E. None of the above
212.
A man of 47 years suffers from insulin-dependent diabetes, on which background
has evolved metabolic acidosis. What process' malfunction majorily helps the
development of acidosis?
A. Ketone bodies synthesis
B. * Ketone bodies utilization
C. Glycogen decomposition
D. Glycogen synthesis
E. Oxidative phosphorylation
213.
A patient, which visited a doctor, has increased level of glucose in blood and
urine. Suspected is diabetes mellitus. Which changes in lipid metabolism can cause this
disease?
A. Hyperketonemia
B. Hypercholesterolemia
C. * Hypoketonemia, ketonuria
D. Hyperphospholipidemia, hyperketonemia
E. Hypophospholipidemia , hypoketonemia
214.
A stroke is similar to a heart attack in all of the following ways EXCEPT:
A. It affects the brain.
B. The risk can be reduced by lifestyle changes.
C. It is a blockage of major arteries.
D. * It results in the death of cells
E. A, B, C
215.
Adiposogenital dystrophy is caused by the lack of secretion of:
A. Oxytocin
B. Vasopressin
C. Gonadotropic hormone
D. * Thyrotropic hormone
E. Melanotropic hormone
216.
After observation of a patient with diabetes mellitus were discovered ketonemia
and ketonuria. Mark, which of the below-mentioned matters is the precussor of ketone
bodies?
A. * Acetyl-CoA
B. Oxaloacetate
C. ?-ketoglutarate
D. Cholesterol
E. Unsaturated fatty acids
217.
All of the following are functions of dietary fat, except
A. Facilitating the absorption of fat-soluble vitamins.
B. Serving as the major source of fuel for RBCs.
C. Acting as stored energy.
D. * Being essential to the synthesis of phospholipid that is a major component of
cell membranes.
E. Facilitating the absorption of water-soluble vitamins.
218.
All of the following are true about arteriosclerosis EXCEPT:
A. It is a collection of fatty deposits inside a blood vessel.
B. It is called "hardening" of the arteries.
C. It is a blockage of an artery.
D. * It affects all blood vessels equally
E. Only A, B, C
219.
All the following statements about carnitine are true except
A. It can be synthesised in the human body
B. It can be synthesized from methionine and lysine
C. It is required for transport of short chain fatty acids into mitochondria
D. * Its deficiency can occur due to haemodialysis
E. can be synthesized from treonin and lysine
220.
All the following statements about ketone bodies are true except
A. * Their synthesis increases in diabetes mellitus
B. They are synthesized in mitchondria
C. They can deplete the alkali reserve
D. They can be oxidized in the liver
E. Only C
221.
Anti-inflammatory corticosteroids inhibit
A. Phospholipase A1
B. * Phospholipase A2
C. Cyclo-oxygenase
D. Lipo-oxygenase
E. Pancreatic lipase
222.
Atherosclerosis and coronary heart diseases are associated with the diet:
A. * High in total fat and saturated fat
B. Low in protein
C. High in protein
D. High in carbohydrate
E. High in vitamins
223.
Bile acids are excreted into the intestine where they become bile salts due to:
A. * The alkaline environment within the intestine
B. The acidic environment of the stomach
C. Alkaline hydrolysis
D. Formation of esters
E. Formation of amides due to the relatively high temperature and low rate of flow
224.
By analysis of patient's lipogram was discovered the increase of some lipoproteins
level. Choose the most atherogenic representative of lipoproteins:
A. LDL
B. * HDL
C. Chylomicrons
D. LDDL
E. Albumins
225.
?Cardiovascular function is most commonly impaired by which disease?
A. Coronary heart disease
B. Arteriosclerosis
C. Stroke
D. Hypertension
E. * All of the above
226.
Cholesterol circulates in blood stream chiefly as
A. Free cholesterol
B. Ester cholesterol
C. Low density lipoproteins
D. * Low density lipoproteins and high density lipoproteins
E. Salts of cholesterol
227.
Doctor prescribed to the patient, exhausted after heavy disease, small doses of
hormones on the background of high-calorie diet. What hormone was prescribed?
A. * Adrenalin
B. Insulin
C. Thyroxine
D. Aldosterone
E. Vasopressin
228.
Elder patient has a recommendation with the aim of prevention liver fatty seepage
to use in food cheese. What essential amino acid, necessary for phospholipids synthesis,
there is in cheese?
A. Arginine
B. Proline
C. Valine
D. Lysine
E. * Methionine
229.
Fat is important to the body for all of the following reasons EXCEPT
A. That it provides essential amino acids.
B. That it stores energy for later use.
C. * That it carries fat-soluble vitamins.
D. That it helps to maintain cell membranes
E. Only B and D
230.
Fatty acid oxidation occurs:
A. Only in adipose tissue
B. * Within liver parenchyma
C. Within mitochondria and peroxisomes of many tissues
D. Only within arterial epithelium
E. Only within venous epithelium
231.
Fatty acid oxidation occurs:
A. Only in adipose tissue
B. * Within liver parenchyma
C. Within mitochondria and peroxisomes of many tissues
D. Only within arterial epithelium
E. Only within venous epithelium
232.
Hyperketonemia is observed in the following cases, but:
A. Starvation
B. Diabetes mellitus
C. Excessive usage of carbohydrates
D. Long stress
E. * Thyrotoxicosis
233.
Hypocholesterolaemia can occur in
A. * Hyperthyroidism
B. Nephrotic syndrome
C. Obstructive jaundice
D. Diabetes mellitus
E. Diabets insipidus
234.
If you wanted to locate some cholesterol within a healthy body, you might expect
to find it:
A. * In the cytoplasm of mitochondria
B. In the cytoplasm of chloroplasts
C. In membranes
D. In the aqueous humor of the eye
E. In pancreatic lipase
235.
Ketone bodies can be used as the source of energy in the following organ(s):
A. * In the brain (after a week of fasting it adapts to use it)
B. In the heart
C. In the liver
D. In kidney
E. None of the above
236.
Obesity generally reflects excess intake of energy and is often associated with the
development of
A. Nervousness
B. * Non-insulin dependent diabetes mellitus
C. Hepatitis
D. Colon cancer
E. Rickets
237.
Obesity increases the risk of
A. Hypertension
B. Diabetes mellitus
C. Ishemic disease
D. * All of these
E. Srtoke
238.
Obesity is accumulation of _______ in the body.
A. Water
B. NaCl
C. * Fat
D. Proteins
E. Vitamins
239.
Oliguria can occur in
A. Diabetes mellitus
B. * Diabetes insipidus
C. Acute glomerulonephritis
D. Chronic glomerulonephritis
E. Liver diseases
240.
One of the leading pathogenetic chains in the development of radial pathology is
the intensification of free-radical oxidation processes. What matters are the main source
of free radicals formation?
A. Water
B. * Lipids
C. Carbohydrates
D. Proteins
E. Metal ions
241.
Patient has ketonemia and ketonuria, amount of glucose in blood 3,5 mM/l,
cholesterol – 4,5 mM/l. What malfunctions could cause this?
A. * Dabetes mellitus
B. Crbohydrates deficit in nutrition
C. Aherosclerosis
D. Ecessive usage of fats
E. Liver disease
242.
Patient with hard form of diabetes has increase of ketone bodies amount in blood.
The main reason of ketonemia development is considered:
A. * Decrease if intracellular oxaloacetate concentration
B. Inhibition of triacylglycerol lipase
C. Activity inhibition of acetyl-CoA-acetyltransferase
D. Activation of Krebs cycle enzymes
E. Stimulation of glycogen synthesis in liver
243.
Smoking increases an individual's chance for heart disease by:
A. Decreasing your body's ability to clot blood.
B. Decreasing the amount of fat deposits around arterial walls
C. Increasing the chance of an irregular heartbeat.
D. * Decreasing heart rate and blood pressure.
E. None of the above
244.
Splitting off acetyl-CoA in the final reaction of S-oxidation of fatty acids is
performed under influence of enzyme:
A. Dehydrogenase
B. Enolase
C. Hydrase
D. Lipase
E. * Thiolase (acetyl-CoA-acyltransferase)
245.
The amount of total lipids in blood – 30 g/l, cholesterol – 9 mmol/l. The blood
plasma is alkaline, by blood centrifugation on the surface is formed white layer. Amount
increase of what transportation forms of lipids in the blood takes place in this situation?
A. Chylomicrons
B. ?-lipoproteins
C. * C-lipoproteins
D. Pre-D-lipoproteins
E. Albumins
246.
The average male human body contains about 19% fat. For purposes of
discussion, physicians often refer to the average man as a "70 Kg man," (actually a little
low by modern standards of obesity). How many pounds of fat would be present in the
body of a 70 Kg man?
A. 13
B. * 51
C. 154
D. 2,2
E. 29
247.
The body needs fatty acids to:
A. Produce cell membranes.
B. Make myelin sheaths.
C. Absorb certain vitamins.
D. * All of the above
E. Precusor of carbohydrates, amino acids
248.
The observation of a patient after radiolesion showed increased amount in the
blood of malonic dialdehyde, which proves the activation of lipids peroxidation. What
violations in cell membranes this can cause?
A. Cholesterol oxidation
B. Transportation proteins structure change
C. Destruction of carbohydrate components
D. Activation of Na+, K+-ATP-ase
E. * Destruction of phospholipids
249.
The observation of a patient showed deficient activity of ferment lecithin
cholesterol acyltransferase (LCAT). Possible result is:
A. Fatty liver degeneration
B. Hyperketonemia
C. * Hypocholesterolemia
D. Hyperlipemia
E. Atherogenic hypocholesterolemia
250.
The observation of a patient showed high amount of cholesterol in ?-lipoprotein
fraction. This can result in:
A. * Ischemic heart disease
B. Hypovitaminosis
C. Ketonemia
D. Steatorrhea
E. Fatty liver degeneration
251.
The observation of a patient showed symptoms of hypovitaminosis B5. By this is
observed malfunction of lipids metabolism, but:
A. Cholesterol and fatty acids synthesis
B. Ketone bodies oxidation
C. * Transportat of fatty acids to mitochondria
D. Glycerine oxidation
E. Fatty acids oxidation
252.
The patient got into the habit of having several uncooked eggs, which contain
antivitamin of biotin – avidin. What malfunctions of lipid metabolism can be the result of
this?
A. Cholesterol biosynthesis
B. Lipids absorption
C. * Fatty acids biosynthesis
D. Glycerine oxidation
E. Lipids transportation in blood
253.
The patient has hypertension, atherosclerotic vessels affection. Name what lipid
usage he needs to decrease in daily food allowance:
A. Cholesterol
B. * Oleic acid
C. Lecithin
D. Mono oleate glyceride
E. Phosphatidyl serine
254.
The patient have symptoms of general adiposity, atherosclerotic changes. From
the list of possible reasons for this should be excluded:
A. Lack of carbohydrates in nutrition
B. Smoking
C. Alcohol abuse
D. Hereditary inclination
E. * Excess of carbohydrates and lipids in nutrition
255.
The patient with burns was prescribed in the treatment preparation complex
vitamin E. What is the base of its action by this pathology?
A. Activation of tissue breathing
B. Normalization of metabolism in muscles
C. * Anti-oxidant action
D. Activation of lipids peroxidation
E. Decrease of organism dehydration
256.
The paсient possibly has atherosclerosis. What feature helped to prove this?
A.
B.
C.
D.
E.
Increase of general lipids in blood
Increase of ketone bodies in blood
Increase of high density liporoteins
Decrease of phospholipids in blood
Increase of low density liporoteins
257.
The vitamin whose cofactors are involved in carrying single carbons for “onecarbon metabolism” is:
A. * Folate or folic acid
B. Nicotinamid
C. Biotin
D. Ascorbic acid
E. 1,25-dihydroxicholecalciferol
258.
To prevent the development of atherosclerosis is possible by means of:
A. * Excessive usage of carbohydrates
B. Limitation of carbohydrates, lipids
C. Excess of lipids in food
D. Limitation of protein in food
E. Limitation of vegetable oil in food
259.
Which food does not have a lot of saturated fat?
A. * Coconut oil.
B. Whole milk.
C. Chicken, with skin.
D. Butter.
E. Whole wheat bread.
260.
Which of the following can be synthesized in the human body if precurors are
available?
A. Oleic acid
B. Palmitoleic acid
C. Arachidonic acid
D. * All of these
E. None of the above
261.
Which of the following condition is characterized by ketonuria but without
glycosuria?
A. Diabetes mellitus
B. Diabetes insipidus
C. * Prolonged starvation
D. Addison’s disease
E. Pellagra
262.
Which of the following factors makes it harder for an obese person to lose
weight?
A. * An increase in metabolism that increases the appetite
B. An increase in heat loss from the body that requires more food intake
C. An increase in taste receptors
D. An increase in thyroxine levels as fat levels accumulate
E. An increase in alpha receptors (the kind that favor fat accumulation) in fat cells
263.
Which of the following major risk factors of heart disease cannot be controlled?
A. * Smoking
B. Hypertension
C. Sedentary lifestyle
D. Heredity
E. All of the above
264.
Which of the following statements is NOT true regarding fatty acids?
A.
B.
C.
D.
E.
Fatty acids dissolve in nonpolar solvents.
* Triglycerides are esters of fatty acids.
Most naturally occurring fatty acids have trans double bonds.
Fatty acids are biosynthesized from acetate.
C and D
265.
Which of these is a general feature of the lipid bilayer in all biological
membranes?
A. Polar, but uncharged, compounds readily diffuse across the bilayer.
B. Individual lipid molecules are free to diffuse laterally in the surface of the bilayer.
C. Individual lipid molecules in one face (monolayer) of the bilayer readily diffuse
(flip-flop) to the other monolayer.
D. * The bilayer is stabilized by covalent bonds between neighboring phospholipid
molecules.
E. Only C
266.
Which one of the following statements is FALSE?
A. Skeletal muscle and the brain lack glucose-6-phosphatase.
B. Acetoacetate and ?-hydroxybutyrate serve as the major source of energy for the
brain during starvation.
C. * Water is an essential nutrient.
D. The essential polyunsaturated fatty acids are palmitoleic and linoleic acids.
E. The principal function of water-soluble vitamins is to serve as coenzymes for
several types of enzymes.
267.
Women of 45 years after long starvation has hyperketonemia. Where is performed
the synthesis of ketone bodies?
A. * Mitochondria of hepatocyte
B. Cytoplasm of hepatocyte
C. Mitochindria of cardiomyocytes
D. Nucleus of cardiomyocytes
E. Cytoplasm of adipocytes
268.
Worker of dry-cleaner's has fatty liver degeneration. What matter synthesis
malfunction in the liver can cause such pathology?
A. Urea
B. * Phosphatidylcholine
C. Phosphatidic acid
D. Cholic acid
E. Tristearin
269.
?-Oxidation was discovered by Knoop when he fed dogs fatty acids labeled with a
benzene ring at the end of the molecule opposite the -COOH group. Metabolism could
not proceed beyond the phenyl group. Dogs fed phenylbutyric acid would produce urine
containing:
A. Phenylacetic acid
B. Benzoic acid
C. * Phenylpyruvic acid
D. Phenyllactic acid
E. Benzene
270.
The amount of total acidity in gastric juice of a patient is normal. Which of the
listed numbers is true in this case?
A. A.30-40 mM/l
B. * B.40-60 mM/l
C. 20-30 mM/l
D. 30-50 mM/l
E. 20-40 mM/l
271.
A 60 year old patient has hyperplasia of G-cells of antral part of stomach. What
changes in gastric juice are most possible for this pathology?
A. * Hyperchlorhydria
B. Hypochlorhydria
C. Achlorhydria
D. Achylia
E. Presence of lactic acid
272.
A healthy 70 kg man eats a well balanced diet containing adequate calories and
62.5 g of high quality protein per day. Measured in grams of nitrogen, his daily nitrogen
balance would be
A. +10 g
B. +6.25 g
C. * 0 g
D. –6.25 g
E. None of the above
273.
A lipotropic factor was prescribed for the patient with liter cirrhosis. Which amino
acid is a lipotropic factor?
A. Lysine
B. Leucine
C. Tryptophan
D. * Methionine
E. All of these
274.
A patient suffers from intensive decay of proteins. Name the compound in the
urine, which is a test of protein putrefaction processes in the intestine:
A. Urates
B. Urea
C. Creatinine
D. * Indican
E. Lactic acid
275.
A phenylketonuria of the newborn was diagnosed. All the following are true about
phenylketonuria except
A. Deficiency of phenylalanine hydroxylase
B. Mental retardation
C. * Increased urinary excretion of p-hydroxyphenyl pyruvic acid
D. Decrease serotonin formation
E. C and D
276.
After burns cause the development of negative nitric balance of a 40 year old
patient took place that was the result of increase of:
A. Glycolysis
B. Lipolysis
C. * Proteolysis
D. Phosphorolysis
E. Fibrinolysis
277.
By decarboxylation of histidine in an organism of 9 year old child in the state of
allergic shock is (are) formed:
A. Tyramine
B. * Histamine
C. Putrescine
D. Cadaverine
E. Serotonine
278.
Coenzyme of amino acids decarboxylases is:
A. Thymidine diphosphate
B.
C.
D.
E.
Favin adenine dinucleotide
Favin mononucleotide
HS-CoA
* PALP
279.
essential amino acid?
A. Alanine
B. Glycine
C. Tyrosine
D. * Tryptophan
E. Both A and D
280.
GABA (gamma aminobutyric acid), which belongs to inhibiting mediators of
central nervous system, is a product of glutamic decarboxylation. What vitamin
prescription is appropriate by convulsive state, caused by decrease if GABA formation?
A. B1
B. B9
C. * B6
D. B5
E. B2
281.
Into the hospital was carried 37 year old man with symptoms of achylia gastrica.
Achylia gastrica is said to be when absence of
A. Pepsin only
B. * Both pepsin and HCl
C. HCl only
D. Bile
E. None of these
282.
Into the hospital was carried 7 years old child in the state of allergic shock, which
evolved after wasp's bite. In the blood increased concentration of histamine. This amine
is formed as a result of reaction of:
A. Dehydration
B. Deamination
C. Reduction
D. * Decarboxylation
E. Hydrooxidation
283.
N-end amino acids in the proteins splitt off:
A. Dipeptidase
B. Carbooxypeptidase
C. * Aminopeptidase
D. Elastase
E. Endopeptidase
284.
Patient has positive nitric balance. The reason of this can be:
A. Starvation
B. Tuberculosis
C. * Pregnancy
D. Malignant neoplasms
E. AIDS
285.
Patient has the diagnosis “malignant carcinoid”, amount of serotonine in blood
increased greatly. Name amino acid, able to form the given biogenic amine:
A. Alanine
B. * 5-oxytryptophane
C. Leucine
D. Threonine
E. Methionine
286.
Patient on a vegetarian diet has negative nitric balance, hyperproteinemia,
affection of colloid-osmotic pressure and water-salt metabolism by normal function of
alimentary canal. This was caused by:
A. * Monotonous protein nutrition
B. Monotonous carbohydrate nutrition
C. Lack of unsaturated fatty acids
D. Lack of phospholipids in food
E. Lack of vitamins in food
287.
?Patient with enterocolitis has affected protein adoption. What mechanism of
amino acids absorption is affected?
A. Pinocytosis
B. Primary active transport
C. Simple diffusion
D. * Secondary sodium-dependent transport
E. Phagocytosis
288.
The total acidity in gastric juice of a 40 year old patient is high. Which of the
listed numbers is true in this case?
A. 30-40 mM/l
B. * 60-80 mM/l
C. 20-30 mM/l
D. 30-50 mM/l
E. 20-40 mM/l
289.
The 45 year old man suffers from stomach ache due to high acidity of gastric
juice. The pH of gastric juice becomes low in
A. Hemolytic anemia
B. * Pernicious anemia
C. Both (A) and (B)
D. Jaundice
E. None of these
290.
The 75 year old man suffers from negative nitrogenous balance due to lack of
essential amino acids. The essential amino acids
A. Must be supplied in the diet because the organism has lost the capacity to aminate
the corresponding ketoacids
B. * Must be supplied in the diet because the human has an impaired ability to
synthesize the carbon chain of the corresponding ketoacids
C. Are identical in all species studied
D. Are defined as those amino acids which cannot be synthesized by the organism at
a rate adequate to meet metabolic requirements
E. Both B and D
291.
The newborn suffers from improper milk digestion due to lower activity of rennin.
Rennin acts on casein of milk in infants in presence of
A. Mg++
B. Zn++
C. Co++
D. * Ca++
E. Both C and D
292.
The patient's Ritis' ratio (AsAT\AlAT) equals 0,8. Which disease does it testify?
A. * Hepatitis
B. Pancreatitis
C. Gastritis
D. Cardiac infarction
E. Nephritis
293.
The patient's Ritis' ratio (AsAT\AlAT) equals 2,8. Which disease does it testify?
A. Hepatitis
B. Pancreatitis
C. Gastritis
D. * Cardiac infarction
E. Nephritis
294.
What type of covalent bonds link the amino acids in a protein?
A. * Peptide bonds
B. Hydrogen bonds
C. Ionic bonds
D. Glycosidic bonds
E. Ester bonds
295.
Which of the following is not an amino acid?
A. Glutamic acid
B. Aspartic acid
C. Glutamine
D. * Palmitic acid
E. Leucine
296.
A dietary deficiency of tryptophan and nicotinate leads to
A. Beri Beri
B. Xerophthalmia
C. Anemia
D. * Pellegra
E. Rickets
297.
All the following statements about pepsin are correct, except
A. It is smaller than pepsinogen
B. It is formed by the action of HCl on its precursor
C. Its optimum pH is 1.0–2.0
D. * It hydrolyses the C-terminal and N-terminal peptide bonds of proteins
E. Its optimum pH is 10–12
298.
Along with CO2, NH3 and ATP the aminoacid that is needed in urea cycle is
A. Alanine
B. Isoleucine
C. * Aspartate
D. Glycine
E. Valine
299.
ATP is required in following reactions of urea cycle:
A. Synthesis of carbamoyl phosphate and citrulline
B. * Synthesis of citrulline and argininosuccinate
C. Synthesis of argininosuccinate and arginine
D. Synthesis of carbamoyl phosphate and argininosuccinate
E. All of the above
300.
by carbamoyl phosphate synthetase I is incorrect?
A. The enzyme catalyzes the rate-limiting reaction in the urea cycle.
B. * Raction is allosterically activated by N-acetylglutamate.
C. The reaction is reversible.
D. The enzyme incorporates CO2 into carbamoyl phosphate.
E. The reaction requires two high energy phosphates for each carbamoyl phosphate
molecule synthesized.
301.
Child, who for a long time had products of vegetable origin, has growth
inhibition, anemia, liver and kidney affection, skin and hair reddening. The cause of this
condition is:
A.
B.
C.
D.
E.
Lack of lipids in food
* Lack of essential amino acid in food
Lack of carbohydratesin food
Lack of macrocells in food
Lack of fats in food
302.
?Control of urea cycle involves the enzyme:
A. * Carbamoyl phosphate synthetase
B. Ornithine transcarbamoylase
C. Argininosuccinase
D. Arginase
E. Fumarase
303.
Cyanide, a potent human toxin, blocks electron flow in electron transport by
binding to Fe2+ . The Fe2+ ion is found in which of the following?
A. Coenzyme Q
B. FADH2
C. Ubiquinone
D. Acetyl – SCoA
E. * Catalase
304.
Cystinuria results from inability to
A. Metabolise cysteine
B. Convert cystine into cysteine
C. Incorporate cysteine into proteins
D. * Reabsorb cystine in renal tubules
E. Only A
305.
Cytosolic and mitochondrial carbamoyl phosphate synthetase have the following
similarity:
A. Both use ammonia as a substance
B. Both provide carbamoyl phosphate for urea synthesis
C. * Both require N-acetylglutamate as an activator
D. Both are allosteric enzymes
E. Both provide carbamoyl phosphate for pirimidine synthesis
306.
Exopeptidases are:
A. Pepsin, trypsin
B. Elastase, colagenase
C. * Carboxypeptidases, aminopeptidases
D. Chymotrypsin, carboxypeptidases
E. Aminopeptidases, elastase
307.
For effective formation of urea in the liver with great intensivity must proceed
ornithinic cycle. In the other case the concentration of ammonia in blood grows rapidly.
By the lack of what amino acid in the food can evolve azotemia?
A. * Arginine
B. Alanine
C. Lysine
D. Serine
E. Methionine
308.
GABA (gamma aminobutyric acid), which belongs to inhibiting mediators of
central nervous system, is a product of glutamic decarboxylation. What vitamin
prescription is appropriate by convulsive state, caused by decrease if GABA formation?
A. B1
B. B9
C. * B6
D. B5
E. B2
309.
glucogenic amino acid?
A. Acetyl-CoA
B. Malonil-CoA
C. * Pyruvate
D. Acetoacetyl-CoA
E. All of the above
310.
Higher plants most often absorb nitrogen from the soil in the form of
A. N2
B. Nitrites.
C. Ammonia.
D. * Nitrates.
E. Amino acids.
311.
Hydroxylation of phenylalanine requires all of the following except
A. * Phenylalanine hydroxylase
B. Tetrahydrobiopterin
C. NADH
D. Molecular oxygen
E. FAD
312.
Into the hospital was carried 7 years old child in the state of allergic shock, which
evolved after wasp's bite. In the blood increased concentration of histamine. This amine
is formed as a result of reaction of:
A. Dehydration
B. Deamination
C. Reduction
D. * Decarboxylation
E. Hydrooxidation
313.
Isoelectric pH of an amino acid is that pH at which it has a
A. Positive charge
B. Negative charge
C. * No charge
D. None of these
E. All of the above
314.
Kwashiorkor occurs when the diet is severely deficient in
A. Iron
B. Calories
C. * Proteins
D. Essential fatty acids
E. Ca++
315.
Maple syrup urine diseases is an inborn error of metabolism of
A. Sulphur-containing amino acids
B. Aromatic amino acids
C. * Branched chain amino acids
D. Dicarboxylic amino acids
E. None of the above
316.
Name the matter in the urine, which is a test of intensivity of protein putrefaction
processes in the intestine:
A. Urates
B. Urea
C. Creatinine
D. * Indican
E. Lactic acid
317.
Negative nitrogenous balance is observed in all listed cases, but:
A. Kidney disease
B. Domination of animal proteins in food
C. Domination of vegetable proteins in food
D. * In old years
E. By pancreatitis
318.
Nitrogen gas accounts for what percentage of the atmosphere?
A. 1%
B. * 7%
C. 55%
D. 78%
E. 99%
319.
Nitrogen is essential for living organisms to make:
A. Lipids.
B. * Proteins.
C. Carbohydrates.
D. Sulfates.
E. Benzene rings.
320.
Observation of 45-years old man, who was on vegetarian diet for a long time,
showed negative nitric balance. What food allowance feature caused this event?
A. Excessive amount of carbohydrates
B. * Insufficient amount of proteins
C. Excessive amount of water
D. Insufficient amount of fats and proteins
E. Insufficient amount of fats
321.
One of the following amino acid is solely ketogenic:
A. * Lysine
B. Alanine
C. Valine
D. Glutamate
E. Arginine
322.
Pancreatic juice contains the precursors of all of the following except
A. * Trypsin
B. Chymotrypsin
C. Carboxypeptidase
D. Aminopeptidase
E. Elastase
323.
Patient has positive nitric balance. The reason of this can be:
A. Starvation
B. Tuberculosis
C. * Pregnancy
D. Malignant neoplasms
E. AIDS
324.
Patient has the diagnosis “malignant carcinoid”, amount of serotonine in blood
increased greatly. Name amino acid, able to form the given biogenic amine:
A. Alanine
B. * 5-oxytryptophane
C. Leucine
D. Threonine
E. Methionine
325.
Patient on a vegetarian diet has negative nitric balance, hyperproteinemia,
affection of colloid-osmotic pressure and water-salt metabolism by normal function of
alimentary canal. This was caused by:
A. * Monotonous protein nutrition
B. Monotonous carbohydrate nutrition
C. Lack of unsaturated fatty acids
D. Lack of phospholipids in food
E. Lack of vitamins in food
326.
Serines are all names of families of what?
A. Carbohydrates
B. Fats
C. * Amino acids
D. Pyridines
E. Vitamins
327.
Sulphur-containing amino acid is
A. Glutathione
B. Chondroitin sulphate
C. * Homocysteine
D. Tryptophan
E. Histidin
328.
The amount of urea in the patient's blood is 16 mmol/l, in daily urine 6 g. The
possible reason for this is the following:
A. Liver disease
B. Starvation
C. * Kidney disease
D. Acidosis
E. Thyrotoxicosis
329.
The defective enzyme in histidinemia is
A. Histidine carboxylase
B. Histidine decarboxylase
C. * Histidase
D. Histidine oxidase
E. None of the above
330.
The process where some bacteria remove nitrate from the soil by converting it to
nitrogen gas is:
A. Nitrification.
B. Ammonification.
C. Assimilation.
D. * Denitrification.
E. Nitrogen fixation.
331.
The reservoir for nitrogen is
A. The atmosphere.
B. Rocks.
C. Ammonia.
D. * Nitrates.
E. Amino acids.
332.
The ruminant obtains nitrogen primarily from
A. * Plant protein
B. The microbes of the gastrointestinal tract
C. Urea
D. Fatty acids produced in the rumen.
E. Carbohydrates
333.
The step in the nitrogen cycle where bacteria convert ammonia (NH3) to nitrate
(NO3-) is:
A. * Nitrification.
B. Ammonification.
C. Assimilation.
D. Denitrification.
E. Nitrogen fixation.
334.
The toxicity of ammonia, especially dangerous for brain, is connected with its'
ability to affect functioning of tricarboxylic acids cycle in mitichondria of brain neurons
as a result of conclusion from the cycle of:
A.
B. Citrate
C. Malate
D. Succinate
E. * Fumarate
335.
The useful reagent for detection of amino acids is
A. Molisch reagent
B. Dichlorophenol Indophenol
C. * Ninhydrin
D. Biuret
E. None of the above
336.
To endopeptidases belong all below-mentioned ferments, but:
A. Pepsin
B. Elastase
C. * Carboxypeptidase
D. Chemotrypsin
E. Trypsin
337.
What is the first step in the nitrogen cycle, in which gaseous nitrogen is converted
into ammonia?
A. * Nitrification
B. Ammonification
C. Assimilation
D. Denitrification
E. Nitrogen fixation
338.
What is the main transportation form of ammonia from the majority of peripheral
tissues to the liver?
A. Urea
B. Aminosuccinamic
C. Citrulline
D. Ornithine
E. * Glutamine
339.
What is/are true regarding the urea cycle?
A. It plays an important role in nitrogen metabolism
B. It has a role in both the catabolism and anabolism of amino acids
C. * It is linked to the citric acid cycle
D. All of the above
E. None of the above
340.
What major way of ammonia deactivation, which formed in the organism as a
result of deamination reactions?
A. Glutamine synthesis
B. Ammonium salts synthesis
C. * Urea synthesis
D. Uric acid synthesis
E. Aminosuccinamic synthesis
341.
What part of the nitrogen cycle deals with the conversion of nitrogen in waste
products or dead organisms into ammonia?
A. Nitrification
B. * Ammonification
C. Assimilation
D. Denitrification
E. Nitrogen fixation
342.
Which amino acid is a lipotropic factor?
A. Lysine
B. Lecuine
C. Tryptophan
D. * Methionine
E. Valine
343.
Which of the following contributes nitrogen atoms to both purine and pyrimidine
rings?
A. * Aspartate
B. Carbamoyl phosphate
C. CO2
D. Glutamine
E. Fumarate
344.
Which of the following molecules will be formed from the carbon skeleton of a
ketogenic amino acid?
A. * Acetyl-CoA
B. Oxaloacetate
C. Pyruvate
D. ?-ketoglutarate
E. All of the above
345.
Which of the following protein is rich in cysteine?
A. Elastine
B. * Collagen
C. Fibrin
D. Keratin
E. None of the above
346.
Which of the following statements is (are) correct regarding nitrogen pollution?
A. Fresh water bodies are more often nitrogen limited than salt water bodies
B. Dissolved ammonia gas can be directly toxic to fish
C. In an undisturbed forest, nitrogen losses per year are generally less than the
amount of nitrogen contributed from the atmosphere.
D. In temperate regions, the greatest potential for leaching of nitrate occurs during
the fall and winter, when evapotranspiration is low relative to precipitation
E. * Answers 2, 3, and 4 above are all correct
347.
Which of the following statements is correct?
A. The two nitrogen atoms that are incorporated into urea enter the cycle as ammonia
and alanine.
B. Urea is produced directly by the hydrolysis of ornithine.
C. ATP is required for the reaction in which argininosuccinate is cleaved to form
arginine.
D. The urea cycle occurs exclusively in the cytosol.
E. * In humans, the major route of nitrogen metabolism from amino acids to urea is
catalyzed by the combined actions of transaminase (aminotransferase) and
glutamate dehydrogenase.
348.
Which of the following would most likely act as an inhibitor of the enzymes of the
citric acid cycle?
A. * A high concentration of ADP
B. A high concentration of ATP
C. A low concentration of NADH
D. A high concentration of NAD
E. More than one of the above.
349.
Which one of the following is an essential amino acid?
A. Arginine
B. Tyrosine
C. * Phenylalanine
D. Proline
E. Ornitine
350.
Why is nitrapyrin (2-chloro-6-trichloromethylpyridine) added to ammonia
fertilizers?
A. To convert the ammonia to nitrate which is the preferred nitrogen source for
plants.
B. * To solubilize the ammonia.
C. To inhibit nitrifying bacteria that convert ammonia to nitrite.
D. To inhibit assimilatory nitrate reduction.
E. None of the above
Test questions for figures
1. The fragment of which compound is shown in figure 106?
A. Cellulose
B. Triacylglycerol
C. * Starch
D. Protein
E. Phospholipid
2. Absorption of which compound can be described by the mechanism shown in figure 110?
A. Fatty acid
B. Glycerol
C. * Glucose
D. Fructose
E. Cholesterol
3. Compound shown in figure 107 consists of:
A. Glucose and glucose
B. Glucose and galactose
C. Glucose and mannose
D. Galactose and fructose
E. * Glucose and fructose
4. Compound shown in figure 108 consists of:
A. Glucose and glucose
B. * Glucose and galactose
C. Glucose and mannose
D. Galactose and fructose
E. Glucose and fructose
5. Compound shown in figure 109 consists of:
A. * Glucose and glucose
B. Glucose and galactose
C. Glucose and mannose
D. Galactose and fructose
E. Glucose and fructose
6. Compound which is formed in the reaction shown in figure 154 plays role of:
A. Mediator of inflammation
B. Mediator of allergic reaction
C. * Mediator of nervous system
D. Mediator of pain
E. Mediator of blood coagulation
7. Degradation of glycogen is shown in figure 129. This mechanism is called:
A. Hydrolysis
B. * Phosphorolysis
C. Ligation
D. Dephosphorylation
E. Deglycosidation
8. Fragment of what biochemical pathway is shown in figure 132?
A. * Glycogenogenesis
B. Glycogenolysis
C. Glycolysis
D. Gluconeogenesis
E. Pentose phosphate pathway
9. How many molecules of ATP are formed in substrate level phosphorylation reactions in
pathway shown in figure 112?
A. 2
B. * 4
C. 6
D. 8
E. 10
10. How many substrate level phosphorylation reactions are there in pathway shown in figure
112?
A. 1
B. * 2
C. 3
D. 4
E. 5
11. One of the products of reaction shown in figure 116 is:
A. Fructose-6-phosphate
B. 2-phosphoglycerate
C. * Glyceraldehyde 3-phosphate
D. Fructose-1,6-biphosphate
E. Pyruvate
12. One of the purposes of biochemical pathway shown in figure 126 is:
A. Production of NADH and FADH2
B. Energetic role
C. * Metabolism of unusual sugars
D. Decomposition of waste products
E. Metabolism of ketone bodies
13. One of the reactions of glycogen degradation is shown in figure 130. What enzyme
catalyzes this reaction?
A. Glycogen synthase
B. Glycogen phosphorylase
C. Isomerase
D. * Phosphoglucomutase
E. Hexokinase
14. Red blood cells with Heinz bodies are shown in figures 128. Dark particles (Heinz
bodies) are denaturated proteins adhered to cell membranes. For which disease such
micrograph is specific?
A. Lactase deficiency
B. * Glucose-6-phosphate dehydrogenase deficiency
C. Hexokinase deficiency
D. Pyruvate kinase deficiency
E. Pyruvate dehydrogenase deficiency
15. The biochemical pathway shown in figure 155 is called:
A. Tricarboxylic acid cycle
B. Kori cycle
C. * Urea cycle
D. Fatty acids synthesis cycle
E. Pentose phosphate cycle
16. The bonds indicated by question-mark in figure 106 are called:
A. Peptide
B. * Glycosidic
C. Ester
D. Phosphate
E. Noncovalent
17. The clinical symptoms shown in figure 157 are specific for:
A. Phenylketonuria
B. Albinism
C. Homocysteinuria
D. * Alcaptonuria
E. Maple syrup urine disease
18. The compound shown in figure 143 is digested by:
A. Lipase
B. Pepsin
C. Trypsin
D. Phospholipase
E. * Esterase
19. The first reaction of glycerol oxidation pathway is shown in figure 144. What enzyme
catalyzes this reaction?
A. Glycerol dehydrogenase
B. * Glycerol kinase
C. Glycerol carboxylase
D. Glycerol decarboxylase
E. Glycerol carboxykinase
20. The first reaction of glycerol oxidation pathway is shown in figure 144. What is the
product of this reaction?
A. Glyceraldehyde 3-phosphate
B. Dihydroxyacetone phosphate
C. * Glycerol 3-phosphate
D. Glycerol 2-phosphate
E. 3-phosphoglycerate
21. The fragment of which biochemical process is shown in figure 151?
A. Oxidation of fatty acids
B. * Synthesis of fatty acids
C. Activation of fatty acids
D. Oxidation of glycerol
E. Digestion of phospholipids
22. The molecule of taurocholic acid is depicted in figure 141. Such molecule is called:
A. Hydrophobic
B. Hydrophilic
C. * Amphipathic
D. Polar
E. Nonpolar
23. The product of reaction shown in figure 113 is:
A. Fructose-6-phosphate
B. 2-phosphoglycerate
C. * Glucose-6-phosphate
D. Fructose-1,6-biphosphate
E. Pyruvate
24. The product of reaction shown in figure 114 is:
A. * Fructose-6-phosphate
B. 2-phosphoglycerate
C. Glucose-6-phosphate
D. Fructose-1,6-biphosphate
E. Pyruvate
25. The product of reaction shown in figure 115 is:
A. Fructose-6-phosphate
B. 2-phosphoglycerate
C. Glucose-6-phosphate
D. * Fructose-1,6-biphosphate
E. Pyruvate
26. The product of reaction shown in figure 117 is:
A. Fructose-6-phosphate
B. 2-phosphoglycerate
C. * Glyceraldehyde 3-phosphate
D. Fructose-1,6-biphosphate
E. Pyruvate
27. The product of reaction shown in figure 118 is:
A. * 1,3-biphosphoglycerate
B. 2-phosphoglycerate
C. Glyceraldehyde 3-phosphate
D. Fructose-1,6-biphosphate
E. Pyruvate
28. The product of reaction shown in figure 119 is:
A. 1,3-biphosphoglycerate
B. 2-phosphoglycerate
C. Glyceraldehyde 3-phosphate
D. Fructose-1,6-biphosphate
E. * 3-phosphoglycerate
29. The product of reaction shown in figure 120 is:
A. 1,3-biphosphoglycerate
B. * 2-phosphoglycerate
C. Glyceraldehyde 3-phosphate
D. Fructose-1,6-biphosphate
E. 3-phosphoglycerate
30. The product of reaction shown in figure 121 is:
A. 1,3-biphosphoglycerate
B. 2-phosphoglycerate
C. Glyceraldehyde 3-phosphate
D. * Phosphoenolpyruvate
E. 3-phosphoglycerate
31. The product of reaction shown in figure 122 is:
A. * Pyruvate
B. 2-phosphoglycerate
C. Glyceraldehyde 3-phosphate
D. Phosphoenolpyruvate
E. 3-phosphoglycerate
32. The product of reaction shown in figure 135 is:
A. Glucose
B. Phosphoenolpyruvate
C. Citrate
D. Acetyl CoA
E. * Oxaloacetate
33. The product of reaction shown in figure 136 is:
A. Glucose
B. * Phosphoenolpyruvate
C. Citrate
D. Acetyl CoA
E. Oxaloacetate
34. The product of the reaction shown in figure 142:
A. Is a building block of cell membranes
B. * Can disrupt cellular membranes
C. Has neurotoxic properties
D. Is an intracellualar messenger
E. Activates transcription in cell nucleus
35. The reaction depicted in figure 158 is characteristic for:
A. * Phenylketonuria
B. Albinism
C. Homocysteinuria
D. Alcaptonuria
E. Maple syrup urine disease
36. The reaction shown in figure 113 comes from:
A. * Glycolysis
B. Gluconeogenesis
C. Oxidation of fatty acids
D. Glycogenolysis
E. Oxidation of glycerol
37. The reaction shown in figure 113 is catalyzed by:
A. * Hexokinase
B. Isomerase
C. Phosphofructokinase
D. Mutase
E. Pyruvatekinase
38. The reaction shown in figure 114 comes from:
A. * Glycolysis
B. Gluconeogenesis
C. Oxidation of fatty acids
D. Glycogenolysis
E. Oxidation of glycerol
39. The reaction shown in figure 114 is catalyzed by:
A. Hexokinase
B. * Isomerase
C. Phosphofructokinase
D. Mutase
E. Pyruvatekinase
40. The reaction shown in figure 115 comes from:
A. * Glycolysis
B. Gluconeogenesis
C. Oxidation of fatty acids
D. Glycogenolysis
E. Oxidation of glycerol
41. The reaction shown in figure 115 is catalyzed by:
A. Hexokinase
B. Isomerase
C. * Phosphofructokinase-1
D. Mutase
E. Pyruvatekinase
42. The reaction shown in figure 116 comes from:
A. * Glycolysis
B. Gluconeogenesis
C. Oxidation of fatty acids
D. Glycogenolysis
E. Oxidation of glycerol
43. The reaction shown in figure 116 is catalyzed by:
A. * Aldolase
B. Isomerase
C. Phosphofructokinase-1
D. Mutase
E. Pyruvatekinase
44. The reaction shown in figure 117 comes from:
A. * Glycolysis
B. Gluconeogenesis
C. Oxidation of fatty acids
D. Glycogenolysis
E. Oxidation of glycerol
45. The reaction shown in figure 117 is catalyzed by:
A. Aldolase
B. * Isomerase
C. Phosphofructokinase-1
D. Mutase
E. Pyruvatekinase
46. The reaction shown in figure 118 comes from:
A. * Glycolysis
B. Gluconeogenesis
C. Oxidation of fatty acids
D. Glycogenolysis
E. Oxidation of glycerol
47. The reaction shown in figure 118 is catalyzed by:
A. Aldolase
B. Isomerase
C. Phosphofructokinase-1
D. Mutase
E. * Dehydrogenase
48. The reaction shown in figure 119 comes from:
A. * Glycolysis
B. Gluconeogenesis
C. Oxidation of fatty acids
D. Glycogenolysis
E. Oxidation of glycerol
49. The reaction shown in figure 119 is catalyzed by:
A. * Phosphoglycerate kinase
B. Isomerase
C. Phosphofructokinase-1
D. Mutase
E. Dehydrogenase
50. The reaction shown in figure 120 comes from:
A. * Glycolysis
B. Gluconeogenesis
C. Oxidation of fatty acids
D. Glycogenolysis
E. Oxidation of glycerol
51. The reaction shown in figure 120 is catalyzed by:
A. Phosphoglycerate kinase
B. Isomerase
C. Phosphofructokinase-1
D. * Mutase
E. Dehydrogenase
52. The reaction shown in figure 121 comes from:
A. * Glycolysis
B. Gluconeogenesis
C. Oxidation of fatty acids
D. Glycogenolysis
E. Oxidation of glycerol
53. The reaction shown in figure 121 is catalyzed by:
A. Phosphoglycerate kinase
B. Isomerase
C. * Enolase
D. Mutase
E. Dehydrogenase
54. The reaction shown in figure 122 comes from:
A. * Glycolysis
B. Gluconeogenesis
C. Oxidation of fatty acids
D. Glycogenolysis
E. Oxidation of glycerol
55. The reaction shown in figure 122 is catalyzed by:
A. Phosphoglycerate kinase
B. Isomerase
C. * Pyruvate kinase
D. Mutase
E. Dehydrogenase
56. The reaction shown in figure 127 comes from:
A. Glycolysis
B. Gluconeogenesis
C. Oxidation of fatty acids
D. Glycogenolysis
E. * Pentose phosphate pathway
57. The reaction shown in figure 135 comes from:
A. Glycolysis
B. * Gluconeogenesis
C. Oxidation of fatty acids
D. Glycogenolysis
E. Oxidation of glycerol
58. The reaction shown in figure 136 comes from:
A. Glycolysis
B. * Gluconeogenesis
C. Oxidation of fatty acids
D. Glycogenolysis
E. Oxidation of glycerol
59. The reaction shown in figure 137 comes from:
A. Glycolysis
B. * Gluconeogenesis
C. Oxidation of fatty acids
D. Glycogenolysis
E. Oxidation of glycerol
60. The reaction shown in figure 138 comes from:
A. Glycolysis
B. * Gluconeogenesis
C. Oxidation of fatty acids
D. Glycogenolysis
E. Oxidation of glycerol
61. The reaction shown in figure 152 is catalyzed by:
A. * Aminotransferase
B. Dehydrogenase
C. Decarboxylase
D. Oxidase
E. Reductase
62. The reaction shown in figure 153 is catalyzed by:
A. Aminotransferase
B. Dehydrogenase
C. * Decarboxylase
D. Oxidase
E. Reductase
63. The scheme of fate of glucose is shown in figure 111. Which metabolite is replaced with
digit 1?
A. Fructose-6-phosphate
B. * Glucose-6-phosphate
C. Lactate
D. Fructose-1,6-biphosphate
E. Phosphoenol pyruvate
64. The scheme of fate of glucose is shown in figure 111. Which pathway is indicated by
digit 2?
A. Glycolysis
B. Gluconeogenesis
C. Glycogenogenesis
D. Glycogenolysis
E. * Pentose phosphate pathway
65. The scheme of fate of glucose is shown in figure 111. Which pathway is indicated by
digit 3?
A. * Glycolysis
B. Gluconeogenesis
C. Glycogenogenesis
D. Glycogenolysis
E. Pentose phosphate pathway
66. The scheme of fate of pyruvate is shown in figure 123. Which metabolite is indicated by
digit 1?
A. Lactate
B. Phosphoenolpyruvate
C. * Ethanol
D. Propynol
E. Methanol
67. The scheme of fate of pyruvate is shown in figure 123. Which metabolite is indicated by
digit 2?
A. * Lactate
B. Phosphoenolpyruvate
C. Ethanol
D. Propynol
E. Methanol
68. The scheme of the reaction of is shown in figure 152.
A. Decarboxylation of amino acids
B. Deamination of amino acids
C. * Transamination of amino acids
D. Oxidation of amino acids
E. Reduction of amino acids
69. The scheme of the reaction of is shown in figure 153.
A. * Decarboxylation of amino acids
B. Deamination of amino acids
C. Transamination of amino acids
D. Oxidation of amino acids
E. Reduction of amino acids
70. The scheme of the regulation of glycogen phosphorylase and glycogen synthase by
epinephrine and glucagone is shown in figure 133. What enzyme is replaced with digit 1
on this scheme?
A. * Protein kinase
B. Phosphatase
C. Phospholipase
D. Hexokinase
E. Phosphorylase
71. The scheme of the regulation of glycogen phosphorylase and glycogen synthase by
epinephrine and glucagone is shown in figure 133. What enzyme is replaced with digit 2
on this scheme?
A. Protein kinase
B. Phosphatase
C. * Glycogen synthase b
D. Hexokinase
E. Glycogen phosphorylase a
72. The scheme of the regulation of glycogen phosphorylase and glycogen synthase by
epinephrine and glucagone is shown in figure 133. What enzyme is replaced with digit 3
on this scheme?
A. Protein kinase
B. Phosphatase
C. Glycogen synthase b
D. Hexokinase
E. * Glycogen phosphorylase a
73. The scheme of which biochemical process is shown in figure 146?
A. * Transport of fatty acyl CoA into mitochondria
B. Transport of fatty acyl CoA into cytoplasm
C. Transport of carnitine into mitochondria
D. Transport of carnitine into cytoplasm
E. Transport of HS-CoA into mitochondria
74. The scheme shown in figure 139 is called:
A. Krebs cycle
B. * Cori cycle
C. Mitchell cycle
D. Fisher cycle
E. Horbachevsky cycle
75. The second bypass of gluconeogenesis is shown in figure 137. What enzyme catalyzes
this reaction?
A. Protein kinase
B. Phosphofructokinase
C. Fructose dehydrogenase
D. * Phosphatase
E. Hexokinase
76. The third bypass of gluconeogenesis is shown in figure 138. What enzyme catalyzes this
reaction?
A. Protein kinase
B. Phosphofructokinase
C. Fructose dehydrogenase
D. * Phosphatase
E. Hexokinase
77. What are the end products of biochemical process shown in figure 142?
A. Triacylglycerol and free fatty acids
B. Diacylglycerol and free fatty acids
C. Free fatty acids
D. Free fatty acids and monoacylglycerol
E. * Lysophosphoglyceride and free fatty acid
78. What are the end products of biochemical process shown in figure 140?
A. Triacylglycerols
B. Diacylglycerols
C. Free fatty acids
D. * Free fatty acids and monoacylglycerol
E. Cholesterol and free fatty acids
79. What biochemical pathway is shown in figure 112?
A. * Glycolysis
B. Gluconeogenesis
C. Glycogenogenesis
D. Glycogenolysis
E. Pentose phosphate pathway
80. What biochemical pathway is shown in figure 126?
A. Glycolysis
B. Gluconeogenesis
C. Oxidation of fatty acids
D. Glycogenolysis
E. * Pentose phosphate pathway
81. What biochemical pathway is shown in figure 131?
A. * Glycogenogenesis
B. Glycogenolysis
C. Glycolysis
D. Gluconeogenesis
E. Pentose phosphate pathway
82. What biochemical pathway is shown in figure 134?
A. Glycogenogenesis
B. Glycogenolysis
C. Glycolysis
D. * Gluconeogenesis
E. Pentose phosphate pathway
83. What biochemical process is shown in figure 140?
A. Digestion of glycogen
B. Digestion of proteins
C. * Digestion of triacylglycerols
D. Digestion of phospholipids
E. Digestion of cholesterol esters
84. What biochemical process is shown in figure 142?
A. Digestion of glycogen
B. Digestion of proteins
C. Digestion of triacylglycerols
D. * Digestion of phospholipids
E. Digestion of cholesterol esters
85. What biochemical process is shown in figure 145?
A. Digeston of fatty acids
B. Synthesis of fatty acids
C. * Activation of fatty acids
D. Digestion of triacylglycerols
E. Digestion of phospholipids
86. What coenzyme is required for the reaction shown in figure 147?
A. NAD
B. * FAD
C. TPP
D. NADP
E. HS-CoA
87. What coenzyme is required for the reaction shown in figure 149?
A. * NAD
B. FAD
C. TPP
D. NADP
E. HS-CoA
88. What coenzyme is required for the reaction shown in figure 150?
A. NAD
B. FAD
C. TPP
D. NADP
E. * HS-CoA
89. What coenzyme is required for the reaction shown in figure 125?
A. * NAD
B. FAD
C. TPP
D. PLP
E. HS CoA
90. What coenzyme is required for the reaction shown in figure 127?
A. NAD
B. FAD
C. TPP
D. * NADP
E. HS CoA
91. What compound is eliminated in the 1st reaction of biochemical pathway shown in figure
124?
A. O2
B. * CO2
C. H3PO4
D. H2
E. NADH
92. What compound is replaced with digit 1 in biochemical pathway shown in figure 134?
A. Glucose
B. Phosphoenolpyruvate
C. Citrate
D. Acetyl CoA
E. * Oxaloacetate
93. What compound is replaced with digit 2 in figure 131?
A. ATP
B. NAD
C. * UTP
D. FAD
E. HSCoA
94. What compound is shown in figure 107?
A. Cellulose
B. Maltose
C. * Sucrose
D. Lactose
E. Glycogen
95. What compound is shown in figure 108?
A. Cellulose
B. Maltose
C. Sucrose
D. * Lactose
E. Glycogen
96. What compound is shown in figure 109?
A. Cellulose
B. * Maltose
C. Sucrose
D. Lactose
E. Glycogen
97. What enzyme catalysis the reaction shown in figure 147?
A. Acyl CoA hydratase
B. Acyl CoA dehydratase
C. * Acyl CoA dehydrogenase
D. Acyl CoA transferase
E. Acyl CoA oxidase
98. What enzyme catalysis the reaction shown in figure 148?
A. * Hydratase
B. Dehydratase
C. Dehydrogenase
D. Transferase
E. Oxidase
99. What enzyme catalysis the reaction shown in figure 149?
A. Hydratase
B. Dehydratase
C. * Dehydrogenase
D. Transferase
E. Oxidase
100.
What enzyme catalysis the reaction shown in figure 150?
A. Hydratase
B. Dehydratase
C. Dehydrogenase
D. Transferase
E. * Thiolase
101.
What enzyme catalyzes the reaction #1 in pathway shown in figure 124?
A.
B.
C.
D.
E.
102.
A.
B.
C.
D.
E.
103.
A.
B.
C.
D.
E.
104.
A.
B.
C.
D.
E.
105.
A.
B.
C.
D.
E.
106.
131?
A.
B.
C.
D.
E.
107.
134?
A.
B.
C.
D.
E.
108.
134?
A.
B.
C.
D.
E.
109.
134?
Lactate dehydrogenase
Isomerase
Pyruvate kinase
Mutase
* Pyruvate decarboxylase
What enzyme catalyzes the reaction #2 in pathway shown in figure 124?
Lactate dehydrogenase
Isomerase
Pyruvate kinase
Mutase
* Alcohol dehydrogenase
What enzyme catalyzes the reaction shown in figure 145?
* Acyl-CoA synthetase
Acyl-CoA dehydrogenase
Acyl-CoA oxidase
Acyl-CoA lyase
Acyl-CoA isomerase
What enzyme catalyzes the reaction shown in figure 156?
carbamoyl phosphate synthetase
Ornithine carbamoyltransferase
Argininosuccinate synthetase
Argininosuccinate lyase
* Arginase
What enzyme catalyzes the reaction shown in figure 125?
* Lactate dehydrogenase
Isomerase
Pyruvate kinase
Mutase
Alcohol dehydrogenase
What enzyme is indicated by the digit 1 in biochemical pathway shown in figure
* Glycogen synthase
Glycogen phosphorylase
Isomerase
Phosphoglucomutase
Hexokinase
What enzyme is replaced with digit 2 in biochemical pathway shown in figure
Protein kinase
* Pyruvate carboxylase
Pyruvate decarboxylase
Phosphatase
Lactate dehydrogenase
What enzyme is replaced with digit 3 in biochemical pathway shown in figure
Protein kinase
Phosphofructokinase
Fructose dehydrogenase
* Phosphatase
Lactate dehydrogenase
What enzyme is replaced with digit 4 in biochemical pathway shown in figure
A.
B.
C.
D.
E.
Protein kinase
Phosphofructokinase
Fructose dehydrogenase
* Phosphatase
Hexokinase
110.
What enzyme participates in the biochemical process shown in figure 142?
A. Lipase
B. Pepsin
C. Trypsin
D. * Phospholipase
E. Esterase
111.
What enzyme participates in the biochemical process shown in figure 140?
A. * Lipase
B. Pepsin
C. Trypsin
D. Phospholipase
E. Esterase
112.
What enzyme participates in the reaction shown in figure 129?
A. Glycogen synthase
B. * Glycogen phosphorylase
C. Branching enzyme
D. Phosphoglucomutase
E. Hexokinase
113.
What is the clear output of ATP molecules in biochemical pathway shown in
figure 112?
A. 5
B. * 7
C. 8
D. 9
E. 10
114.
What is the end product of biochemical pathway shown in figure 155?
A. Acetyl CoA
B. Pyruvate
C. Lactate
D. Glycerol
E. * Urea
115.
What is the end product of biochemical pathway shown in figure 131?
A. Glucose
B. Fructose
C. Glucose 6-phosphate
D. Glucose 1-phosphate
E. * Glycogen
116.
What is the main purpose of biochemical pathway shown in figure 126?
A. Production of NADH and FADH2
B. Energetic role
C. * Production of NADPH and pentoses
D. Decomposition of waste products
E. Metabolism of ketone bodies
117.
What is the product (#3) of biochemical pathway shown in figure 124?
A. Lactate
B. Phosphoenolpyruvate
C. * Ethanol
D. Propynol
E. Methanol
118.
What is the product of reaction shown in figure 125?
A. * Lactate
B. Phosphoenolpyruvate
C. Ethanol
D. Propynol
E. Methanol
119.
What product is formed in the fatty acid oxidation reaction shown in figure 147?
A. Acetyl CoA
B. * Enoyl CoA
C. 3-Hydroxyacyl CoA
D. 3-Ketoacyl CoA
E. Acyl CoA
120.
What product is formed in the fatty acid oxidation reaction shown in figure 148?
A. Acetyl CoA
B. Enoyl CoA
C. * 3-Hydroxyacyl CoA
D. 3-Ketoacyl CoA
E. Acyl CoA
121.
What product is formed in the fatty acid oxidation reaction shown in figure 149?
A. Acetyl CoA
B. Enoyl CoA
C. 3-Hydroxyacyl CoA
D. * 3-Ketoacyl CoA
E. Acyl CoA
122.
What product is formed in the reaction of activation of fatty acid shown in figure
145?
A. Triacylglycerol
B. Diacylglycerol
C. Monoacylglycerol
D. * Acyl CoA
E. Free fatty acid
123.
What product is formed in the reaction shown in figure 129?
A. Glucose
B. Fructose
C. Glucose 6-phosphate
D. * Glucose 1-phosphate
E. Fructose 1-phosphate
124.
What type of reaction is shown in figure 119?
A. Oxidative phosphorylation reaction
B. * Substrate level phosphorylation reaction
C. Oxidation-reduction reaction
D. Isomerization reaction
E. Ligation reaction
125.
What type of reaction is shown in figure 122?
A. Oxidative phosphorylation reaction
B. * Substrate level phosphorylation reaction
C. Oxidation-reduction reaction
D. Isomerization reaction
E. Ligation reaction
126.
Which coenzyme is required for the reaction presented in figure 152?
A.
B.
C.
D.
E.
NAD
FAD
TPP
NADP
* PLP
127.
Which coenzyme is required for the reaction presented in figure 153?
A. NAD
B. FAD
C. TPP
D. NADP
E. * PLP
128.
Which compound is formed in the decarboxylation reaction shown in figure 154?
A. Histamine
B. Glutamine
C. * GABA
D. Cadaverine
E. Putrescine
129.
Which compound is replaced with question-mark in the scheme depicted in the
figure 146?
A. Fatty acid
B. * Carnitine
C. HS-CoA
D. Acetyl-CoA
E. Citrate
130.
Which enzyme catalyzes the splitting of bond between two constituents of
compound shown in figure 107?
A. * Sucrase
B. Amilase
C. Pepsin
D. Maltase
E. Lactase
131.
Which enzyme catalyzes the splitting of bond between two constituents of
compound shown in figure 108?
A. Sucrase
B. Amilase
C. Pepsin
D. Maltase
E. * Lactase
132.
Which enzyme catalyzes the splitting of bond between two constituents of
compound shown in figure 109?
A. Sucrase
B. Amilase
C. Pepsin
D. * Maltase
E. Lactase
133.
Which enzyme catalyzes the splitting of bonds indicated by question-mark in
figure 106?
A. Sucrase
B. * Amilase
C. Pepsin
D. Phosphatase
E. Lactase
134.
112?
A.
B.
C.
D.
E.
135.
112?
A.
B.
C.
D.
E.
136.
112?
A.
B.
C.
D.
E.
137.
112?
A.
B.
C.
D.
E.
Which metabolite is replaced with digit 1 in biochemical pathway shown in figure
* Fructose-6-phosphate
Glucose-6-phosphate
1,3-biphosphoglycerate
Fructose-1,6-biphosphate
Phosphoenol pyruvate
Which metabolite is replaced with digit 2 in biochemical pathway shown in figure
Fructose-6-phosphate
Glucose-6-phosphate
* 1,3-biphosphoglycerate
Fructose-1,6-biphosphate
Phosphoenol pyruvate
Which metabolite is replaced with digit 3 in biochemical pathway shown in figure
Fructose-6-phosphate
* 2-phosphoglycerate
1,3-biphosphoglycerate
Fructose-1,6-biphosphate
Phosphoenol pyruvate
Which metabolite is replaced with digit 4 in biochemical pathway shown in figure
Fructose-6-phosphate
2-phosphoglycerate
1,3-biphosphoglycerate
Fructose-1,6-biphosphate
* Pyruvate